* Part 2 Boards *

Ace your homework & exams now with Quizwiz!

What is the minimum distance between adjacent implants? Implant and a natural tooth? Implant and mini-implant?

-3mm -1mm -2.4mm

What is the minimum vertical height of bone to place an implant?Minimum width of bone? Minimum apex distance from nerve?

-8mm -6mm -2mm

Strawberry tongue seen in which conditions?

-Scarlet Fever -Kawasaki disease -Toxic shock syndrome

What does Gonstead classify as "special listings"?

-Sp(spinous), -La(lamina), -Tp(transverse process), -M(mammillary)

What is the "junctional epithelium" composed of that is attached to the implant?

-hemidesmosomes and basal lamina

What makes up the pterygomandibular space?

-medial and inferior border is made up by medial muscle -lateral border is made up by the medial of ramus -posterior is made up by the parotid gland -anterior is made up by the buccinator muscle

Scopolamine

-used for motion sickness -anticholenergic eye drop used to induce mydriasis (dilate pupil)

Apexification

-used on nonvital tooth -promotes closure of the apex with calcium hydroxide

Apexogenesis

-vital tooth but damaged coronal pulp, healthy radical pulp -promotes complete development of root

1 RAD = ___ REM

1

when performing rescue breathing on adults, give ___________ ventilations every __________ sec

1 and 5

AP lumbar posit?

1" below top of iliac crest Cass = 14x17 FFD= 40 Tilt = 0

Neuromusculoskeletal diagnosis

1. Case History (11%) 2. Posture and Gait Assessment (5%) 3. Orthopedic Examination (24%) Cervical spine Thoracic spine Lumbar spine and pelvis Extremities 4. Neurologic Examination (21%) Cranial nerve/CNS testing Motor testing Sensory testing Reflex testing Superficial reflexes Deep tendon reflexes Pathological reflexes 5. Diagnosis or Clinical Impression (39%) CNS disorders Peripheral neurologic/neurovascular disorders Muscular disorders Bone and joint disorders

General Diagnosis

1. Case History (8%) 2. Vital Signs (3%) 3. Head and Neck Examination/Findings (6%) Eye Ear Nose and sinuses Mouth and throat Neck 4. Thorax Examination/Findings (including heart and lungs) (7%) Inspection Auscultation Percussion Palpation 5. Abdominal Examination/Findings (7%) Inspection Auscultation Percussion Palpation 6. Rectal and Male Urogenital Examination/Findings (4%) 7. Diagnosis or Clinical Impression (55%) Eyes, ears, nose and throat Respiratory diseases Cardiovascular diseases Blood and lymphatic diseases Gastrointestinal diseases Genitourinary diseases Endocrine and metabolic disorders Immunological disorders Nutritional disorders Infectious diseases 8. Laboratory Interpretation (10%) Urinalysis Hematology Chemistry/serology Special studies

associated clinical science

1. Geriatrics (14%) Anatomic and physiologic process of aging Geriatric disorders and case management 2. Dermatology (11%) Types of skin lesions Microbial infections Tumors Dermatitis Scaling diseases Vesicular and bullous diseases Pigmentary disorders Inflammatory reactions Disorders of hair follicles and sebaceous glands 3. Sexually Transmitted Diseases (9%) Gonorrhea Syphilis AIDS Chlamydia Herpes Other 4. Toxicology (9%) Pharmacology Poisoning Environmental and occupational toxicology 5. Emergency Procedures (13%) Cardiopulmonary arrest Poisoning Thermal injuries Shock Head and spinal injuries Choking Fractures, dislocations, strains and sprains Other 6. Psychology (11%) Theories and general concepts Mental and emotional disorders Assessment methods and case management 7. Gynecology and Obstetrics (13%) Gynecological examination Breast examination and disorders Gynecological disease and abnormalities Pregnancy Labor and delivery Postpartum Obstetric disorders 8. Pediatrics (14%) Care of newborn Growth and development Congenital disorders Childhood infectious diseases Pathological conditions and childhood injuries Emotional disorders and learning disabilities Nutritional problems 9. Jurisprudence, Ethics and Basic Economics (6%)

chiropractic practice

1. Spinal Analysis and Patient Evaluation (27%) Case history and observation findings Manual examination of the spine and extremities Diagnostic procedures 2. Chiropractic Adjustive Technique (28%) Cervical techniques Thoracic techniques Lumbar techniques Pelvic techniques Costal and extremity techniques 3. Patient Care (25%) Case management Contraindications for treatment Home care, prevention and rehabilitation 4. Community Health and Wellness (11%) Public health organizations Healthy People initiatives Screening activities for health promotion Tobacco, alcohol and substance abuse Exercise and healthy diet for obesity Behavior theories and lifestyle change Wellness counseling 5. Occupational and Environmental Health (9%) Work-based health risks Worker protection and ergonomics Injury and violence Impact of environment on human health Pollution and wastes

principles of chiropractic

1. The Chiropractic Paradigm (6%) 2. Concepts of Subluxation and Spinal Lesions (30%) Proprioceptive insult/somatosomatic reflex models Neural compression/traction models Visceral reflex models Vascular insufficiency models Axonal aberration/trophic models Neuroimmunomodulation models Biomechanical models Other 3. Basic Science Concepts in Chiropractic (32%) Anatomical Pathophysiological Biomechanical 4. Applied Chiropractic Principles (32%) Subluxation etiologies General effects of adjustment and manipulation Wellness

diagnostic imaging

1. X-ray Technology (10%) Physics and principles Quality assurance Radiographic imaging Radiologic protection 2. Radiographic Positioning and Normal Anatomy (21%) Spine and pelvis Extremities Thorax and abdomen 3. Imaging Diagnosis or Clinical Impression (50%) Congenital anomalies and normal variants Trauma Arthritides Tumors/tumor-like processes Nutritional, metabolic and endocrine conditions Hematologic conditions Infectious conditions Miscellaneous skeletal conditions Thoracic and abdominal conditions 4. Methods of Interpretation (8%) Biomechanical analysis/ mensuration procedures Imaging interpretation and report writing 5. Clinical Applications of Special Imaging (11%) MRI CT Miscellaneous

according to the inverse square law if a 40 inch FFD is doubled to an 80 inch FFD the intensity of the radiation is ______________?

1/4

Double the FFD = _____________ the radiation while half of the FFD = ______________ the redaiont

1/4; 4x

the rate of compression for adult CPR is what?

100 per minute

Normal range of respiratory rates for adults?

12-20

fracture

128 hz tuning fork

what is the normal Grid ratio found in a chiro office?

12:1

normal resp rate for 6-10 yo?

15-30

normal resp rate of 6mo- 3o

20-30

normal resp rate of newborn to 5 mo old?

25-40

What is the biological width of an implant?

3-4mm

How much leeway space for each side of mand and max?

3.1mm mand 1.3mm max

normal newborn resp rate?

30-50

What is the recommended re-evaluation after periodontal therapy?

4-6 weeks

If unconscious diabetic undergoes hypoglycemia what do you do?

50% dextrose IV

Normal ranges of Heart rate

60-100 BPM

Bell's Palsy affects what CN

7; facial nerve

Pn below knee

=disc pick flexion/distraction technique

lying of stomach or standing makes it better

=extension makes it better =Mckenzie ex

You are evaluating a patient 5 days after extraction of tooth #17. The patient complains of a severe throbbing pain that started yesterday, 4 days after extraction. The patient most likely has which of the following conditions? A. Dry socket B. Subperiosteal abscess C. Periapical periodontitis in tooth #18 D. Neuropathic pain

A, A dry socket (alveolar osteitis) occurs on the third to fourth day after extraction and, except for pain, does not have the classic signs of infection

Even though the state-of-the-art treatment for facial fractures is with internal rigid fixation using bone plates and screws, a proper occlusal relationship must be established prior to fixation of the bony segments if the reduction is to be satisfactory. A. True B. False

A, A proper occlusal relationship is a prerequisite for satisfactory bony reduction. This is most commonly accomplished by the use if intermaxillary fixation, or wiring the jaws closed, during surgery

The term ALARA refers to _____. A. Reducing patient exposure to as low as is reasonably achievable B. As little as Roentgen allowed, an algorithm for limiting patient exposure C. A legal requirement to optimize occupational exposure in dental radiology D. Optimizing image quality E. Reducing the costs of radiographic examinations

A, ALARA (As Low As Reasonably Achievable) is a concept for minimizing patient and occupational exposure.

Inhibiting a-glucosidase and reducing glucose absorption from the gastrointestinal tract is the mechanism of action of which drug? A. Acarbose B. Acetoheximide C. Glyburide D. Metformin E. Pioglitazone

A, All of the choices are oral hypoglycemic agents. Only acarbose inhibits α-glucosidase

Sodium reabsorption in the thick ascending limb of the loop of Henle is inhibited by which drug? A. Bumetanide B. Chlorthalidone C. Hydrochlorothiazide D. Spironolactone E. Triamterene

A, All the drugs listed are diuretics. However, only bumetanide acts on the ascending limb of the loop of Henle. It is called a "loop" or "high ceiling" diuretic because of its site of action in the nephron and maximal effect, respectively.

Root resorption is correlated to the pattern of stress distribution in the PDL and type of tooth movement. A. True B. False

A, Although somewhat controversial, it is believed that types of tooth movements that concentrate force in small areas of the PDL are more likely to result in root resorption during orthodontic treatment.

Which statement is false about behavioral contracts? A. It is a legal and binding agreement between health care professional and patient. B. It helps solidify an agreement with a patient. C. It should always be open to modification. D. It helps clarify agreements. E. The clinician should give a copy to the patient and keep one for himself or herself

A, Although the behavioral contract is not a legal document, it can be a useful approach in solidifying behavioral strategies and goals

A painless, well-circumscribed 1 ¥ 3-cm radiolucent lesion with radiopaque focus was found in the posterior mandible of an 11-year-old boy. Which of the following should be included in a differential diagnosis? A. Ameloblastic fibro-odontoma B. Paget's disease C. Dentigerous cyst D. Ameloblastoma E. Langerhans cell disease

A, Ameloblastic fibro-odontoma is the only lesion listed that is lucent with opaque foci. The patient's age is also characteristic for this lesion. Paget's disease may show a mixed opaquelucent pattern, but it occurs only over the age of 50 years

When faced with a frightened child patient, which would be the most appropriate or most effective response? A. Ask the child about his or her fears. B. Reschedule the appointment for a later date. C. Reassure the child. D. Tell the child that dentistry shouldn't be frightening. E. Chastise the child.

A, Asking the child about his or her fears will create an environment in which the child is encouraged to discuss any worries or concerns and to ask questions. This will also serve to alleviate anxiety, provide an opportunity to correct any misperceptions regarding dentistry, and to further establish or maintain trust and rapport.

Oral and genital lesions are seen in patients with which of the following diseases? A. Behçet's syndrome B. Peutz-Jegher's syndrome C. Herpangina D. Wegener's granulomatosis E. Hairy leukoplakia

A, Behçet's syndrome includes lesions in the mouth, eye, and genitals. The other diseases do not affect the genitalia

Which of the following agencies monitors and prevents disease outbreaks, implements disease prevention strategies, and maintains national health statistics? A. CDC B. FDA C. DEA D. IHS E. None of the above

A, CDC is correct. The U.S. Food and Drug Administration (FDA) is responsible for protecting the health of the nation against impure and unsafe foods, drugs, cosmetics, and other potential hazards. The Drug Enforcement Administration (DEA) determines the levels of controlled substances that have abuse potential. The Indian Health Services (IHS) focuses on the goal of raising the health status of Native Americans and Native Alaskans.

Which of the following is an arrangement between a plan and a group of dentists whereby the providers agree to accept certain payments (usually less than their usual fees) in anticipation of a higher volume of patients? A. PPO B. Capitation C. HMO D. IPA E. None of the above

A, Capitation is a payment mechanism whereby the dentist is paid a fixed amount irrespective of the number of patients seen or services provided. Health Maintenance Organizations (HMOs) are also called capitation plans because of the payment mechanism they use. An Individual Practice Association is a type of plan that combines the risk of capitation with fee for service reimbursement

The most effective topical antimicrobial agent currently available is _____. A. Chlorhexidine B. Stannous fluoride C. Phenolic compounds D. Sanguinarine

A, Chlorhexidine is the most effective antimicrobial agent currently available

Which of the following types of malocclusions is most common? A. Class I malocclusion B. Class II malocclusion C. Class III malocclusion D. Open bite malocclusion

A, Class I is the most common malocclusion, at about 50% of the U.S. population, compared to Class II (15%) and Class III (about 1%).

To ensure high radiographic image quality, it is important to daily _____. A. Check the temperature of the processing solutions B. Clean the processing equipment C. Clean the intensifying screens D. Calibrate the mA linearity

A, Daily check of the processing solution temperature, whether using automatic processing or manual tanks, and comparison with the manufacturer's recommended values will improve image quality. The other procedures are useful but can be performed less frequently.

Which of the following statements is not true regarding bonding systems? A. Even though dentin bonding occurs slowly, it results in a stronger bond than to enamel. B. Enamel bonding occurs quickly, is strong, and is long-lasting. C. One-bottle dentin bonding systems may be simpler but are not necessarily better. D. Dentin bonding is still variable because of factors such as sclerosis, tubule size, and tubule location.

A, Dentin bonding in laboratory studies may create bond strengths similar to or greater than bond strengths to enamel. However, clinical studies cannot corroborate that the dentin bond is stronger. In fact, the bond may deteriorate over time. Sufficient information is not available to accurately predict the bond potential to dentin in every application. Bonding to enamel, however, is predictable and good. The attempt to simplify the bonding mechanism has resulted in less materials being involved and less decision making on the part of the operator—both in an effort to get more predictable results. However, the newer bonding systems have not yet been proven to be better.

Deterministic effects are those that _____. A. Show a severity of response proportional to dose B. Are seen only in the oral cavity C. Are found following exposure to low levels of radiation D. Result from particulate radiation such as alpha and beta particles, but not x-rays E. None of the above

A, Deterministic effects are those with dose thresholds, thus requiring at least moderate levels of exposure, and where the severity of response is proportional to dose

The primary function of developer is to _____. A. Reduce crystals of silver halide to solid silver grains B. Reduce solid silver grains to specks of silver halide C. Remove unexposed silver halide crystals D. Remove exposed silver halide crystals

A, Developer reduces silver bromide to solid silver grains.

When is distraction osteogenesis preferred over a traditional osteotomy? A. When a large advancement is needed. B. When a small advancement is needed. C. When exacted interdigitation of the occlusion is needed. D. When the treatment needs to be done in a very short period of time. E. Distraction osteogenesis is always preferred over a traditional osteotomy.

A, Distraction osteogenesis is preferred over traditional osteotomies when large skeletal movements are required, and the associated soft tissue cannot adapt to the acute changes and stretching that results. Larger movements may be at increased risk of some relapse. This is particularly true in a patient with a cleft palate, where there is significant soft tissue scarring from previous surgeries

Advantages of fixed wire retention compared to a removable Hawley-type retainer include which of the following? A. Does not require the patient to remember to wear it. B. Is easier to clean. C. The design can be altered to achieve minor tooth movements. D. It can incorporate an acrylic bite plate to avoid relapse of overbite correction

A, Fixed retention requires no patient cooperation to achieve retention. However, fixed retainers are more difficult to clean and cannot be modified to move teeth or control overbite relapse

It is generally accepted that the maximum thickness of a composite increment that allows for proper cure is _____. A. 1-2 mm. B. 2-4 mm. C. 4-6 mm. D. There is no maximum thickness restriction

A, Generally, composite can be properly polymerized in 1- to 2-mm increments.

Which interleukin (IL) is important in the activation of osteoclasts and the stimulation of bone loss seen in periodontal disease? A. IL-1 B. IL-2 C. IL-8 D. IL-10

A, IL-1 is important in the activation of osteoclasts and stimulation of bone loss

The following are all desirable properties of an ideal local anesthetic, except _____. A. It should have potency sufficient to give complete anesthesia even if harmful results occur at therapeutic doses B. It should be relatively free from producing allergic reactions C. It should be stable in solution and readily undergo biotransformation in the body D. It should either be sterile or capable of being sterilized by heat without deterioration

A, Ideally, a local anesthetic should be relatively free from producing allergic reactions and it should be stable in solution and readily undergo biotransformation in the body. It is an absolute requirement that it should either be sterile or capable of being sterilized by heat without deterioration. If proper doses are used and are properly injected, there is a high success rate of obtaining anesthesia, while being able to minimize adverse effects

All of the following reasons are likely to indicate the need for restoration of a cervical notch except _____. A. Patient age. B. Esthetic concern. C. Tooth is symptomatic. D. Tooth is deeply notched axially.

A, If a patient has a notched cervical area that is very sensitive or very esthetically objectionable, restoration is usually indicated. If the notched area is very deep, adverse pulpal or gingival responses may occur. Although more notched areas are encountered in older patients, a patient's age is not a factor in the need for restoration

Which of the following is the most likely cause of an occlusal rest fracture? A. Inadequate rest-seat preparation B. Improper rest location C. Structural metal defects D. Occluding against the antagonist tooth

A, In McCracken's Removable Partial Prosthodontics, ed 11 (St Louis, Mosby, 2005), McCracken states, "Failure of an occlusal rest rarely results from a structural defect in the metal and rarely if ever is caused by distortion. Therefore the blame for such failure must often be assumed by the dentist for not having provided sufficient space for the rest during mouth preparations

The most obvious clinical sign of trauma from occlusion is increased tooth mobility. The most obvious radiographic sign of trauma from occlusion is an increase in the width of the periodontal ligament space. A. Both statements are true. B. Both statements are false. C. The first statement is true, the second statement is false. D. The first statement is false, the second statement is true.

A, Increased tooth mobility is the most common clinical sign of trauma from occlusion. Increased periodontal ligament width is the most common radiographic sign

Local anesthetics act on what type of receptor? A. An ion channel receptor B. A nuclear receptor C. A 7-membrane domain receptor linked to Gs D. A 7-membrane domain receptor linked to Gq E. A membrane receptor with tyrosine kinase activity

A, Inhibiting sodium channels leads to the inhibition of the nerve action potential and inhibition of nerve conduction. Sodium channels are examples of ion channel receptors. Ion channel receptors contain several subunits arranged in a barrel shape. Drugs that bind to the channel can alter conductance to the ion associated with that channel

Your patient is 9 years old. The mandibular left first primary molar has a large, carious lesion on the distal and on the occlusal and the tooth has greater mobility than what you would normally expect. You should _____. A. Take a radiograph of the area B. Perform a pulpotomy C. Perform a pulpectomy D. Extract the tooth and consider space maintenance

A, It is difficult to know which treatment is indicated without more information than is presented in the question. The tooth could be mobile due to furcation involvement, internal or external root resorption, exfoliation, or a combination of all the above. Obtaining more clinical information by taking a radiograph is necessary before any further treatment is rendered.

Elevation of cardiovascular signs with epinephrine, injected in a local anesthetic solution in a cardiovascularly compromised patient, occurs at about what threshold? A. 40 μg B. 100 μg C. 200 μg D. 1000 μg

A, Jastak and Yagiela have published data demonstrating that well-monitored, cardiovascularly compromised patients begin to show elevation of vital signs when more than about 40 μg (0.04 mg) of epinephrine is administered in the local anesthetic solution

The width of keratinized gingiva is measured as the distance from the _____. A. Free gingival margin to the mucogingival junction B. Cementoenamel junction to the mucogingival junction C. Free gingival groove to the mucogingival junction D. Free gingival margin to the base of the pocket

A, Keratinized gingiva extends from the free gingival margin to the mucogingival junction. The attached gingival extends from the free gingival groove to the mucogingival junction

A 25-year-old patient presenting with generalized marginal gingivitis without any systemic problems or medications should be classified with which periodontal prognosis? A. Good B. Fair C. Poor D. Questionable

A, Marginal gingivitis not complicated by systemic problems or medications usually can be treated successfully with phase 1 therapy, and a patient with this diagnosis would have a good prognosis

Which pulpotomy medicament demonstrates better success rates than formocresol? A. Mineral trioxide aggregate B. Calcium hydroxide C. Resin-modified glass ionomer cement D. Fifth-generation bonding agents

A, Mineral trioxide aggregate (MTA) pulpotomies have shown very good promise and generally show higher success rates than formocresol pulpotomies. However, at this time MTA is very expensive and is not used as often as formocresol or ferric sulfate

Which of the following is true regarding possible complications resulting from dental extractions? A. Patients with numbness lasting more than 4 weeks should be referred for microneurosurgical evaluation. B. Infections are common, even in healthy patients. C. Dry socket occurs in 10% of third molar patients. D. Teeth lost into the oropharynx are usually swallowed, and thus do not require further intervention

A, Most nerve injuries are transient; however, in an injury that lasts greater than 4 weeks, a surgical evaluation is indicated

Which is a nicotinic receptor? A. Receptor for the neurotransmitter at the skeletal-neuromuscular junction B. Receptor for the neurotransmitter at the junction between the postganglionic sympathetic nerve and sweat glands C. Receptor for the neurotransmitter at the junction between the postganglionic parasympathetic nerve and the parotid gland D. Receptor for the neurotransmitter at the junction between the postganglionic sympathetic nerve and blood vessels E. Receptor for the neurotransmitter at the junction between the postganglionic parasympathetic nerve and the heart

A, Nicotinic receptors are located at the skeletal-neuromuscular junction, ganglia, junction of the sympathetic nerve to the adrenal gland and the adrenal chromaffin cells, as well as in the central nervous system

Patients experiencing stress and anxiety typically require _____ interpersonal distance for comfortable interaction. A. Greater B. Less C. The same as patients who are not experiencing stress and anxiety D. Individualized E. Behaviorally controlled

A, Patients who are experiencing stress and anxiety typically feel more comfortable in having greater interpersonal space than they normally would when not experiencing stress and anxiety

Research suggests that life events and perceived stress/distress _____ predictors of selfreported health concerns. A. Are B. Are not C. Are sometimes D. Have little to do with E. None of the above

A, Perceived stress and distress in one's life has been demonstrated to be a significant predictor (positively correlated) with self-reported health concerns.

Which of the following indices is not reversible? A. DMFT B. GI C. PI D. OHI-S E. None of the above

A, Periodontal disease (measured by the PI) and gingival disease, measured by the GI, are reversible processes. The amount of the debris and calculus, measured by the OHI-S, can decrease too. Caries is not a reversible process

Central and peripheral giant cell granulomas share which of the following features? A. Microscopic appearance B. Clinical behavior C. Recurrence rate D. Similar forms of treatment E. Radiographic appearance

A, Peripheral and central giant cell granulomas have very different clinical presentations and behaviors, but identical light microscopic features

The _____ nerve block is recommended for management of several maxillary molar teeth in one quadrant. A. Posterior superior alveolar (PSA) B. Inferior alveolar (IA) C. Long buccal (LB) D. Nasopalatine (NP)

A, Posterior superior alveolar (PSA). This is the only injection listed that leads to pulpal anesthesia in the maxilla. The nasopalatine (NP) is a maxillary injection that leads to soft-tissue anesthesia of the premaxilla only. The inferior alveolar (IA) and long buccal (LB) are mandibular injections

A permanent incisor with a closed apex is traumatically intruded. What is the treatment of choice? A. Gradual orthodontic repositioning and calcium hydroxide pulpectomy B. Surgical repositioning and calcium hydroxide pulpectomy C. Gradual orthodontic repositioning and conventional endodontic therapy D. Surgical repositioning and conventional endodontic therapy

A, Rapid root resorption, pulp necrosis, and ankylosis are common sequelae to intruded permanent teeth with mature apices. Treatment includes: a. Gradual repositioning orthodontically (2-3 weeks) b. Stabilize for 2 to 4 weeks c. Calcium hydroxide pulpectomy 2 weeks after injury

Which of the following odontogenic cysts occurs as a result of stimulation and proliferation of the reduced enamel epithelium? A. Dentigerous cyst B. Lateral root cyst C. Radicular cyst D. Odontogenic keratocyst E. Gingival cyst

A, Reduced enamel epithelium that overlies the crown of an unerupted tooth may give rise to a cyst occurring in the same position. This is, by definition, a dentigerous cyst. The stimulus for cystic epithelial proliferation is unknown

A 7-year-old patient presents with a quadrant of teeth showing abnormal formation of both enamel and dentin. All of his other teeth appear clinically normal. Radiographically, the affected teeth can be described as "ghost teeth." He has _____. A. Regional odontodysplasia B. Dens evaginatus C. Dentin dysplasia D. Ectodermal dysplasia E. Cleidocranial dysplasia

A, Regional odontodysplasia is often called "ghost teeth" because of the thin layers of dentin and enamel produced. One quadrant of teeth is affected, and the teeth are nonfunctional

Which class of antihypertensive drug most effectively reduces the release of renin from the kidney? A. β-adrenergic receptor blockers B. ACE inhibitors C. α-adrenergic receptor blockers D. Calcium channel blockers E. Angiotensin II receptor blockers

A, Renin release from the kidney is enhanced by stimulation of the β1-adrenergic receptors in the juxtaglomerular cells. From the above list, only β blockers reduce renin release. Although angiotensin converting enzyme inhibitors and angiotensin II receptor blockers act on the renin-angiotensin system, they do not inhibit renin release. In fact, they tend to increase plasma renin.

Scalers are used to remove supragingival deposits. Curettes are used to remove either supragingival or subgingival deposits. A. Both statements are true. B. Both statements are false. C. First statement is true. Second statement is false. D. First statement is false. Second statement is true.

A, Scalers, with their pointed ends and back, are designed for supragingival instrumentation; curettes, with their rounded ends and back, can be used for both supragingival and subgingival instrumentation

A major difference between total-etch and selfetching primer dentin bonding systems include all of the following except _____. A. The time necessary to apply the material(s) B. The amount of smear layer removed C. The bond strengths to enamel D. The need for wet bonding

A, Self-etch dentin bonding systems differ from totaletch dentin bonding systems by removing less of the smear layer (they use a less potent acid), creating a weaker bond to enamel (especially nonprepared enamel), and not requiring wet bonding which may be necessary for some of the total-etch systems. Even though fewer actual materials may be needed with some of the self-etch systems, they need to be applied in multiple coats and therefore the time necessary to apply the materials is similar for both systems

The functional inquiry questionnaire reveals that the mother has had negative dental experiences and remains very nervous regarding her dental care. How would this most likely influence her 3-year-old child's reaction to dentistry? A. Increase the likelihood of a negative behavior. B. Increase the likelihood of a positive response to dentistry. C. Will likely cause an initial positive reaction, which changes to a negative reaction with the slightest stress. D. Maternal anxiety has little effect on a child's behavior in a dental setting.

A, Studies show that there is a high correlation between maternal anxiety and a child's negative behavior in the dental office. This effect is greatest for children less than 4 years of age

What bacterial species are found in increased numbers in the apical portion of toothassociated attached plaque? A. Gram-negative rods B. Gram-positive rods C. Gram-positive cocci D. Gram-negative cocci

A, Subgingival plaque can be in the cervical area or more apical. In both areas it can be either tooth-associated or tissue-associated. The apical tooth-associated plaque is composed primarily of gram-negative rods

The free gingival graft technique can be used to increase the width of attached gingival tissue. Apically displaced full-thickness or partialthickness flaps can also be used to increase the width of attached gingiva. A. Both statements are true. B. Both statements are false. C. First statement is true. Second statement is false. D. First statement is false. Second statement is true

A, Surgical techniques designed to increase the width of attached gingiva include free gingival grafts and apically repositioned flaps

Anticipating correct administration of the (long) buccal injection, what areas will be anesthetized? A. Soft tissues and periosteum buccal to the mandibular molar teeth B. Soft tissues and periosteum lingual to the mandibular molar teeth C. Soft tissues and periosteum lingual to the mandibular premolar teeth D. Soft tissues and periosteum buccal to the mandibular premolar teeth

A, The (long) buccal injection anesthetizes the soft tissues and periosteum buccal to the mandibular molar teeth

Which of the following federal agencies is the U.S. government's principal agency for protecting the health of all Americans and providing essential human services? A. DHHS B. NIH C. HRSA D. AHRQ E. None of the above

A, The DHHS is the U.S. government's principal agency for protecting the health of all Americans and providing essential human services. DHHS includes 11 agencies and more than 300 programs. The agencies listed in the answers are part of the DHHS. The National Institutes of Health (NIH) is the world's premier medical research organization. The Health Resources and Services Administration (HRSA) provides access to essential health care services for people who are low-income, uninsured, or who live in rural areas or urban neighborhoods where health care is scarce. The Agency for Healthcare Research and Quality (AHRQ) supports research on health care systems, health care quality and cost issues, access to health care, and effectiveness of medical treatments

Occlusal radiographs are useful for all of the following except _____. A. For views of the TMJ B. For displaying large segments of the mandibular arch C. When the patient has limited opening D. When there are sialoliths in the floor of the mouth E. When there is buccal-lingual expansion of the mandible

A, The TMJ is much too far from the occlusal plane (the location of occlusal film) to be imaged with this technique. The other choices are all proper indications for using occlusal film.

Bonding of resins to dentin is best described as involving _____. A. Mechanical interlocking B. Ionic bonding C. Covalent bonding D. Van der Waals forces

A, The bond of adhesives to dentin (and enamel) is primarily a mechanical interlocking of the material within the dentin (or enamel). The etching causes some removal of the surface, creating irregularities or spaced collagen fibrils into which the adhesive enters. When polymerized, the adhesive is mechanically locked into the surface

What would be the effect of prior administration of a competitive drug antagonist on the concentration-response profile of a drug agonist on a graded concentration-response curve? (Assume that both drugs act at the same receptor.) A. The agonist curve would shift to the left. B. The agonist curve would shift to the right. C. The agonist curve would not change. D. The agonist curve would not shift but would reach a lower maximal effect than the curve with agonist alone. E. The agonist curve would both shift to the left and have a lower maximal effect

A, The characteristic response to a competitive antagonist is a parallel shift to the right of the agonist curve, with the two curves reaching the same maximal effect

A comparison of screen film/intensifying screen combinations with direct-exposure films reveals that screen film/intensifying screen combinations _____. A. Render less resolution B. Require more exposure C. Require special processing chemistry D. Are preferred for intraoral radiography

A, The dispersion of visible light from the crystals in the phosphor layer of the intensifying screen reduces image resolution compared to directexposure film.

For a dental hand instrument with a formula of 10-8.5-8, the number 10 refers to _____. A. The width of the blade, in tenths of a millimeter B. The primary cutting edge angle, in centigrades C. The blade length, in millimeters D. The blade angle, in centigrades

A, The first number is the width of the blade or primary cutting edge in tenths of a millimeter (0.1 mm). The second number of a four-number code indicates the primary cutting edge angle, measured from a line parallel to the long axis of the instrument handle in clockwise centigrades. The angle is expressed as a percent of 360 degrees. The instrument is positioned so that this number always exceeds 50. If the edge is locally perpendicular to the blade, then this number is normally omitted, resulting in a threenumber code. The third number (second number of a three-number code) indicates the blade length in millimeters. The fourth number (third number of a three-number code) indicates the blade angle, relative to the long axis of the handle in clockwise centigrade.

The incisive papilla provides a guide for the anteroposterior placement of maxillary anterior denture teeth. The labial surfaces of natural teeth are generally 8 to 10 mm anterior to this structure. A. Both statements are true. B. The first statement is true, and the second statement is false. C. The first statement is false, and the second statement is true. D. Both statements are false

A, The incisive papilla provides a guide for the anteroposterior position of the maxillary anterior teeth. The labial surfaces of the central incisors are usually 8 to 10 mm in front of the papilla. This distance varies depending of the amount of resorption of the residual ridge, the size of the teeth, and the labiolingual thickness of the alveolar process

Two cystic radiolucencies in the mandible of a 16-year-old boy were lined by thin, parakeratinized epithelium showing palisading of basal cells. All teeth were vital and the patient had no symptoms. This patient most likely has which of the following? A. Odontogenic keratocysts B. Periapical granulomas C. Periapical cysts D. Traumatic bone cysts E. Ossifying fibromas

A, The key to this question is the description of the cystic lining of thin, parakeratinized epithelium with basal cell palisading—typical of odontogenic keratocyst. Tooth vitality, lack of symptoms, and more than one lesion are also supportive.

All of the following statements about slotretained complex amalgams are true except _____. A. Slots should be 1.5 mm in depth. B. Slots should be 1 mm or more in length. C. Slots may be segmented or continuous. D. Slots should be placed at least 0.5 mm inside the DEJ.

A, The longer a slot, the better. They should be inside the DEJ and prepared with an inverted cone bur to a depth of 1 mm.

An adult patient with a Class II molar relationship and a cephalometric ANB angle of 2 degrees has which type of malocclusion? A. Class II dental malocclusion B. Class II skeletal malocclusion C. Class I dental malocclusion D. Class II skeletal malocclusion

A, The molars are Class II, but the skeletal relationship described by the ANB (the A-P angular difference between the maxilla and mandible) measurement is normal, so the malocclusion is dental in origin.

Which of the following is not appropriate treatment for an odontogenic abscess? A. Placing the patient on antibiotics and having them return when the swelling resolves B. Surgical removal of the source of the infection as early as possible C. Drainage of the abscess with placement of surgical drains D. Close observance of the patient during resolution of the infection E. Medical management of the patient to correct any compromised states that might exist

A, The primary principle of management of odontogenic infections is to perform surgical drainage and removal of the cause. Abscesses will not resolve on antibiotics alone and may progress even if the patient is on antibiotics

What is the approximate elimination half-time for penicillin V? A. 0.5 hour B. 2 hours C. 4 hours D. 8 hours E. 12 hours

A, The short elimination half-time for penicillin V is due to rapid excretion of penicillin in the urine. About 90% of this renal excretion is a result of active tubular transport, a rapid and efficient process. (Very little metabolism of penicillin occurs.)

You have placed a dental implant for replacement of tooth #9. Preoperatively you obtained a panoramic and a periapical film. During the surgery, you used a crestal incision, series of drills, and paralleling pins as necessary. Upon restoration of the crown, obtaining ideal esthetics is difficult because the implant is placed too close to the labial cortex, causing the restoration to appear overcontoured. Which of the techniques below could most adequately have prevented this problem? A. Using an anterior surgical template B. Obtaining preoperative tomograms of the alveolus C. Using a tissue punch technique D. Using a smaller size of implant

A, The surgical guide template is a critical factor for the placement of implant in the esthetic area.

Following the administration of a local anesthetic, most patients can be maintained in conscious sedation at _____. A. 20%-40% nitrous oxide B. 20%-40% oxygen C. 50% nitrous oxide D. 10% nitrous oxide

A, The total flow rate is 4 to 6 L/min for most children. The practitioner can check the bag and make adjustments if necessary. The maintenance dose of nitrous oxide during an operative procedure is typically about 30%. In other words, a standard maintenance dose would usually be 4 L oxygen and 2 L of nitrous oxide. Of course after a lengthy administration, it is wise to reduce the concentration due to tissue saturation and nausea.

Your patient is 7 years old and has a very large, carious lesion on tooth T. What radiological factors should be used in determining the best treatment of choice between pulpotomy and primary endodontics? A. Furcation involvement B. External root resorption C. Internal root resorption D. Two of the above E. All of the above

A, The treatment decision in this case should be made on the presence or absence of furcation involvement. Absence of furcation involvement generally indicates a vital pulp. Of course, it is necessary to have vital tissue to perform a pulpotomy. Presence of furcation involvement generally indicates progression to a nonvital pulp. If furcation involvement is present, a pulpectomy would be the treatment of choice in the absence of external or internal root resorption

Which of the following ADA's Principles of Ethics states that a dentist has a duty to respect the patient's right to self-determination and confidentiality? A. Patient Autonomy B. Nonmaleficence C. Beneficence D. Justice E. Veracity

A, There are five principles in the ADA Principles of Ethics: ● Patient Autonomy ("self-governance"). The dentist has a duty to respect the patient's rights to self-determination and confidentiality. ● Nonmaleficence ("do no harm"). The dentist has a duty to refrain from harming the patient. ● Beneficence ("do good"). The dentist has a duty to promote the patient's welfare. ● Justice ("fairness"). The dentist has a duty to treat people fairly. ● Veracity ("truthfulness"). The dentist has a duty to communicate truthfully.

In HIV diagnosis, the Western blot assay is used to confirm the results of a positive ELISA test. Therefore, we can say that the Western blot test will confirm a _____. A. True-positive result B. True-negative result C. False-positive result D. False-negative result E. None of the above

A, Very specific tests are appropriate for confirming the existence of a disease. If the result of a highly specific test is positive, the disease is almost certain. High specificity is required in situations where the consequences of a false-positive diagnosis are serious or unduly alarming (e.g., HIV positivity).

Loss of tooth substance by mechanical wear is _____. A. Abrasion B. Attrition C. Erosion D. Abfraction

A, Wasting diseases of the teeth include erosion (corrosion; may be caused by acidic beverages), abrasion (caused by mechanical wear as with toothbrushing with abrasive dentifrice), attrition (due to functional contact with opposing teeth), and abfraction (flexure due to occlusal loading).

A 7-year-old child has a history of recurrent pain and discomfort in a second molar, which has a necrotic pulp. You present the treatment options to the parents. "There are several ways in which we can treat this problem. We could do a pulpectomy in which we . . . . We could do something called a pulpotomy, which involves. . . . We could apply a pulp cap which is . . . . We could remove the tooth. Or we could leave the tooth untreated for now and see how things go." You have phrased the options so that they are in what you believe to be the order of descending desirability and you have indicated that to the patient. Which option is most likely to be chosen by the parents? A. Pulpectomy B. Pulpotomy C. Pulp cap D. Extraction E. No treatment

A, When a number of alternatives are presented and the first on the list is more desirable, there is a tendency for individuals to select the first option and view the successive options as less desirable.

Cervical Flexion Posit?

ADI space-transverse lig stab Cass = 8x10 (sideways) FFD= 72 Tilt = 0

In the Lovette Positive Classification, on what side is the convex scoliosis?

AI sacrum side

what is the best xray for a jefferson's burst fx

APOM

Palpatory findings: Spongy edema at posterior inferior margin of the SI joint. This is seen with what pelvic subluxation?

AS ilium

What pelvic subluxation presents with the following: higher femur head, posterior sacrum, shorter innominate, smaller obturator foramen, and decreased lumbar lordosis?

AS ilium

In a scoliosis, on what side should the sole lift be used?

AS ilium side

Autoimmune hepatitis

ASMA, AMA, ALKM-1 antibodies, hemolytic anemia, ITP, celiac, type 1 DM, thyroiditis

Paroxysmal superventricular tachycardia

AV node re-entry, no structural heart disease, vagal maneuvers, adenosine

Mesenteric ischemia

Abdominal pain out of proportion to exam, lactic acidosis, hypoactive bowel sounds, pro-coagulation

Enterohemorrhagic E. coli

Abdominal pain, afebrile, bloody diarrhea, antibiotic use, HUS, TTP, undercooked ground beef

Budd-Chiari syndrome

Abdominal pain, ascites, hepatomegaly, hepatic vein clot, spider web collaterals, polycythemia vera

Acute intermittent porphyria

Abdominal pain, confusion, headaches, hallucinations, dizziness, primdone, urine porphyins

Trichinellosis

Abdominal pain, splinter hemorrhages, chemosis, periorbital edema, myalgias, weakness, eosinophilia

Cauda equina syndrome

Absent rectal tone, urinary incontinence, motor and sensory loss in extremities, metastatic prostate cancer

What is the most common problem with implants?

Abutment or prosthesis screw loosening

Chagas disease

Achalasia, dilated cardiomyopathy, megacolon

Polymyalgia rheumatica

Aching pain in neck, shoulders, hips, morning stiffness, prednisone

Retropharyngeal abscess

Acute necrotizing mediastinitis, trismus

Allergic interstitial nephritis

Acute renal failure, arthralgia, rash, sulfonamide ingestion, eosinophilic WBC casts

Adrenal tuberculosis

Adrenal calcification, primary adrenal insufficiency

Zinc deficiency

Alopecia, abnormal taste, bullous pustules around body orifices, impaired wound healing

Hepatocellular carcinoma

Alpha fetoprotein, ascites, hepatitis B

Acetaminophen toxicity

Aminotransferases > 5000, alcoholism, N-acetylcysteine

Patient is taking Methotrexate (immunosuppressant), what antibiotic is contraindicated?

Amoxicillin, because decrease renal clearance

Mucormycosis

Amphotericin B, necrotic nasal turbinates, chemosis, proptosis, diplopia, rhizopus

The ASIS's move anterior to the pubic symphysis.

Anterior Pelvic Tilt

Systemic sclerosis

Anti topoisomerase I antibodies, anti-centromere antibodies, calcinosis cutis, hypertension, pulmonary hypertension, GI dysmotility

Dermatomyositis

Anti-Mi-2 antibody, ovarian cancer, proximal muscle weakness

Sjogren's syndrome

Anti-Ro/La antibodies, keratoconjunctivitis sicca, xerostomia, submandibular gland swelling, distal RTA

CREST syndrome

Anti-centromere antibodies

Heyde's syndrome

Aortic stenosis, rectal bleeding, angiodysplasia, von Willebrand disease

Tick-borne paralysis

Ascending paralysis, no CSF abnormalities, absent deep tendon reflexes, toxin-mediated

Mononeuritis multiplex

Assemitrical weakness and sensory loss, severe pain, vasculitis, diabetes mellitus

Allergic bronchopulmonary aspergillosis

Asthma, cystic fibrosis, eosinophilia, hemoptysis, fever

Churg-Strauss vasculitis

Asthma, p-ANCA, alveolar hemorrhage, eosinophilic granulomas, peripheral neuropathy

Thiamine deficiency

Ataxia, ophthalmoplegia, nystagmus, dementia

Ischemic colitis

Atherosclerosis, splenic flexure, thumbprinting

Digitalis toxcitiy

Atrial tachycardia with AV block

Most common site of herpes?

Attached gingiva

Systemic lupus erythematosus

Autoimmune hemolysis, spherocytosis, granular focal proliferative glomerulonephritis, pancytopenia, anti-Smith/dsDNA, oral ulcers

Hepatitis A

Aversion to smoking, dark urine, hepatomegaly

In the Lovette Positive Classification, to what side will the spinous deviate?

Away from convexity

In an adult of normal size, penetration to a depth of _____ mm places the needle tip in the immediate vicinity of the foramina, through which the posterior superior alveolar (PSA) nerves enter the posterior surface of the maxilla. A. 10 B. 16 C. 20 D. 30

B, 16 mm. The proper depth of penetration for the PSA nerve can be said to be half the length (16 mm) of a long needle or three-fourths the length (15 mm) of a short dental needle. Penetration beyond 16 mm has a significantly higher incidence of positive aspiration and hematoma formation

25-gauge needles are preferred to smaller-diameter ones due to all of the following reasons except _____. A. Greater accuracy in needle insertion for 25-gauge needles B. Increased rate of needle breakage for 25-gauge needles C. Aspiration of blood is easier and more reliable through a larger lumen D. There is no difference in pain of insertion

B, 25-gauge needles have a much lower incidence of breakage versus any other needle size commonly used in dentistry, whereas 30-gauge needles have by far the worst record

A 1.0-ml volume of a 2% solution contains _____. A. 18 mg B. 20 mg C. 36 mg D. 54 mg

B, A 2% solution is 20 mg/mL. 1.0 mL of a 20 mg/mL solution is 20 mg.

Which of the following statements regarding caries risk assessment is correct? A. The presence of restorations is a good indicator of current caries activity. B. The presence of restorations is a good indicator of past caries activity. C. The presence of dental plaque is a good indicator of current caries activity. D. The presence of pit-and-fissure sealants is a good indicator of current caries activity

B, A restored tooth indicates potential past carious activity but not current activity. Plaque presence does not necessarily indicate caries presence and sealants are used for preventive purposes, not caries treatment

What is a second-order bend? A. A bend to position a tooth buccolingually B. A bend to provide angulation of a tooth in mesiodistal direction (tip) C. A bend to provide correct angulation of a tooth in labiolingual direction (torque) D. A bend to rotate a tooth

B, A second-order bend is placed to provide angulation of a tooth in the mesiodistal direction, also called tip. A first-order bend is placed in an archwire to position a tooth in the labiolingual direction (in-out bend) and/or to rotate a tooth as seen in the occlusal plane. A bend to provide angulation in the labio-lingual direction is called a third-order bend (torqueing bend).

Which drug is most selective as a glucocorticosteroid? A. Aldosterone B. Dexamethasone C. Fludrocortisone D. Hydrocortisone

B, Aldosterone and fludrocortisone are selective mineralocorticosteroids. Hydrocortisone has significant mineralocorticoid and glucocorticoid activity. Dexamethasone has very little mineralocorticoid activity

Your patient exhibits enamel hypoplasia near the incisal edges of all permanent incisors and cuspids, except for the maxillary lateral incisors, which appear normal. At what age would you suspect some kind of systemic problem? A. Prior to birth B. From birth to 1 year of age C. From 1 to 2 years of age D. From 2 to 3 years of age

B, All anterior permanent teeth begin calcification during the first 6 months, except for maxillary lateral incisors. The maxillary lateral incisor may be used as a key to timing; if this tooth is affected, the causative event is likely to have occurred at 1 year of age or older

What is the direct effect of local anesthetics on blood vessels in the area of injection? A. Constriction B. Dilation C. Sclerosis D. Thrombosis

B, All local anesthetics are vasodilators to some degree

Which of the following local anesthetics causes the least amount of vasodilation? A. Lidocaine B. Mepivacaine C. Bupivacaine D. Articaine

B, All local anesthetics cause some amount of vasodilation. Those packaged as plain drugs (i.e., without vasoconstrictor) cause less vasodilation than do those drugs that must be packaged with vasoconstrictor to have efficacy. Of the listed drugs, Mepivacaine is the only one packaged in dental cartridges without vasoconstrictor

In a patient who displays excessive maxillary incisor at rest, has an excessive lower face height, and has a deep overbite, which of the following would be the preferred method of overbite correction? A. Eruption of posterior teeth to rotate the mandible open B. Intrusion of maxillary incisors C. Intrusion of mandibular incisors D. Flaring of maxillary and mandibular incisors

B, All of the choices are possible solutions to correct a deep overbite. Erupting posterior teeth would increase the already excessively long lower face height, whereas intrusion of maxillary incisors would improve the excessive maxillary incisor show at rest.

Of the following, which is the least reliable way to predict the timing of the peak of the adolescent growth spurt for an individual? A. Plotting changes in height over time on a growth curve B. Following eruption timing of the dentition C. Taking a hand-wrist radiograph to assess skeletal development D. Observing changes in secondary sex characteristics

B, Although developmental indicators generally correlate well with each other, using dental age to predict timing of growth is the least reliable of the methods offered

Which of the following statements about an amalgam tooth/cavity preparation is true? A. The enamel cavosurface margin angle must be 90 degrees. B. The cavosurface margin should provide for a 90-degree amalgam margin. C. All prepared walls should converge externally. D. Retention form for Class Vs can be placed at the DEJ.

B, Although the amalgam margin must be 90 degrees, the enamel margin might not be 90 degrees, especially on the occlusal surface. Most walls converge occlusally, but many Class V amalgam preparations have walls that diverge externally. No retention form should be placed at the DEJ; otherwise, the adjacent enamel will be undermined and subject to fracture

When carving a Class I amalgam restoration, which statement is false? A. Carving may be made easier by waiting 1 or 2 minutes after condensation before it is started. B. The blade of the discoid carver should move parallel to the margins resting on the partially set amalgam. C. Do not carve deep occlusal anatomy. D. The carved amalgam outline should coincide with the cavosurface margins

B, Amalgam carving should result in coincidence with the cavosurface margin and should not result in deep occlusal anatomy because such form may create acute amalgam angles that are subject to fracture. Depending on the condensation rate of the amalgam used, waiting a couple of minutes prior to initiating carving may allow the amalgam to harden enough that the carving will be easier and overcarving will be minimized. When carving the occlusal cavosurface margin, the discoid carver should rest on the adjacent unprepared enamel, which will serve as a guide for proper removal of amalgam back to the margin.

On a panoramic radiograph of a 13-year-old patient, there is evidence of crown formation of the third molars but no root formation yet. These teeth fall into the category of impacted teeth. A. True B. False

B, An impacted tooth is one that fails to erupt into the dental arch within the expected time. Consequently the third molar in a 13-year-old patient would be classified as unerupted or in the process of erupting

An interdental crater has how many walls? A. One wall B. Two walls C. Three walls D. Four walls

B, An interdental crater has two bony walls remaining. These walls are usually the facial and lingual walls.

The most radiosensitive of the following cells in terms of cell killing is the _____. A. Salivary gland acinar cell B. Basal epithelial cell C. Endothelial cell D. Neuron. E. Polymorphonuclear leukocyte

B, Basal epithelial cells are the most mitotically active of the cells on the list, and thus are the most radiosensitive

Which of the following medications often result in overgrowth of gingival tissues? A. Penicillin, calcium channel blockers, phenytoin B. Calcium channel blockers, phenytoin, and cyclosporin C. Cyclosporin, penicillin, and cephalosporins D. Ampicillin, tetracycline, and erythromycin

B, Calcium channel blockers, cyclosporin, and phenytoin often result in overgrowth of gingival tissues.

Which of the following are cells of the innate immune system? a. Neutrophils and monocytes/macrophages b. T cells and B cells c. Mast cells and dendritic cells d. Plasma cells A. a and b B. a and c C. b and d D. b and c

B, Cells of the innate immune system include neutrophils, monocytes/macrophages, mast cells, and dendritic cells. Cells of the specific (adaptive) immune system include T cells, B cells, and plasma cells.

When Class III elastics are used, the maxillary first molars will _____. A. Move distally and intrude B. Move mesially and extrude C. Move mesially and intrude D. Move only mesially; there will be no movement in the vertical direction

B, Class III elastics are worn from the maxillary first molars to the mandibular canines. The force system created by Class III elastics will produce mesial movement and extrusion of the maxillary first molars.

In comparison to amalgam restorations, composite restorations are _____. A. Stronger B. More technique-sensitive C. More resistant to occlusal forces D. Not indicated for Class II restorations

B, Composite restorations are more techniquesensitive than amalgam restorations because the bonding process is very specific (requiring exact, correct usage of the various materials and an isolated, noncontaminated field), and the insertion and contouring of composites are more demanding and time-consuming. Composites are not stronger than amalgam and have similar wear resistance compared to amalgams. Composites are indicated for Class II restorations

How many days does it usually take for surface epithelialization to be complete following a gingivectomy? A. 3-7 B. 5-14 C. 14-18 D. 20-27

B, Epithelial cells migrate approximately 0.5 mm/day. Following a gingivectomy, it takes 5 to 14 days for surface epithelialization to be complete.

A 6-year-old patient likes to tell you stories about school. Each time he begins a story, you stop working to listen. After three long sessions, you realize that the child is attempting to avoid or delay the dental work by telling stories. You decide that from this point on you are going to continue working while engaged in conversation with the patient. At first, the child tells you more stories about school and, further, tries other strategies to get your attention and stop your work. He eventually settles down and allows you to work, whether or not you are engaged in conversation. This is an example of _____. A. Shaping B. Extinction C. Modeling D. Stimulus control E. Power

B, Extinction is the process of identifying all positive reinforcements (in this case, the dentist ceasing work on the child's teeth) that maintain a behavior and ceasing or withholding these

Which statement is false regarding motivation? A. Motivation is strengthened when a person succeeds and is weakened when a person fails to achieve his/her goals. B. Motivation is increased when the patient focuses on long-term goals. C. Motivating a patient can be achieved by generating interest, showing your concern, and providing information. D. Encourage a sense of personal acceptance in the face of the inevitable difficulties involved in breaking old habits and establishing new ones. E. Help a patient cope with relapses by emphasizing the knowledge gained

B, Focusing on long-term goals is not only a poor motivator, it is often a pitfall in the effort to change behavior, as patients are less motivated when goals seem too big, impossible, or far from their current circumstances.

The globulomaxillary lesion of bone _____. A. Is associated with the crown of an unerupted tooth B. Occurs between maxillary lateral and canine teeth C. Typically causes pain D. Typically presents as a mixed lucent-opaque lesion with ill-defined margins E. Is always associated with a nonvital tooth

B, Globulomaxillary lesion is a clinical term used to designate any lucency that occurs between the maxillary lateral incisor and canine

Stimulation of gluconeogenesis and lipolysis are most characteristic of which hormone? A. Calcitonin B. Cortisol C. Insulin D. Parathyroid hormone E. Progesterone

B, Glucocorticoids characteristically stimulate gluconeogenesis and lipolysis. Insulin has the opposite effects. The other hormones have minor or negligible effects

A patient has difficulty inhibiting the gag reflex during x-ray procedures. You suggest that the patient take several x-ray packets home and practice holding the packets in his or her mouth for increasingly longer periods of time. Which of the following techniques does this best exemplify? A. Reinforcement B. Graded exposure C. Modeling D. Behavioral control E. Systematic desensitization

B, Graded exposure is the systematic process of exposing the patient to a hierarchy of increasingly anxiety-provoking stimuli

Which drug poses the greatest risk of a cardiac arrhythmia when administered at the same time as epinephrine? A. Desflurane B. Halothane C. Isoflurane D. Propofol E. Sevoflurane

B, Halothane sensitizes the heart to epinephrine and other catecholamines

In tooth movement, the formation of a hyalinized zone on the pressure side is due to _____. A. The application of light, continuous forces B. The application of heavy forces C. The normal forces of mastication D. Abnormal swallowing patterns

B, Heavy forces cause compression of the PDL with hyalinization

If your radiographs start coming out too light, it may be that _____. A. Your exposure time is too long B. Your developer needs changing C. Your developer is too hot D. The fixer needs changing E. The films are not sufficiently washed

B, If proper processing procedures are followed, the developer will become depleted with age and need changing.

Inadequate margins of restorations should be corrected primarily because they _____. A. Cause occlusal disharmony B. Interfere with plaque removal C. Create mechanical irritation D. Release toxic substances

B, Inadequate or overhanging margins serve as a nidus for dental plaque accumulation and make plaque removal difficult

Which of the following statements is not true regarding internal root resorption? A. It happens rarely in permanent teeth. B. It appears as an asymmetrical "moth-eaten" lesion in radiographs. C. Chronic pulpal inflammation is the primary cause. D. Prompt endodontic therapy will stop the process.

B, Internal resorption is most commonly identified during routine radiographic examination. Histologically, it appears with chronic pulpitis, including chronic inflammatory cells, multinucleated giant cells adjacent to granulation tissue, and necrotic pulp coronal to resorptive defect. Only prompt endodontic therapy will stop the process and prevent further tooth destruction

The impression material that is mainly composed of sodium or potassium salts of alginic acid is _____. A. Polyether B. Irreversible hydrocolloid C. Polyvinyl siloxane D. Polysulfide

B, Irreversible hydrocolloid (IH) or alginate is the material of choice to produce diagnostic casts. Its composition is mainly sodium or potassium salts of alginic acid. They react chemically with calcium sulfate to produce insoluble calcium alginate

A portion of which cranial nerve is anesthetized when performing an infraorbital nerve block? A. VII B. V C. III D. II

B, It is the intent with all intraoral injections of local anesthesia that you anesthetize a portion of the fifth cranial nerve. With an improperly placed needle in a mandibular block, it is possible to inadvertently anesthetize a portion of the seventh cranial nerve, and it is possible to inadvertently anesthetize the sixth cranial nerve with certain second-division nerve blocks

Localized aggressive periodontitis in the primary dentition is seen most commonly in the primary molar area. It is most common in Asian children. A. The first statement is true and the second statement is true. B. The first statement is true and the second statement is false. C. The first statement is false and the second statement is true. D. The first statement is false and the second statement is false.

B, Localized aggressive periodontitis in the primary dentition, previously known as localized prepubertal periodontitis (LPP) is most common in the primary molar area and occurs most commonly in African-American children. Treatment includes debridement and antibiotic therapy

According to Malamed, slow injection is defined as the deposition of 1 ml of local anesthetic solution in not less than _____. A. 15 seconds B. 30 seconds C. 60 seconds D. 2 minutes

B, Malamed recommends that one cartridge of local anesthetic be delivered over not less than 1 minute. Therefore, 1 mL (one-half cartridge) should be delivered over not less than one-half minute (30 seconds).

On the health history form, the mother of a 6-year-old new patient notes that the child is moderately mentally challenged. The dentist should _____. A. Refer to a pediatric dentist B. Use a Tell-Show-Do technique of behavior management C. Use conscious sedation D. Use restraints after obtaining informed consent

B, Many mentally challenged individuals can be mainstreamed and treated as any other patient. Because a moderately challenged 6-year-old may function as a preschool child, the normal management techniques are likely applicable. The correct answer for such a question will include some kind of normalization response

Most natal and neonatal teeth are primary teeth. They should be extracted. A. The first statement is true and the second statement is true. B. The first statement is true and the second statement is false. C. The first statement is false and the second statement is true. D. The first statement is false and the second statement is false.

B, Most natal and neonatal teeth are primary teeth (90%); very few are supernumerary teeth (10%). Most are mandibular incisors (85%). Extraction of primary teeth should be accomplished only if they are extremely mobile and there is danger of aspiration. Most commonly, natal and neonatal teeth are left in position

Which statement is false regarding the relationship between pain and fear? A. Fear initially inhibits pain due to a release of endorphins from the pituitary, resulting in an analgesic effect. B. Although muscle tension contributes to the experience of anxiety, it does not contribute to the perception of pain. C. Any autonomic activation causes one to have a lower pain threshold. D. Catastrophic thinking and a perceived lack of control are common factors that influence pain perceptions. E. Misattribution occurs when patients identify an event as painful because they can identify a fearful stimulus

B, Muscle tension is associated with the experience of anxiety. Heightened anxiety contributes to lower pain thresholds/sensitivity to the perception of pain

What makes it possible for nickel-titanium archwires to exhibit superelastic behavior? A. This behavior is based on a reversible transformation within the austenitic phase. B. This behavior is based on a reversible transformation between the austenitic and martensitic phases. C. This behavior is based on a reversible transformation within the martensitic phase. D. This behavior is based on an irreversible transformation within the martensitic phase.

B, Nickel-titanium archwires can exist in more than one phase: austenitic and martensitic phases. Superelastic behavior of these wires is attributed to the reversible transformation between these two phases.

A 32-year-old male patient presented with a 1 ¥ 2-cm macular red-blue lesion in his hard palate. The lesion was asymptomatic and had been present for an unknown duration. He had no dental abnormalities and no significant periodontal disease. This could be all the following except _____. A. Vascular malformation B. Nicotine stomatitis C. Ecchymosis D. Kaposi's sarcoma E. Erythroplasia

B, Nicotine stomatitis appears as opacification of the palate, with red dots representing inflamed salivary ducts

Obstructive sleep apnea syndrome (OSAS) often results in all of the following except _____. A. Excessive daytime sleepiness B. Aggressive behavior C. Personality changes D. Depression

B, OSAS may result in mood disorders, daytime fatigue, and personality changes. Aggressive behavior is not considered a sequela of OSAS

Many factor affect tooth/cavity preparation. Which of the following would be the least important factor? A. Extent of the defect B. Size of the tooth C. Fracture lines D. Extent of the old material

B, Obviously, a tooth preparation is dictated by the extent of the carious lesion or old restorative material, the creation of appropriate convenience form for access and vision, and the anticipated extensions necessary to provide an appropriate proximal contact relationship. Fracture lines present should normally be included the restoration. However, it is rare that the size of the tooth will affect the design of the tooth preparation

Sarah S. is a young child who consistently presents as anxious, hypervigilant, and upset during dental visits. Sarah is often accompanied by her parent, who appears to be very concerned about the child and wants to be involved at all times in her evaluation and treatment. During this visit, Sarah's treatment requires an injection and a rubber dam application, which you anticipate may lead to increased anxiety. Which strategy would be the least effective in completing the rubber dam application? A. Tell-Show-Do B. Distraction C. Ask the child to be a helper D. Structure time E. Rehearsals

B, Of the choices, distraction would most likely be the least effective approach—the attention of a very anxious individual cannot typically be easily diverted. In such cases, distraction can have detrimental effects such as compromising rapport and/or increasing anxiety by failing to provide a positive coping experience. Providing education and coping strategies—increasing predictability, familiarity, and controllability—are typically more effective strategies in working with anxious patients

A patient is sitting in the chair immediately following an extraction. She says, "Thank you. That wasn't as bad as I expected, but my sister told me that the first night after having a tooth pulled is very painful. What if the medication you're giving me isn't strong enough?" Choose the most appropriate response. A. "Did she make you feel worried about that?" B. "It sounds like you're worried that you might not have enough pain relief when you're home." C. "I understand your concern." D. "Don't worry. I'll give you plenty of pain medicine." E. "It sounds like your sister had a unusually bad experience. Don't believe what others tell you, and certainly don't let that worry you. You'll be fine."

B, Of the options given, the best response would be to interpret what the patient is trying to communicate and reflect the communication back to him or her. This will gently encourage the patient to openly express and discuss the concern with the clinician. It also serves to establish an environment of openness and acceptance

Which of the following would not be expected to cause delayed healing of an extraction site? A. A patient older than 60 years of age B. A patient younger than 10 years of age C. A patient with diabetes D. A patient with a heavy smoking habit

B, Older age, diabetes, and smoking are risk factors for delayed healing

Principles of operant conditioning teach us that _____. A. If you praise your 5-year-old patient and reward him for keeping his legs still while you are drilling, this will make the child happy and more likely to like you and less likely to resist your requests. B. If you praise your 5-year-old patient and reward him for keeping his legs still while you are drilling, this will increase the likelihood that he will remain still in similar situations in the future. C. If you make the dental environment a childfriendly place, your young patient will be more comfortable. D. If you pair the dental chair with having a parent present, the child will be less likely to be anxious. E. None of the above.

B, Operant conditioning posits that behavior is largely influenced by the consequences associated with the particular behavior

The reshaping or recontouring of nonsupportive alveolar bone is called _____. A. Ostectomy B. Osteoplasty C. Osteography D. All of the above

B, Ostectomy is the removal of supporting alveolar bone. Osteoplasty is the reshaping or recontouring of nonsupporting alveolar bone

Instrumentation of the teeth to remove plaque, calculus and stains is defined as _____. A. Coronal polishing B. Scaling C. Gingival curettage D. Root planing

B, Polishing is used to remove plaque and stains from the teeth. Gingival curettage is used to remove the epithelial lining of a periodontal pocket. Root planing is used to create a smooth root surface through the removal of calculus and rough cementum. Scaling is used to remove plaque, calculus, and stains from the tooth

Which of the following microorganisms is frequently associated with localized aggressive periodontitis? A. Porphyromonas gingivalis B. Actinobacillus actinomycetemcomitans C. Actinomyces viscosus D. Streptococcus mutans

B, Porphyromonas gingivalis has been associated with chronic periodontitis. Actinomyces viscosus is usually associated with health or gingivitis. Streptococcus mutans is associated with dental caries. Actinobacillus actinomycetemcomitans has been associated with localized aggressive periodontitis

Which of the following depicts the usual order of extraction of teeth if serial extraction is chosen as the treatment to alleviate severe crowding? A. Primary second molars, primary first molars, permanent first premolars, primary canines B. Primary canines, primary first molars, permanent first premolars C. Primary first molars, primary second molars, primary canines D. Primary canines, permanent canines, primary first molars, permanent first premolars

B, Primary canines are extracted to encourage alignment of the crowded incisors. However, the incisors align and upright, borrowing space otherwise needed for eruption of the permanent canine. Primary first molars are then extracted to encourage eruption of the first premolar so it may be extracted to make room for the permanent canine to erupt

What is the radiographic sign of successful pulpotomy in a permanent tooth? A. Open apex B. That the apex has formed C. Loss of periradicular lucency D. No internal resorption

B, Pulpotomy is normally not recommended in permanent teeth unless root development is incomplete. If incomplete, the calcium hydroxide pulpotomy is recommended. This is performed in permanent teeth with immature root development and with healthy pulp tissue. The success is indicated when the root apex, if not completely formed, completes its full development. This procedure is only done on teeth free of symptoms

Which of the following reactions is least likely to be observed during orthodontic treatment? A. Root resorption B. Devitalization of teeth that are moved C. Mobility of teeth that are moved D. Development of occlusal interferences

B, Root resorption is common during orthodontic treatment, although lesions often repair on the root surface. Mobility of teeth is also common as the PDL reorganizes and widens during tooth movement. It is uncommon for teeth to become devitalized as a result of orthodontic movement unless they have also been substantially compromised by injury or infection

An adult patient presents with a 0.5 ¥ 0.5-cm submucosal mass in the posterior lateral tongue. Biopsy shows a neoplasm composed of glandlike elements and connective tissue elements. It is covered by normal-appearing epithelium. This could be which of the following? A. Oral wart B. Pleomorphic adenoma (mixed tumor) C. Granular cell tumor D. Idiopathic leukoplakia E. Peripheral giant cell granuloma

B, Salivary gland tumors present as submucosal masses. The combination of epithelial and connective tissue elements is indicative of pleomorphic adenomas, also known as mixed tumors. Oral warts and leukoplakias are surface or epithelial lesions. Peripheral giant cell granulomas are exclusively gingival lesions, and granular cell tumors are composed exclusively of cells with grainy or granular cytoplasm.

Which of the following is not a characteristic of sickle scalers? A. Two cutting edges. B. Rounded back. C. Cutting edges meet in a point. D. Triangular in cross section. E. Used for removal of supragingival deposits

B, Scalers have a pointed back; curettes have a rounded back, making them suitable for subgingival instrumentation

The radiolucent portions of the images on a processed dental x-ray film are made up of _____. A. Microscopic grains of silver halide B. Microscopic grains of metallic silver C. A gelatin on a cellulose acetate base D. Unexposed silver bromide

B, Silver halide in the emulsion of an exposed film is converted into grains of metallic silver in the developer.

When comparing pin retention with slot retention for a complex amalgam restoration, which of the following statements is false? A. Slots are used where vertical walls allow opposing retention locks. B. Slots provide stronger retention than pins. C. Slots and grooves can be used interchangeably. D. Pin retention is used primarily where there are few or no vertical walls.

B, Slots and pins may be used interchangeably. They both provide good secondary retention form. Slots are usually better when there exist box forms or vertical walls in the preparation, and pins are usually better when there are few or no vertical walls. The retention is similar for both

During the mixed dentition, a 1-mm diastema develops between the maxillary incisors. Which of the following is most likely? A. The diastema will need orthodontic intervention to be closed B. The diastema will resolve once the canines erupt C. The diastema will only resolve when all of the permanent teeth erupt D. The diastema will continue to widen as permanent teeth erupt

B, Small diastemas between the maxillary incisors of 2 mm or less will generally close on their own as more permanent teeth, specifically the canines, erupt. Presence of a midline diastema before canine eruption is referred to as "the ugly duckling stage."

Tight capillary cell junctions resulting in an added barrier to the entry of drugs is most characteristic of which organ or tissue? A. Adrenal gland B. Brain C. Heart D. Liver E. Lung

B, The brain has especially tight capillary junctions as well as glial cells that result in a blood-brain barrier.

The use of selective COX-2 inhibitors has recently been restricted or discontinued because of what type of adverse effects? A. Carcinogenesis B. Cardiovascular disorders C. Convulsive disorders D. Striated muscle disorders E. Skeletal disorders

B, The cardiovascular risks may be associated with adverse hematologic effects, but the exact mechanism is not yet known

Putting a force through which of the following points would cause pure translation of a tooth without rotation, tipping, or torque? A. Center of rotation B. Center of resistance C. Center of the bracket D. Apex of the root

B, The center of resistance is defined as the point at which force application will cause pure translation of a tooth.

To obtain the most geometrically accurate image, which of the following is false? A. The film should be parallel to the object. B. The central ray should be parallel to the object. C. The central ray should be perpendicular to the film. D. The object-to-film distance should be short. E. The object-to-anode distance should be long

B, The central ray should be perpendicular to the object.

Bones of the cranial base include which of the following? A. Maxilla, mandible, and cranial vault B. Ethmoid, sphenoid, and occipital C. Palatal, nasal, and zygoma D. Frontal and parietal

B, The cranial base includes, from anterior to posterior, the ethmoid, sphenoid, and occipital bones

Excessive vertical angulation causes _____. A. Overlapping B. Foreshortening C. Elongation D. Cone-cutting

B, The film should be parallel to the long axis of the tooth and the central ray of the beam should be perpendicular to both the film and the tooth. Increasing the vertical angulation foreshortens the image of the tooth

The _____ nerve block is useful for dental procedures involving the palatal soft tissues distal to the canine. A. Nasopalatine (NP) B. Greater palatine (GP) C. Long buccal (LB) D. Inferior alveolar (IA)

B, The greater palatine (GP) injection provides softtissue anesthesia of the hard palate from the junction of the premaxilla to the junction of hard and soft palate and from the gingival margin to the midline of the palate

Which of the following injections, when properly performed, does not lead to pulpal anesthesia? A. Inferior alveolar (IA) B. Lingual C. Posterior superior alveolar (PSA) D. Infraorbital (IO) (true anterior superior alveolar nerve block)

B, The inferior alveolar, PSA, and IO injections all lead to pulpal anesthesia when performed properly. The lingual injection leads to soft-tissue anesthesia only

When using a cervical-pull headgear, the forces generated on the maxillary first molar cause this tooth to move in which of the following ways? A. Mesially and to extrude B. Distally and to extrude C. Mesially and to intrude D. Distally and to intrude

B, The line of force generated by a cervical headgear will cause the maxillary first molar to move distally, usually also tip distally, and to extrude. A high-pull headgear would cause the molar to move distally and intrude.

The major mechanisms for the destruction of osseointegration of implants are _____. A. Related to surgical technique B. Similar to those of natural teeth C. Related to implant material D. Related to nutrition

B, The major causes for loss of osseointegrated implants are similar to those of natural teeth: poor hygiene, occlusal load, and the resultant inflammatory processes that occur.

The primary contraindication(s) for the use of a composite restoration is (are) _____. A. Occlusal factors B. Inability to isolate the operating area C. Nonesthetic areas D. Extension onto the root surface

B, The only constant contraindication for the use of composite is when the operating area cannot be properly isolated, thereby decreasing the potential success of the bond.

The pulp tissue of primary teeth _____. 1. In general, is smaller proportionately than permanent pulps in relation to tooth crown size. 2. Is closer to the outer surface of the tooth than the permanent teeth. 3. Follows the general surface contour of the crown. 4. Has the mesial pulp horn closer to the surface than the distal pulp horn. A. Only 1, 2, and 4 are correct. B. Only 2, 3, and 4 are correct. C. Only 1, 3, and 4 are correct. D. 1, 2, 3, and 4 are correct

B, The pulp chambers of primary teeth are proportionately larger compared to the size of the crown. This is significant because there is a higher risk of accidental pulp exposures on primary teeth. In particular, the mesial-buccal pulp horn of the first primary molar is close to the external surface of the tooth.

During local anesthetic administration, the patient should be placed in a _____ position. A. Trendelenburg B. Supine C. Reclined D. Semi-supine

B, The supine position is correct. This position will prevent fainting during or immediately after the injection of local anesthetic. Reclined or semisupine is not back far enough and Trendelenburg is too far

If the fluoride level in the drinking water is greater than 0.6 ppm at any age, no supplemental systemic fluoride is indicated. If the patient is less than 12 months old, no supplemental systemic fluoride is indicated, whatever the water fluoride level. A. The first statement is true and the second statement is true. B. The first statement is true and the second statement is false. C. The first statement is false and the second statement is true. D. The first statement is false and the second statement is false.

B, The systemic fluoride "Rule of 6s" states: a. If fluoride level is greater than 0.6 ppm, no supplemental systemic fluoride is indicated. b. If the patient is less than 6 months old, no supplemental systemic fluoride is indicated. c. If the patient is greater than 16 years old, no supplemental systemic fluoride is indicated. Therefore, the statement, "If the patient is less than 12 months old, no supplemental systemic fluoride is indicated" is false.

The optimal volume of local anesthetic solution delivered for a true anterior superior alveolar (ASA) nerve block is usually about _____. A. 0.5 mL B. 1.0 mL C. 1.5 mL D. 1.8 mL

B, The true anterior superior alveolar (ASA) nerve block, also called the infraorbital nerve block, requires a volume of one-half cartridge of local anesthetic solution, or about 1.0 mL.

The benzodiazepine receptors BZ1 and BZ2 are located on which ion channel? A. Calcium B. Chloride C. Magnesium D. Potassium E. Sodium

B, The two benzodiazepine receptor subtypes (targets for drugs such as diazepam) are located on the same chloride channel as is the GABAA receptor.

With regard to the mercury controversy related to the use of amalgam restorations, which statement is incorrect? A. There is lack of scientific evidence that amalgam poses health risks to humans except for rare allergic reactions. B. Alternative amalgam-like materials (with low or no mercury content) have promise about mercury. C. True allergies to amalgam rarely have been reported. D. Efforts are underway to reduce the environmental mercury to which people are exposed to lessen their total mercury exposure

B, There are no known alternative low- or nomercury systems that have been developed which provide the same properties or clinical performance as amalgam. The other statements are true

Closure of a 2-mm maxillary midline diastema should be accomplished orthodontically in an 8-year-old in which of the following circumstances? A. If the lateral incisors are missing B. If the space creates an esthetic concern and the child is being teased about it C. If there is also deep overbite present D. If mild crowding is also present

B, There is a high likelihood that a small diastema of 2 mm or less will close on its own over time as the permanent teeth erupt. However, if a child suffers psychological trauma because of esthetic concerns, the diastema can be closed. Parents should be informed of the reason for treatment and understand that there are some risks of performing orthodontic treatment that they are assuming

Triturating a dental amalgam will _____. A. Reduce the size of the alloy particles B. Coat the alloy particles with mercury C. Reduce the crystal sizes as they form D. Dissolve the alloy particles in mercury

B, Triturating (mixing) the amalgam particle with the mercury is intended to result in coating the particles with a surface of mercury and creating the desirable phases in the set amalgam. All of the alloy particle is not dissolved in the mercury and the size is not significantly reduced

In the conventional Class I composite preparation, retention is achieved by which of the following features? 1. Occlusal convergence 2. Occlusal bevel 3. Bonding 4. Retention grooves A. 2 and 4 B. 1 and 3 C. 1 and 4 D. 2 and 3

B, Typically, the Class I composite preparation has occlusally converging walls that provide primary retention form. The actual bonding also provides retention form. However, an occlusal bevel is not indicated on Class I preparations, nor are retention grooves utilized.

You are unsure of the location of an opaque mass seen over a molar root on a periapical view. A second view of the same region, made with the x-ray machine oriented more from the mesial, reveals that the object has moved mesially with respect to the molar roots on the first view. The location of the object is _____. A. Buccal to the roots B. Lingual to the roots C. In the same plane as the roots D. Insufficient information to form an opinion

B, Use the rule of "SLOB": Same Lingual, Opposite Buccal

Rectangular collimation is recommended because it _____. A. Deflects scatter radiation B. Decreases patient dose C. Increases film density D. Increases film contrast

B, Using a rectangular collimator restricts the area of the patient's face exposed to the size of the receptor, thus reducing more than half the patient exposure

The best way to carve amalgam back to occlusal cavosurface margin is to _____. A. Use visual magnification B. Use a discoid-cleoid instrument guided by the adjacent unprepared enamel C. Make deep pits and grooves D. Use a round finishing bur after the amalgam has set

B, Using the adjacent unprepared enamel at the cavosurface margin to guide the discoid carving instrument when carving away excess amalgam at the occlusal margin is the best way to develop the junction correctly.

When osseointegration occurs, which of the following best describes the implant-bone interface at the level of light microscopy following osseointegration? A. Epithelial attachment B. Direct contact C. Connective tissue insertion D. Cellular attachment

B, When evaluated by light microscopy, there appears to be direct contact at the bone-implant interface

When preparing a Class III or IV composite tooth preparation, which of the following is false regarding placement of retention form? A. Often involves gingival and incisal retention B. Is placed at the axiogingival line angle regardless of the depth of the axial wall C. May be needed in large preps D. Is usually prepared with a No. 1/4 round bur

B, When needed for large restorations, retention form usually consists of a gingival groove and incisal cove prepared with a small round bur (No. 1/4). The placement of the groove or cove is dependent on the DEJ, placing the retention 0.2 mm internal to the DEJ entirely in dentin. It is not placed at the axiogingival or axioincisal line angles if those line angles are deeper than ideal; otherwise, the retention form may be too deep or cause a pulpal exposure

Children in the primary dentition most often present with _____. A. An increased overbite B. A decreased overbite C. An ideal overbite D. A significant open bite

B, Young children often present with minimal overbite or anterior edge-to-edge relationship. Habits such as thumb-sucking increase the likelihood that less overbite will be present

Hypovolemic shock is dx with what BP?

BP 70/50 and R is 132/min

The ASIS's are in the same plane as the symphysis pubis.

Balanced Pelvis

Pt c/o of sudden numbness and drooping of the whole left side of face, and pain directly behind her left ear. She also has slurring of speech, dygeusia, hyperacusis, and difficulty drinking. She also had a viral infections 3 days prior. condition?

Bell's Palsy

Multiple myeloma

Bence-Jones, amyloid, back pain, anemia, renal dysfunction, elevated ESR, hypercalcemia, constipation, rouleaux

Macular degeneration

Bilateral central vision loss, drusen

Glucose-6-phosphate dehydrogenase deficiency

Bite cells, Heinz bodies, jaundice, dark urine

Hereditary spherocytosis

Black pigmented (calcium bilirubinate) gallstones, splenomegaly, reticulocytosis, osmotic fragility test, folic acid supplementation

Immune thrombocytopenic purpura

Bleeding, epistaxis, hemorrhagic bullae of oral cavity, autoimmune hemolytic anemia, prednisone, IVIG

Acute monocytic leukemia

Bleeding, gingival hyperplasia, skin lesions, positive alpha-naphthyl esterase

what exercise is used for patients with CP and patterning?

Bobarth

Graft-versus-host disease

Bone marrow transplantation, macropapular rash, blood-positive diarrhea, transaminitis, jaundice, donor T cells

Prostate cancer

Bone metastasis, cauda equina syndrome

Maxillary sinusitis

Brain abscess, anaerobes, headaches

Actinomycosis

Branching bacteria, penicillin, head and neck abscesses, yellow drainage of sulfur granules

_____________ Radiation is the majority of the beam that interacts with the tartget nucleus

Bremsstrahlung

Hemochromatosis

Bronzed skin, restrictive cardiomyopathy, hepatomegaly, recent-onset diabetes, arthropathy, hypogonadism, Listeria, Yersinia, HCC, phlebotomy

When giving an IA nerve block, the needle pierces which muscle?

Buccinator muscle

this type of exercises is for peripheral disease?

Buerger allen

Pulmonary granuloma

Bulls eye nodule

The local anesthetic agent that is most appropriate for use in most children is _____. A. 3% mepivacaine B. 2% mepivacaine with 1:20,000 levonordefrin C. 2% lidocaine with 1:100,000 epinephrine D. 0.5% bupivacaine with 1:200,000 epinephrine

C, 2% lidocaine with 1:100,000 epinephrine is the local anesthetic that allows the greatest volume to be administered safely. Therefore, it is the local anesthetic drug of choice for administration in children. Mepivacaine in either 2% or 3% allows less volume to be safely administered and bupivacaine is not FDA-approved for administration to children.

A test result that erroneously excludes an individual from a specific diagnostic or reference group is called _____. A. Erroneous B. False positive C. False negative D. Mistaken E. None of the above

C, A false positive test is a test result which erroneously assigns an individual to a specific diagnostic or reference group.

6. A finishing bur has how many blades compared to a cutting bur? A. Fewer blades. B. Same number of blades. C. More blades. D. Number of blades is unrelated to the bur type

C, A finishing bur is designed to provide a smoother surface and therefore has more blades than a cutting bur. The increased blade numbers results in a smoother cut surface

The success of an amalgam restoration is dependent on all of the following features of tooth/cavity preparation except _____. A. Butt-joint cavosurface margin that results in a 90-degree margin for the amalgam B. Adequate tooth removal for appropriate strength of the amalgam C. Divergent (externally) preparation walls D. Adequate retention form features to mechanically lock the amalgam in the preparation

C, A successful amalgam restoration requires 90-degree amalgam margins. Amalgam margins less than 90 degrees result in increased potential for fracture of the amalgam. Greater than 90-degree amalgam margins are good for the amalgam but the corresponding enamel margin will be less than 90 degrees and therefore potentially undermined and have potential for fracture. Since the amalgam is not bonded to the tooth, it must be retained in the tooth with undercuts, either in the primary or secondary preparation. An amalgam restoration needs a minimum of 1-mm thickness in nonstress areas and 1.5 to 2 mm in areas that may be under load. Therefore, the preparation must provide this dimension. Except for Class V amalgams, the prepared walls generally converge to the exterior. Thus, the prepared walls may diverge or converge externally.

Which of the following is true regarding crowding of the dentition? A. Crowding of the primary dentition usually resolves as the permanent teeth erupt. B. Spacing in the primary dentition usually indicates spacing will be present in the adult. C. Approximately 15% of adolescents have crowding severe enough to consider extraction of permanent teeth as part of treatment. D. Lower incisor crowding is more common in African-American than white populations.

C, According to data available, approximately 15% of adolescents have severe crowding that would require major amounts of expansion or extractions to resolve. The other statements are false: crowding in the primary dentition is very rare and would indicate crowding will occur in the permanent dentition; spacing in the primary dentition is normal; and African-Americans generally have less crowding than whites.

The administration of which compound will give "epinephrine reversal" (drop in blood pressure from epinephrine) if given prior to administration of epinephrine? A. Guanethidine B. Propranolol C. Phenoxybenzamine D. Tyramine

C, Alpha adrenoceptor blockers, like phenoxybenzamine, inhibit the vasoconstrictor effect of epinephrine but not the vasodilator effect of epinephrine. Therefore, the administration of alpha blockers results in epinephrine reversal. Propranolol would only block the vasodilator effect of epinephrine. Guanethidine and tyramine act largely at prejunctional sites and don't block adrenergic receptors.

What is the most objective clinical indicator of inflammation? A. Gingival color B. Gingival consistency C. Gingival bleeding D. Gingival stippling

C, Although changes in gingival color and consistency and loss of gingival stippling can be indicators of gingival inflammation, bleeding on probing is the most objective clinical indicator

Which of the following procedures would be considered the least invasive surgical treatment for TMJ complaints? A. Splint therapy B. Arthrocentesis C. Arthroscopy D. Disc removal E. Total joint replacement

C, Although less invasive, arthrocentesis and splint therapy are not considered surgical interventions

Which one of the following acids is generally recommended for etching tooth structure? A. Maleic acid B. Polyacrylic acid C. Phosphoric acid D. Tartaric acid E. EDTA

C, Although some of the self-etch bonding systems use milder acid, the primary acid system used for etching tooth structure is phosphoric acid.

Injecting a local anesthetic into an area of inflammation would have which effect? A. Increase the rate of onset of anesthesia. B. Decrease the rate of metabolism of the anesthetic. C. Reduce the net anesthetic effect of the drug. D. Reduce the vasodilator effect of the local anesthetic. E. Reduce the need for a vasoconstrictor with the local anesthetic.

C, An area of inflammation is an area of low pH. The acid environment would convert more of the drug into the charged form, making it less able to diffuse to the nerve cells. This would reduce the rate of onset and the net anesthetic effect of the drug.

A 1-year-old patient has his first dental examination. The dentist reviews with the parent when to expect the next teeth to erupt, teething, oral hygiene tips for toddlers, and discusses fluoride issues with bottled water and toothpaste. The term that describes this proactive approach to dental care is _____. A. Risk assessment B. Probability counseling C. Anticipatory guidance D. Preventive support counseling

C, Anticipatory guidance is counseling patients and parents regarding the child's home oral healthcare that is age-appropriate and is focused on prevention. Subjects to discuss with parents include: a. Oral hygiene b. Oral development c. Fluoride d. Diet and nutrition e. Oral habits f. Trauma and injury prevention

Pick the organism that is usually clinically sensitive to clarithromycin but not to penicillin V. A. Streptococcus viridans B. Leptotrichia buccalis C. Mycoplasma pneumoniae D. Streptococcus pneumoniae E. Streptococcus pyogenes

C, Because it lacks a cell wall, Mycoplasma pneumoniae is not sensitive to cell wall inhibitors such as penicillin V. The macrolides, such as clarithromycin, are ribosomal protein synthesis inhibitors that are effective against Mycoplasma pneumoniae. Streptococcus viridans, Streptococcus pneumoniae, and Streptococcus pyogenes are gram-positive cocci. Leptotrichia buccalis is a gram-negative oral bacillus

If a local anesthetic has a low pKa, then it will usually have a _____. A. Greater potency B. Higher degree of protein binding C. Faster onset of action D. Greater vasodilating potential

C, By definition, a low pKa means a fast onset of action. Hydrophobicity and protein binding directly affect duration of action and potency

What are the major organic constituents of bacterial plaque? 1. Calcium and phosphorous 2. Sodium and potassium 3. Polysaccharides and proteins 4. Glycoproteins and lipids A. 1 and 2 B. 2 and 3 C. 3 and 4 D. 2 and 4

C, Calcium, phosphorous, sodium, and potassium are inorganic components of dental plaque. Polysaccharides, proteins, glycoproteins, and lipids are organic components of dental plaque

Which of the following factors are involved in the cognitive appraisal of a threat? A. Interference, adaptability, longevity, and reactance B. Adaptability, preventability, inevitability, and constancy C. Controllability, familiarity, predictability, and imminence D. Validity, reliability, adaptability, and predictability E. Accountability, reliability, validity, and familiarity

C, Controllability, familiarity, predictability, and imminence are significant factors influencing the cognitive appraisal of stress

What is the mechanism of the analgesic action of aspirin? A. Stimulates μ opioid receptors B. Blocks histamine H2 receptors C. Inhibits cyclooxygenase D. Inhibits lipoxygenase E. Blocks sodium channels in nerves

C, Cyclooxygenase (COX) is a key enzyme in the synthesis of prostaglandins. Prostaglandins, including PGE2 and PGF2α, are important mediators for such functions as pain, and are a product of COX. Aspirin inhibits both COX-1 and COX-2.

Ectopic lymphoid tissue would most likely be found in which of the following sites? A. Hard gingiva B. Soft gingiva C. Floor of mouth D. Dorsum of tongue E. Vermilion of the lip

C, Ectopic (normal tissue, abnormal site) lymphoid tissue is commonly seen in floor of the mouth as well as in posterior lateral tongue, soft palate, and tonsilar pillar. It appears as one or more small, dome-shaped yellow nodules

In the treatment of an acute periodontal abscess, the most important first step is to _____. A. Prescribe systemic antibiotics B. Reflect a periodontal flap surgery C. Obtain drainage D. Prescribe hot salt mouth washes

C, Establishment of drainage is the first step in treating an acute periodontal abscess. The patient may then use self-applied mouth rinses and be prescribed antibiotics if there is evidence of systemic involvement (e.g., fever, lymphadenopathy). A flap would be reflected in a subsequent appointment if the abscess did not resolve and became a chronic problem

All of the following may be indications to consider extraction of permanent teeth in an orthodontic patient except _____. A. Excessive crowding B. Class II interarch relationship C. Flat lip profile D. Anterior open bite

C, Excessive crowding may necessitate extractions. Also, extraction of maxillary premolars may be indicated to camouflage a Class II molar relationship. Anterior open bites may be improved by uprighting anterior teeth to increase overbite. Flat lips will not be improved by extraction of permanent teeth but other considerations may still necessitate extraction even in those patients

What is the half-life of Hg in the human body? A. 5 days B. 25 days C. 55 days D. 85 days E. 128 days

C, Fifty-five days is the half-life of mercury in the body.

In a patient with incomplete cleft palate, which of the following aspects is most likely to remain open? A. The anterior aspect B. The middle aspect C. The posterior aspect D. The right aspect

C, Fusion of the palate proceeds from anterior to posterior, so any disturbance that occurs during that time will stop fusion at that point, leading to an opening posteriorly

Which of the following best distinguishes periodontitis from gingivitis? A. Probing pocket depth B. Bleeding on probing C. Clinical attachment loss D. Presence of suppuration

C, Gingivitis is characterized by inflammation of the gingival tissues with no loss of clinical attachment. Periodontitis is characterized by inflammation with loss of clinical attachment

Herpes simplex virus is the cause of which of the following? A. Minor aphthous ulcers B. Herpetiform aphthae C. Herpes whitlow D. Herpangina E. Herpes zoster

C, Herpes whitlow is a term used for secondary herpes simplex infections that occur around the nail bed. The cause of aphthous ulcers is unknown, herpangina is caused by Coxsackie virus, and herpes zoster is caused by varicellazoster virus.

The imaging evaluation of the temporomandibular joint is most likely to include any of the following except _____. A. Panoramic radiographs B. TMJ tomograms C. Xeroradiography D. Magnetic resonance ima

C, Imaging tools used in the evaluation of TMJ pathology include panoramic radiographs, traditional and computer generated tomograms, MRIs, nuclear imaging, and arthography

A group of researchers undertook a study to assess the relationship between squamous cell carcinoma and chewing tobacco. The researchers determined past exposure records among subjects who had been diagnosed with the disease. This type of study was a _____. A. Clinical trial B. Community trial C. Retrospective cohort study D. Case control study E. Randomized clinical trial

C, In a retrospective cohort study, the investigator chooses a sample of individuals who have the outcome of interest (in this case, squamous cell carcinoma) and then look into the past for possible variables that may have caused the disease (e.g., chewing tobacco).

The following teeth are erupted in an 8-year-old patient. What is the space maintenance of choice? A. Band-loop space maintainer. B. Lower lingual holding arch. C. Nance holding arch. D. Distal shoe space maintainer

C, In space maintenance, the clinician must always be mindful of the exfoliation sequence of teeth. In this situation, the authors would normally exfoliate prior to the eruption of the second permanent premolar, tooth #13. If a band loop space maintainer were used, there may be no anterior abutment if there is a normal exfoliation sequence. This could result in mesial tipping of the permanent molar and space loss. A Nance holding arch or a palatal holding arch would be an appropriate choice.

In the primary dentition, the mandibular foramen is located where in relation to the plane of occlusion? A. Higher than the plane of occlusion B. Much higher than the plane of occlusion C. Lower than the plane of occlusion D. The same level as the plane of occlusion

C, In the primary dentition patient, the mandibular foramen is located lower than the plane of occlusion. Therefore, mandibular block injections for these patients are made somewhat lower than as is done for the adult patient

In this section of a scientific article, the researcher interprets and explains the results obtained. A. Summary and conclusion B. Results C. Discussion D. Abstract E. None of the above

C, In the results section the researcher describes the specific findings and actual outcomes of the project but does not interpret them. The interpretation and analysis of the results are part of the discussion, where the researcher attempts to explain his results.

Light smokers are likely to have less severe periodontitis than heavy smokers. Former smokers are likely to have more severe periodontitis than current smokers. A. Both statements are true. B. Both statements are false. C. The first statement is true, the second statement is false. D. The first statement is false, the second statement is true.

C, Individuals who smoke cigarettes are more likely to have periodontal disease than are nonsmokers. The number of cigarettes smoked and the number of years of smoking affect the severity of disease. Former smokers usually have less disease than do current smokers

Which local anesthetic is most hydrophobic and has the highest degree of protein binding? A. Mepivacaine B. Lidocaine C. Bupivacaine D. Procaine

C, Lipid solubility (therefore, hydrophobicity) and protein binding are the most important factors in determining duration of action of a local anesthetic. Bupivacaine has the longest duration of action of the listed local anesthetics and also has the highest hydrophobicity; it is bound 95% to protein. The other listed agents have lower hydrophobic qualities and are 75% or less bound to protein.

Which antihypertensive drug also increases bradykinin levels? A. Candesartan B. Furosemide C. Lisinopril D. Metoprolol E. Nifedipine

C, Lisinopril, by virtue of the fact that it inhibits angiotensin-converting enzyme (ACE) (also called peptidyl dipeptidase), inhibits the breakdown of bradykinin.

According to Malamed, the maximum local anesthetic dose of lidocaine (with or without vasoconstrictor) is _____. A. 1.5 mg/kg. B. 2.0 mg/kg. C. 4.4 mg/kg. D. 7.0 mg/kg

C, Malamed recommends that 4.4 mg/kg (2.0 mg/lb) of lidocaine be the maximum administered, regardless of whether vasoconstrictor is in the formulation. The package insert for lidocaine allows up to 7 mg/kg when lidocaine is packaged with vasoconstrictor

The nevoid basal cell carcinoma syndrome includes multiple basal cell carcinomas, bone abnormalities, and which of the following? A. Osteomas B. Café-au-lait macules C. Odontogenic keratocysts D. Hypoplastic teeth E. Lymphoma

C, Multiple odontogenic keratocysts are part of the nevoid basal cell carcinoma syndrome

A 43-year-old-male patient presents with an asymptomatic anterior palatal swelling. A radiograph shows a 1 × 1-cm lucency and divergence of tooth roots #8 and #9. All teeth in the area are vital. This is most likely a(an) _____. A. Periapical granuloma B. Aneurysmal bone cyst C. Nasopalatine duct cyst D. Globulomaxillary lesion E. Dermoid cyst

C, Nasopalatine duct cysts are anterior midmaxillary lesions that occur in the nasopalatine canal. The associated lucency is often heart-shaped because of the superimposition of the nasal spine over the lesion. They do not devitalize teeth

Which of the following are antigen-presenting cells? A. Neutrophils B. T-lymphocytes C. Macrophages D. Plasma cells

C, Neutrophils are one of the primary defense cells of the innate immune system. T-lymphocytes are important activators of the specific (adaptive) immune system. Macrophages are antigenpresenting cells. Plasma cells produce antibodies

When a diagnosis of odontogenic keratocyst is made, the patient should be advised as to _____. A. The need for full-mouth extractions B. The association with colonic polyps C. The associated recurrence rate D. The likelihood of malignant transformation E. The need for additional laboratory studies

C, Odontogenic keratocysts are notable because of their recurrence rate, their aggressive clinical behavior, and their occasional multiplicity. When multiple, they may be part of the nevoid basal cell carcinoma syndrome

During admission, a patient interrupts you on a number of occasions with stories about past dental experiences while you are attempting to take a complete medical history. Your best response would be _____. A. Say nothing, listen to the patient, and finish your intake as best you can. B. Say, "I'd like to focus on your present experience and right now I need to know your medical history." C. Say, "It seems like you've had some important experiences and I would like to hear more about them, but first, let's discuss this health questionnaire before we address it, okay?" D. Say, "I don't need to know the details of your dental history. Please inform me of the experiences asked about in the questionnaire." E. Say, "We have about 30 minutes to complete this questionnaire and get started in your examination, so let's focus on that."

C, Of the options given, it is best to acknowledge that the patient is trying to convey information that is important to him or her and establish that there will be a time to talk about those issues, while gently redirecting him or her to the task at hand

Methemoglobinemia is an adverse effect associated with which local anesthetic due to its metabolism to o-toluidine? A. Lidocaine B. Mepivacaine C. Prilocaine D. Bupivacaine E. Benzocaine

C, Only prilocaine is metabolized to o-toluidine

Which of the following is the most important preventive and therapeutic procedure in periodontal therapy? A. Professional instrumentation B. Subgingival irrigation with chlorhexidine C. Patient-administered plaque control D. Surgical intervention

C, Patient cooperation and effectiveness in removing bacterial plaque is of primary importance in maintaining a healthy periodontium

What features best characterize the predominant microflora associated with periodontal health? A. Gram-positive, anaerobic cocci and rods B. Gram-negative, anaerobic cocci and rods C. Gram-positive, facultative cocci and rods D. Gram-negative, facultative cocci and rods

C, Periodontal health is characterized by a microflora dominated by gram-positive, facultative cocci and rods

Restoration of an appropriate proximal contact results in all of the following except _____. A. Reduction/elimination of food impaction at the interdental papilla B. Provides appropriate space for the interdental papilla C. Provides increased retention form for the restoration D. Maintenance of the proper occlusal relationship

C, Proper proximal contacts reduce the potential for food impaction, thereby preserving the health of the underlying soft tissue. A missing proximal contact may result in tooth movement that will have an adverse effect on the occlusal relationship of the tooth. Having a correct contact does not enhance the retentive properties of the restorative material.

The best strategy for addressing dental fear that is based upon distrust of the dentist is to _____. A. Use distraction techniques B. Use cognitive coping strategies C. Enhance informational and behavioral control D. Teach diaphragmatic breathing E. Reassure the patient that he or she can trust you

C, Providing the patient with information and control over his or her environment is likely to contribute to increased trust over time. Avoiding the issue of trust or providing reassurance that the patient can trust you without evidence is likely to maintain poor trust

Which of the following methods of radiographic assessment are best for identifying small volumetric changes in alveolar bone density? A. Bitewing B. Periapical C. Subtraction D. Panoramic

C, Radiographs must be taken in a standardized format at repeated visits to be assessed for small changes in bone density over time, using subtraction radiography. Radiographs are usually standardized by using a bite registration block to relocate the x-ray at the same place and angulation each time.

Fanconi syndrome from outdated tetracyclines affects predominantly which organ? A. Brain B. Heart C. Kidney D. Pancreas E. Stomach

C, Renal tubular acidosis, aminoaciduria, and hyperphosphaturia are some of the manifestations of proximal tubule damage in Fanconi syndrome

According to Scammon's growth curves, which of the following tissues has a growth increase that can be used to help predict timing of the adolescent growth spurt? A. Neural tissues B. Lymphoid tissues C. Reproductive tissues

C, Reproductive tissues grow at the same time as the adolescent growth spurt and the appearance of secondary sexual characteristics can be used to help predict the timing of growth.

When placement of proximal retention locks in Class II amalgam preparations is necessary, which of the following is incorrect? A. One should not undermine the proximal enamel. B. One should not prepare locks entirely in the axial wall. C. Even if deeper than ideal, one should use the axial wall as a guide for proximal lock placement. D. One should place locks 0.2 mm inside the DEJ to ensure that the proximal enamel is not undermined.

C, Retention locks, when needed in Class II amalgam preparations, should be placed entirely in dentin, thereby not undermining the adjacent enamel. They are placed 0.2 mm internal to the DEJ, are deeper gingivally (0.4 mm) than occlusally (i.e., they fade out as they extend occlusally), and translate parallel to the DEJ. If the axial wall is deeper than normal, the retention lock is not placed at the axiofacial or axiolingual line angles but, rather, is positioned 0.2 mm internal to the DEJ. If placed at the deeper location, it may result in pulp exposure, depending on the location of the axial wall depth.

X-ray film is composed of _____. A. Silver halide crystals suspended in plastic and coated on a gelatin base B. Sodium thiosulphate crystals and suspended within a plastic base C. A plastic base coated with silver halide crystals suspended in gelatin D. Fluorescent particles that react to x-radiation

C, Silver halide is not fluorescent, and thus choices 1 and 2 are incorrect

Doubling the force applied at the bracket of a tooth would have what effect on the moment affecting tooth movement? A. The moment would decrease by 50%. B. The moment would not change. C. The moment would double. D. The moment would increase by four times

C, Since M = Fd, doubling the force would double the moment, or tendency to rotate, tip, or torque.

Which of the following is true regarding the possibilities for reconstruction of an atrophic edentulous ridge prior to denture construction? A. Dental implants are used only as a last resort after bone grafting attempts have failed. B. Distraction osteogenesis is too new a technique to be applied to ridge augmentation. C. Potential bone graft harvest sites for ridge reconstruction include rib, hip, and chin. D. The need for ridge augmentation is more common in the maxilla than in the mandible

C, Sites commonly used for the reconstruction of the atrophic mandibular ridge are dictated by the deficiency and include chin, hip, ribs, prosthetic materials, and donor bone (human and bovine). Dental implants are commonly used, not only as a last resort. The use of distraction of ridge augmentation has been reported and is useful in certain applications. The mandibular alveolar ridge is more problematic in terms of resorption and denture retention, which more commonly necessitates reconstructive measures

The following are characteristics of a postpalatal seal of complete dentures, except which one? A. Compensates for shrinkage of the acrylic resin caused by its processing. B. May reduce the gag reflex. C. Improves the stability of the maxillary denture. D. It is most shallow in the midpalatal suture area

C, Stability is resistance to movement toward the residual ridge. The function of the posterior palatal seal is to improve retention, not stability. Stability is determined by the size, height, or shape of the ridge

Which of the following is the primary reason for splinting teeth? A. For esthetics B. To improve hygiene C. For patient comfort D. As a preventive measure

C, Teeth are usually splinted to improve patient comfort during mastication

Factors that affect the success of dentin bonding include all of the following except _____. A. Dentin factors such as sclerosis, tubule morphology, and smear layer B. Tooth factors such as attrition, abrasion, and abfraction C. Material factors such as compressive and tensile strengths D. C-factor considerations

C, Tensile and compressive strengths may have relevance for composite materials but not for dentin bonding systems. The success of bonding is dependent on the various dentin structural factors, tooth factors, polymerization shrinkage, C-factor considerations, and technique sensitivity

The most common mandibular surgical osteotomy to advance the mandible is _____. A. A LeFort I osteotomy B. A segmental maxillary osteotomy C. A bilateral sagittal split osteotomy D. An intraoral vertical ramus osteotomy

C, The BSSO is the most commonly used osteotomy for mandibular advancement

Placing the toothbrush bristles at a 45-degree angle on the tooth and pointing apically so the bristles enter the gingival sulcus describes which brushing technique? A. Charter B. Stillman C. Bass D. Roll

C, The Bass technique of brushing is designed to direct the bristles of the brush toward the gingival sulcus.

According to the CDC, the acceptable water quality in a dental office should be _____. A. < 125 CFU/mL B. < 250 CFU/mL C. < 500 CFU/mL D. < 750 CFU/mL E. < 1000 CFU/mL

C, The CDC recommends, at a minimum to meet nationally recognized drinking water standards, less than 500 colony-forming units (CFUs) of heterotrophic bacteria per milliliter. In 1995, the ADA addressed the dental water concern by asking manufacturers to provide equipment with the ability to deliver treatment water with < 200 CFU/ml of unfiltered output from waterlines

Miller Class I recession defects can be distinguished from Class II defects by assessing the _____. A. Location of interproximal alveolar bone B. Width of keratinized gingiva C. Involvement of the mucogingival junction D. Involvement of the free gingival margin

C, The Miller classification system for mucogingival defects takes into consideration the degree of recession (whether or not it extends to the mucogingival junction) and presence or absence of bone loss in the interdental area. Both Class I and Class II defects are characterized by no loss of bone in the interproximal areas. In Class I defects, the marginal tissue recession does not extend to the mucogingival junction. In Class II defects, recession does extend to or beyond the mucogingival junction

The Schwann cell is the cell of origin for which of the following tumors? A. Odontogenic myxoma B. Rhabdomyoma C. Neurofibroma D. Mixed tumor E. Leiomyoma

C, The Schwann cell is of neural origin and gives rise to one of several neoplasms, including neurofibroma and Schwannoma

A patient is administered haloperidol. Along with the haloperidol, the patient also receives benztropine. What is the most likely reason for administering the benztropine? A. To reduce the effects of histamine release B. To aid in the therapeutic response to haloperidol C. To reduce the motor adverse effects of haloperidol D. To overcome a decrease in salivary flow resulting from haloperidol E. To reduce the rate of kidney excretion of haloperidol

C, The antimuscarinic action of benztropine tends to reduce the Parkinsonlike symptoms and some other motor symptoms caused by haloperidol, a dopamine receptor blocker. It does not improve the antipsychotic effect of haloperidol. Histamine release appears to play little role in this interaction. Benztropine actually reduces salivary flow and xerostomia can easily result from its administration. Benztropine has little effect on renal clearance of haloperidol

Your patient is 8 years old. Tooth #8 was avulsed and you replanted it within 30 minutes. What is the best splint to use? A. Rigid fixation for 7 days B. Rigid fixation for 2 months C. Nonrigid fixation for 7 days D. Nonrigid fixation for 2 months

C, The appropriate splint for an avulsed tooth is a nonrigid splint, which is left in place for about 7 to 14 days. A 0.016 × 0.022 stainless steel orthodontic wire, a 0.018 round stainless steel wire, and a monofilament nylon (20- to 30-lb test) line are considered nonrigid. Long-term rigid splinting of replanted teeth increases risk of replacement root resorption (ankylosis). Rigid splinting is indicated for root fractures and remains in place for 2 to 3 months. A 0.032-0.036 stainless steel wire is considered a rigid splint.

It is important that the film base be _____. A. Opaque B. Very rigid C. Flexible D. Completely clear E. Sensitive to x-rays

C, The base needs to be flexible to go through automatic processors and be put into film mounts. Usually, the base is not completely clear and it is the emulsion that is sensitive to x-rays

A correlation analysis shows that as the income of the population increases, the number of decayed teeth decreases. Therefore, an expected value for this correlation coefficient (r) would be _____. A. 0 B. 1 C. −1 D. 2 E. −2

C, The correlation coefficient (r) quantifies the relationship between variables (x and y). A positive correlation coefficient indicates that the variables increase in the same direction; a negative correlation coefficient indicates that the variables vary in opposite directions. The correlation coefficient ranges from −1 to +1.

Which drug blocks H1 histamine receptors but is least likely to cause sedation? A. Diphenhydramine B. Hydroxyzine C. Fexofenadine D. Albuterol E. Famotidine

C, The first three choices are all H1 histamine receptor blockers. Fexofenadine, however, is largely excluded from the central nervous system, unlike diphenhydramine and hydroxyzine. Albuterol is a β2 adrenergic receptor agonist. Famotidine is a H2 histamine receptor antagonist

How many human drug testing phases are carried out before a drug is marketed? A. One B. Two C. Three D. Four

C, The fourth phase constitutes postmarketing surveillance.

When the gingival margin is gingival to the CEJ in a Class II amalgam preparation, the axial depth of the axiogingival line angle should be _____. A. 0.2 mm into sound dentin B. Twice the diameter of a No. 245 carbide bur C. 0.75 to 0.80 mm D. The width of the cutting edge of a gingival marginal trimmer

C, The guide for axial wall depth for a typical Class II preparation that has a gingival margin occlusal to the CEJ is 0.2 to 0.5 mm internal to the DEJ— the greater depth is necessary when placing retention locks. However, when there is no enamel proximally, the axial wall needs to be deep enough internally to provide for adequate strength of the amalgam material as well as to have room to place retention locks, if needed. This depth is approximately 0.75 mm.

Your patient indicates that he is taking medication for atrial fibrillation. He reports that a blood test has indicated that he has an INR number of 4.0. An emergency dental extraction is now required. Which postoperative medication would pose the greatest risk for an adverse effect in this patient? A. Acetaminophen B. Amoxicillin C. Aspirin D. Codeine E. Ibuprofen

C, The international normalized ratio (INR) value indicates that the patient has received anticoagulant therapy for his atrial fibrillation. Aspirin increases the risk of postsurgical bleeding. The combination of increase in prothrombin time, surgery, and the antiplatelet effect of aspirin make aspirin contraindicated in this situation. Ibuprofen's effect on the platelet is reversible, whereas the effect of aspirin on the platelet is irreversible. Thus, aspirin poses a greater risk than does ibuprofen in this situation

Your patient is a 23-year-old college student whom you suspect may have sustained a mandible fracture during an altercation. Which of the following is false? A. At least two x-rays should be obtained. B. The most common x-ray obtained would be a panoramic radiograph. C. The most likely area for this patient's mandible to be fractured is the mandibular dental alveolus. D. Point tenderness, changes in occlusion, step deformities, and gingival lacerations should all be noted on physical exam.

C, The mandibular condyle is the most common location of mandibular fractures. The alveolus, ramus, and coronoid are the least common sites

The third molar impaction most difficult to remove is the _____. A. Vertical B. Mesioangular C. Distoangular D. Horizontal

C, The most difficult impaction to remove is the distoangular tooth. This is because the withdrawal pathway runs into the ramus of the mandible and requires greater surgical intervention

Which of the following is the most likely cause pulpal necrosis following trauma to a tooth? A. Ankylosis B. Calcific metamorphosis C. Pulpal hyperemia D. Dilaceration

C, The other three answers may occur as the result of trauma but do not cause loss of vitality. Pulpal hyperemia causes increased intrapulpal pressure and swelling, which may result in an interruption of the pulp's blood supply. Without an adequate blood supply, the pulp becomes necrotic. This process can take time, and symptoms (either radiographic or clinical) may not present for weeks or even months. Typically, follow-up examination and radiographs are indicated at 1-, 2-, and 6- month intervals following a traumatic incident

All of the following describe lidocaine as packaged in dental cartridges except _____. A. Provided in a 2% solution B. Provided with or without epinephrine C. Has a pKa = 8.1 D. Has a rapid onset

C, The pKa of lidocaine is 7.9. It is packaged as a 2% solution both with and without epinephrine and has a rapid onset of action.

Causes of postoperative sensitivity with amalgam restorations include all of the following except _____. A. Lack of adequate condensation, especially lateral condensation in the proximal boxes B. Voids C. Extension onto the root surface D. Lack of dentinal sealing

C, The primary causes of postoperative sensitivity for amalgam restorations are voids (especially at the margins), poor condensation (that may result in void), or inadequate dentinal sealing. Extension onto the root surface does not necessarily result in increased sensitivity

Which of the following statements is true regarding the choice between doing a composite or amalgam restoration? A. Establishing restored proximal contacts is easier with composite. B. The amalgam is more difficult and techniquesensitive. C. The composite generally uses a more conservative tooth/cavity preparation. D. Only amalgam should be used for Class II restorations

C, The restoration of a proximal contact is easier with amalgam than with composite. Amalgam is easier to use and is less technique-sensitive. Either material can be used for Class II restorations. Because an amalgam restoration requires a tooth preparation that has (1) a specified depth (for strength of the amalgam), (2) cavosurface marginal configurations that result in 90-degree amalgam margins, and (3) an undercut form to its walls or secondary retention form features, they require more tooth structure removal than do composite tooth preparations. Composite tooth preparations require (1) removal of the fault, defect, or old material, (2) removal of friable tooth structure, and (3) no specific depths— they are more conservative

The size of the x-ray tube focal spot influences radiographic _____. A. Density B. Contrast C. Resolution D. Magnification E. Both C and D

C, The smaller the focal spot size, the greater the resolution. Density, contrast, and magnification are unchanged, other factors remaining equal

A wire extending from the molars to the incisors is activated to intrude the incisors. What is the side effect on the molars? A. The molars will tip forward and intrude B. The molars will rotate mesiobuccally C. The molars will tip distally and extrude D. The molars will rotate distobuccally

C, The sum of the forces and moments on an appliance must equal zero. If the incisors intrude, the molars will extrude. These two forces form a couple with a moment in one direction. The molars will experience a couple in the opposite direction, which will cause them to tip distally

Trauma from occlusion refers to _____. A. The occlusal force B. The damage to the tooth C. The injury to the tissues of the periodontium D. The widened periodontal ligament

C, The term trauma from occlusion refers to the tissue injury that occurs when occlusal forces exceed the adaptive capacity of the tissues. An occlusion that produces such an injury is called a traumatic occlusion. The tooth may become damaged as a result of excessive occlusal forces. The periodontal ligament also may become widened as a result of the force

The modified Widman flap uses three separate incisions. It is reflected beyond the mucogingival junction. A. Both statements are true. B. Both statements are false. C. First statement is true. Second statement is false. D. First statement is false. Second statement is true.

C, Three incisions are made in the modified Widman flap—internal bevel, crevicular, and interdental. It is designed to provide exposure of the tooth roots and alveolar bone. However, the flap is not reflected beyond the mucogingival junction

Your patient is continually taking a small daily dose of aspirin (82 mg) prescribed by the patient's physician. The object of this therapy is most likely what mechanism? A. To mimic the effect of endogenous endorphins B. To inhibit the production of prostaglandin E1 C. To inhibit the production of thromboxane A2 D. To inhibit the production of arachidonic acid E. To inhibit the production of leukotrienes

C, Thromboxane A2 increases platelet aggregation. Its inhibition is the target of low-dose aspirin which inhibits cyclo-oxygenase. Inhibition of this enzyme leads to a reduction in important down stream products, including thromboxane A2

Which of the following is considered a reversible carious lesion? A. The lesion surface is cavitated. B. The lesion has advanced to the dentin radiographically. C. A white spot is detected upon drying. D. The lesion surface is rough or cha

C, When an alteration (a break in continuity) occurs to the tooth surface from a carious attack, restoration is usually necessary. When a lesion is evident in the dentin with an x-ray, the lesion usually needs a restoration

The reason to invert a rubber dam is _____. A. To prevent the dam from tearing B. To prevent the underlying gingival from accidental trauma C. To provide a complete seal around the teeth D. All of above

C, When the rubber dam edge around the tooth is turned gingivally (inverted), it significantly reduces the leakage of moisture occlusally, thereby sealing around the tooth better and resulting in a better isolated operating area

Bremsstrahlung radiation results from _____. A. X-rays interacting with electrons B. Electrons interacting with electrons C. Electrons interacting with nuclei D. L shell electrons falling into the K shell E. Photons interacting with nuclei F. Photons converting into electrons

C, X-ray photons (Bremsstrahlung radiation) results from the interaction of high-speed electrons with tungsten nuclei in the target

X-rays are produced in most conventional dental x-ray machines _____. A. Continuously during operation B. When there is a large space charge C. Half the time during operation D. When the anode carries a negative charge E. Only when the beam is collimated F. Only during the first half of each second

C, X-rays are produced in most dental x-ray machines half the time (i.e., in bursts at the rate of 60 per second, each lasting 1/120th second) due to the alternating current supplied to the tube.

In the sub-occipital triangle, where is the Obliquus Capitus Superior located?

C1 transverse process to occiput

calcification in a joint line

CPPD chondrocalcinosis pseudo-gout

the deposition of calcium nodules in the skin, exaggerated vasoconstriction in the hands, with fingers undergoing white-blue-red color transitions in the cold difficulty swallowing, skin thickening on the fingers and dilatedcapillaries on the face, hands and mucous membranes all describe what condition?

CREST Syndrome

Swimmer's view posit?

CT junction Cass = 8x10 FFD= 40 Tilt = 5 deg caud (ft)

what is the number one mineral deficiency in the US?

Calcium

Venous thromboembolism

Cancer patients, low molecular weight heparin

Neutral Lateral cervical Posit?

Cass = 10x12 FFD= 72 Tilt = 0

Lateral T/S posit?

Cass = 14x17 FFD= 40 Tilt = 0

Lateral chest posit?

Cass = 14x17 FFD= 40 Tilt = 0

PA chest posit?

Cass = 14x17 FFD= 72 Tilt = 0

AP T/S posit?

Cass = 7x17 FFD= 40 Tilt = 0

Lateral L5/S1 spot posit?

Cass = 8x10 FFD= 40 Tilt = 0

APLC posit?

Cass = 8x10 FFD= 40 Tilt = 15 deg (hd)

Cervical Articular pillar posit?

Cass = 8x10 FFD= 40 Tilt = 35 deg ceph

Cervical Extnesion posit?

Cass = 8x10 (sideways) FFD= 72 Tilt = 0

this part of the xray tube produces more xrays has a larger effective focal spot less detail and used on thicker body parts

Cathode of xray

Most common reason for failure of a restoration in primary teeth?

Cavity prep

Secondary syphillis

Centrifugal rash, generalized lymphadenopathy, Jarisch-Herxheimer reaction

Rheumatoid arthritis

Cervical spine pain, hyperreflexia, morning stiffness, anti-CCP antibodies, swan neck/boutonnière deformities, osteoporosis

Pericarditis

Chest pain improved with sitting up, PR depression, diffuse concave up ST elevation

Amaurosis fugax

Cholesterol emboli (Hollenhorst bodies), carotid artery disease, transient loss of vision, whitened edematous retina, curtain over eye, TIA

Tropical sprue

Chronic diarrhea, megaloblastic anemia, hyperactive bowel sounds, villous blunting, chronic inflammatory cells

Gout

Chronic kidney disease, hyperuricemia, spindle-shaped birefringent crystals

Esophageal varices

Cirrhosis, beta blocker prophylaxis

Wilson's disease (hepatolenticular degeneration)

Cirrhosis, dysarthria, depression, personality change, Fanconi syndrome, zinc, direct hyperbilirubinemia

Alpha-1 antitrypsin deficiency

Cirrhosis, panacinar emphysema, PAS-positive diastase digestion-negative hepatocytes

what type of scatter correlates to film fog/

Classical; Thompson; Coherent Scatter

What type of torque is used to correct the open wedge?

Clockwise or Counterclockwise torque

What type of exercise is performed with the hand or foot fixed and cannot move, usually stays in constant contact with the surface?

Closed Chain Exercise

Acromegaly

Coarse facial features, doughy hands, skin tags, uncontrolled hypertension, carpal tunnel syndrome, pituitary adenoma

what deficiency causes a positive shilling test megaloblastic anemia and pernicious anemia

Cobalamin (B12)

Osteosarcoma

Codman's triangle, sunburst pattern

LeFort III

Complete separation of midface including nasoethmoidal complex, zygomas, and the maxilla

what is the name of the primary form of scatte radtion?

Compton Scatter

Wernicke's encephalopathy

Confusion, ataxia, ophthalmoplegia, thiamine deficiency, glucose administration contraindication

Rotor syndrome

Conjugated hyperbilirubinemia, jaundice, dark urine

Dubin-Johnson syndrome

Conjugated hyperbilirubinemia, jaundice, weakness, dark granular pigment in hepatocytes, urinary coproporphyrin

Reiter's syndrome

Conjunctivitis, urethritis, arthritis, mucocutaneous lesions

Vitamin D overdose

Constipation, abdominal pain, polyuria, polydipsia, sarcoidosis

Hypersensitivity pneumonitis

Cough, dyspnea, fever, pulmonary fibrosis, ground glass opacities, bird droppings, mold, restrictive lung pattern

Silicosis

Cough, dyspnea, inspiratory crackles, egg shell calcifications in hilar adenopathy, nodular interstitial markings

Sarcoidosis

Cough, erythema nodosum, hypercalcemia, non-caseating granulomas, hilar lymphadenopathy, achalasia, uveitis

Paget's disease (osteitis deformans)

Cranial enlargement, hearing loss, headaches, elevated alkaline phosphatase, urinary hydroxyproline, femoral bowing

Rapidly progressive glomerulonephritis

Crescent formation on light microscopy

Structural Landmarks: C6

Cricoid Cartilage, Carotid tubercle, last moveable segment

Chronic hepatitis C

Cryoglobulinemia, glomerulonephritis, B-cell lymphoma, Sjogren's disease, thyroiditis, lichen planus, ITP, arthralgias, porphyria, purpura

Malaria (P. falciparum)

Cyclic fever, myalgias, splenomegaly, anemia, hypotension, tachycardia

Schistosomiasis

Cystitis, recurrent hematuria, ova in urine, Middle East and Africa

Hairy cell leukemia

Cytoplasmic projections, splenomegaly, fibrotic bone marrow, tartrate-resistant acid phosphatase, cladribine

According to Malamed, how many cartridges of 2% lidocaine can be safely administered to a child weighing 40 lb? A. Three cartridges B. One cartridge C. Nine cartridges D. Two cartridges

D, 2% lidocaine contains 36 mg of lidocaine per cartridge. Since 80 mg is the amount of lidocaine that can safely be administered to this child, the number of cartridges that can be administered is 80 mg divided by 36 mg per cartridge, which is roughly two cartridges

A "skirt" feature for a gold onlay preparation _____. A. Has a shoulder gingival margin design B. Is prepared by a diamond held perpendicular to the long axis of the crown C. Is used only for esthetic areas of a tooth D. Increases both retention and resistance forms

D, A skirt is a "mini-crown" preparation around a line angle. It should be prepared by a diamond instrument in the long axis of the tooth crown, extended to the gingival one-third, and result in an appropriate amount of tooth removal. It is placed to increase both retention form (having opposing skirt vertical walls retentive with each other) and resistance form (enveloping the line angles like a barrel hoop around a barrel). It extends the outline form and therefore may be least appropriate for highly esthetic areas in the mouth

The tooth preparation technique for a Class I amalgam on a mandibular first molar does not include which of the following? A. Maintaining a narrow isthmus width B. Initial punch cut placed in the most carious pit C. Establishment of pulpal depth of 1.5 to 2 mm D. Orientation of bur parallel to the long axis of the tooth

D, A tooth preparation for a mandibular molar should have a narrow isthmus, should be initiated in the most carious (or distal) pit, and should establish the initial pulpal floor depth of 1.5 to 2 mm. However, it should be oriented parallel to the long axis of the crown, which tilts to the lingual. If prepared in the long axis of the tooth, there is greater potential of weakening the lingual cusps.

Diffuse soft swelling of the lips and neck following the ingestion of drugs, shellfish, or nuts is known as _____. A. Fixed drug reaction B. Anaphylaxis C. Urticaria D. Acquired angioedema E. Contact allergy

D, Acquired angioedema is a rapidly developing allergic reaction that results in characteristic nonerythematous swelling of lips, face, and neck

Which of the following chemical agents is not a disinfectant? A. Iodophors B. Sodium hypochlorite C. Synthetic phenol D. Isopropyl alcohol E. Glutaraldehyde

D, Alcohol is not an accepted disinfectant. Alcohol evaporates too quickly to be an effective disinfectant. The term disinfection is reserved for chemicals applied to inanimate surfaces, and the word antiseptic is used for antimicrobial agents (such as alcohol) that are applied to living tissues

Which two drugs have mechanisms of analgesic action that are most similar? A. Fentanyl, ibuprofen B. Aspirin, codeine C. Oxycodone, acetaminophen D. Ibuprofen, naproxen E. Aspirin, ibuprofen

D, All of the choices are combinations of an opioid and an inhibitor of cyclo-oxygenase (COX), except two: ibuprofen, naproxen and aspirin, ibuprofen. Ibuprofen and naproxen are both reversible inhibitors of COX, and are propionic acid derivatives. Aspirin is a salicylate and is an irreversible inhibitor.

A cervical lesion should be restored if it _____. A. Is carious B. Is very sensitive C. Is causing gingival inflammation D. All of the above

D, All of these factors indicate a cervical lesion should be restored. In addition, if the lesion is large and the pulpal or gingival tissues are in jeopardy, it should be considered for restoration.

A good preventive and treatment strategy for dental caries would include _____. A. Limiting cariogenic substrate B. Controlling cariogenic flora C. Elevating host resistance D. All of the above

D, Altering the organism, its nutrients, and its environment will all enhance prevention and treatment objectives.

Which of the following is the primary etiologic factor associated with periodontal disease? A. Age B. Gender C. Nutrition D. Bacterial plaque

D, Although age, gender, and nutrition may have an impact on periodontal disease, the accumulation of the bacterial plaque biofilm is the primary initiator of the disease

Defects in which inflammatory cell have most frequently been associated with periodontal disease? A. The T-lymphocyte B. The mast cell C. The plasma cell D. The neutrophil

D, Although defects in any of the host defense cells could impact periodontal disease susceptibility, defects in neutrophils have been most frequently described.

Which of the following is a potential sequela of an acute periapical abscess? A. Central giant cell granuloma B. Peripheral giant cell granuloma C. Osteosarcoma D. Periapical granuloma E. Periapical cemento-osseous dysplasia

D, An acute exudate (pus) at the apex of a tooth will follow the path of least resistance (e.g., into surrounding bone, gingiva, or skin). If the offending tooth is not treated and the abscess becomes chronic, a periapical granuloma may result

Before the exploration of any intrabony pathologic lesion, which type of biopsy must always be done? A. Cytologic smear B. Incisional biopsy C. Excisional biopsy D. Aspiration biopsy

D, Any radiolucent lesion that requires biopsy should undergo aspiration before surgical exploration. This procedure may yield material for biopsy, and will rule out a vascular lesion (e.g., AV malformation), which could be dangerous to enter without prior diagnosis

Which of the following local anesthetics has the shortest half-life? A. Lidocaine B. Prilocaine C. Bupivacaine D. Articaine

D, Articaine has an ester bond and an amide bond. Since esters are biotransformed much more rapidly than amides, articaine has a much shorter halflife than the others

Choose the incorrect statement about Class V amalgam restorations. A. The outline form is usually kidney- or crescentshaped. B. Because the mesial, distal, gingival, and incisal walls of the tooth preparation are perpendicular to the external tooth surface, they usually diverge facially. C. Using four corner coves instead of two fulllength grooves conserves dentin near the pulp and may reduce the possibility of a mechanical pulp exposure. D. If the outline form approaches an existing proximal restoration, it is better to leave a thin section of tooth structure between the two restorations (< 1 mm) than to join the restorations

D, Because of the typical shape of a carious lesion in the cervical area, the resulting restoration is kidney- or crescent-shaped and the extensions are to the line angles, resulting in the mesial and distal walls diverging externally. The convexity of the tooth in the gingival one-third results in the occlusal and gingival walls diverging externally. There are several retention groove designs that are appropriate, including four corner coves, occlusal and gingival line angle grooves, or circumferential grooves. However, as with any restoration, if there is only a small amount of tooth structure (< 1 mm) between the new and existing restoration, it is best to join the two restorations together and prevent the possibility of fracture of the small amount of remaining tooth structure.

A 22-year-old college student presents with oral pain, erythematous gingival tissues with blunt papillae covered with a pseudomembrane, spontaneous gingival bleeding, and halitosis. There is no evidence of clinical attachment loss. What form of periodontal disease does this patient most likely have? A. Gingivitis associated with dental plaque B. Localized aggressive periodontitis C. Generalized chronic periodontitis D. Necrotizing ulcerative gingivitis

D, Because there is no loss of attachment, the diagnosis would not be periodontitis. The clinical description of pain, erythema, blunt papillae, pseudomembrane, and halitosis is consistent with necrotizing ulcerative gingivitis

Which drug lacks the amine group that other anesthetics have and is used only topically? A. Procaine B. Mepivacaine C. Lidocaine D. Benzocaine E. Prilocaine

D, Benzocaine lacks the amine group that procaine, mepivacaine, prilocaine, and lidocaine have. This amine group can become protonated, thus making these drugs more water-soluble and facilitating an injectible form. Benzocaine must be provided in a cream or oil-based preparation allowing just a topical form. Procaine and mepivacaine have poor topical anesthetic properties

The principal goals of bonding are _____. A. Sealing and thermal insulation B. Strengthening teeth and esthetics C. Esthetics and reduction of postoperative sensitivity D. Sealing and retention E. Retention and reduction of tooth flexure

D, Bonding is primarily for sealing the dentin and enhancing the retention of the restorative material in the preparation. Esthetic benefits are a welcome side benefit when using a composite restoration. Thermal insulation is provided by the use of composite as compared to amalgam but is not a benefit of the bonding. Bonding will not alter tooth flexure under normal load but may better help bond the unprepared tooth structure together

During the healing of a surgically treated intrabony (infrabony) pocket, regeneration of a new periodontal ligament, cementum, and alveolar bone will only occur when cells repopulate the wound from which of the following sources? A. Gingival epithelium B. Connective tissue C. Alveolar bone D. Periodontal ligament

D, Cells from the periodontal ligament are proposed to allow for regeneration of the periodontal tissues

The preferred surgical procedure to correct a Class II malocclusion due to a deficient mandible is which of the following? A. Maxillary impaction B. Maxillary setback C. Mandibular setback D. Mandibular advancement

D, Class II correction by surgery requires moving the mandible forward or the maxilla back. In a patient with a deficient mandible it is preferable to move the mandible forward. Moving the maxilla back significantly is difficult or impossible

Class II elastics are used by stretching an elastic between which of the two following points? A. From the posterior to the anterior within the maxillary arch B. From the posterior to the anterior within the mandibular arch C. From the posterior of the maxillary arch to the anterior of the mandibular arch D. From the posterior of the mandibular arch to the anterior of the maxillary arch

D, Class II elastics work in the direction that would be used to correct a Class II malocclusion, to pull the mandibular teeth forward and the maxillary teeth distally.

Which of the following scenarios is an example of classical conditioning? A. You teach a dentally-anxious patient diaphragmatic breathing unconditional stimulus (US), which naturally induces the physiological relaxation response unconditional response (UR). You seat that anxious patient in the dental chair for an examination conditional stimulus (CS) and ask them to use their breathing skills during the exam (US). While using the breathing skills, the patient will feel more relaxed conditional response (CR). B. You teach a dentally-anxious patient diaphragmatic breathing (US), which naturally induces the physiological relaxation response (UR). You ask the patient to practice that technique at home (CS) and also use it during procedures to reduce the subjective experience of anxiety (CR). C. You teach a dentally-anxious patient diaphragmatic breathing (US), which naturally induces the physiological relaxation response (UR). You seat that anxious patient in the dental chair for an examination (CS) and ask them to use their breathing skills during the exam (US). The focus on breathing serves as a distraction (US) from what the patient feels is threatening and fearful (CR), and, therefore reports less anxiety (CR). D. You teach a dentally-anxious patient diaphragmatic breathing (US), which naturally induces the physiological relaxation response (UR). You seat that anxious patient in the dental chair for an examination (CS) and ask them to use their breathing skills during the exam (US). After a number of these experiences, the patient will feel relaxed during the exam while using the breathing technique (UR) and without using it at all (CR). E. None of the above.

D, Classical conditioning (also known as respondent or Pavlovian conditioning) occurs when a neutral stimulus, one that is not associated with a particular response, is paired with an unconditioned stimulus ([US]; one that naturally elicits a particular response [UR]). After a number of pairings, the neutral stimulus (CS) elicits a conditioned response (CR), which is essentially a weaker form of the UR) without the presence of the US

Cone-cutting results from _____. A. Too great a target-film distance B. Not selecting the proper kVp C. Not enough time exposure D. The x-ray machine being improperly aimed

D, Cone-cutting results from misalignment of the xray tube. Use a film-holding device with an external guide.

You are performing a 5-year follow-up on a 43-year-old implant patient. When comparing radiographs, you estimate that there has been almost 0.1 mm of lost bone height around the implant since it was placed. Which of the following is indicated? A. Removal of the implant and replacement with a larger-size implant. B. Removal of the implant to allow healing before another one can be placed 4 months later. C. Remaking the prosthetic crown because of tangential forces on the implant. D. The implant is doing well; this amount of bone loss is considered acceptable

D, Criteria for implant success include mean vertical bone loss of less than 0.02 mm annually after the first year of service. In this question, no further treatment is necessary at this time

The idiopathic condition in which destructive inflammatory lesions featuring necrotizing vasculitis are seen in the lung, kidney, and upper respiratory tract is known as _____. A. Epidermolysis bullosa B. Stevens-Johnson syndrome C. Sturge-Weber syndrome D. Wegener's granulomatosis E. Secondary syphilis

D, Destructive inflammation in the three sites noted is characteristic of Wegener's granulomatosis

Which one of the following drugs enters the target cell and acts on a nuclear receptor? A. Diazepam B. Epinephrine C. Insulin D. Prednisone E. Heparin

D, Diazepam, epinephrine, and insulin act at ion channel receptors, G-protein-linked receptors, and tyrosine kinase-linked receptors, respectively. These three receptor types are cell surface receptors. Thyroid hormone and steroid hormones or drugs, such as prednisone, act on nuclear receptors, accounting for much of their action. Heparin's action is to stimulate antithrombin III in the plasma. Its action is extracellular

Which of the following materials has the highest linear coefficient of expansion? A. Amalgam B. Direct gold C. Tooth structure D. Composite resin

D, Direct gold and tooth structure have similar linear coefficients of expansion. Amalgram exhibits twice that expansion whereas composite expansion would be even greater (2.5 times greater than tooth structure)

Which of the following terms refers specifically to the process where an antimicrobial agent destroys (germicide) or avoids the growth (microbiostatic) of pathogenic microorganisms on inanimate surfaces? A. Antisepsis B. Microbacterial control C. Sterilization D. Disinfection E. Asepsis

D, Disinfection refers only to the inhibition or destruction of pathogens. Spores are not killed during disinfection procedures. By custom, the term disinfection is reserved for chemicals applied to inanimate surfaces, and the word antiseptic is used for antimicrobial agents that are applied to living tissues

Why are rounded internal line angles desirable in the preparation of amalgam restorations in primary teeth? A. They increase retention B. They conserve tooth structure C. They increase resistance D. They decreases internal stresses in the restorative material

D, Due to the small size of primary molars and, therefore, small restorations as well, it is helpful to reduce stresses within the restorative material. It has been demonstrated that rounded internal line angles aid in reducing stress when compared to sharp internal line angles. Many of the burs recommended for use in primary molars have a rounded end to help achieve softened internal line angles

In a patient with missing permanent maxillary lateral incisors, the decision of whether to substitute canines in the lateral spaces depends on all of the following except _____. A. The amount of crowding in the maxillary arch B. The interarch relationship between the maxillary and mandibular dentition C. The esthetic appearance of the permanent canines D. The type of orthodontic appliance used to align the teeth

D, Excessive crowding may influence the decision in favor of canine substitution. However, esthetic concerns may deter a decision to substitute canines for lateral incisors. Patients with a Class II interarch relationship requiring maxillary extractions anyway may be better served to substitute canines for laterals rather than extracting healthy first premolars

What type of epidemiology is primarily used in intervention studies? A. Descriptive B. Analytical C. Observational D. Experimental E. None of the above

D, Experimental epidemiology is used primarily in intervention studies. Once etiology for a particular disease has been determined, the researchers will try to establish the effectiveness of a particular program of prevention or therapy. Descriptive epidemiology is used to quantify disease status in a community. Analytical epidemiology, also called observational epidemiology, is used to determine the etiology of a disease

It is generally desirable that x-ray films be all of the following except _____. A. High speed B. Fine grain size C. Coated with emulsion on both sides D. Sensitive to visible light

D, Film is sensitive to visible light but this is not a desired characteristic like the other choices

Although many plaque bacteria coaggregate, which of the following bacteria is believed to be an important bridge between "early colonizers" and "late colonizers" as plaque matures and becomes more microbiologically complex? A. Porphyromonas gingivalis B. Streptococcus gordonii C. Hemophilus parainfluenzae D. Fusobacterium nucleatum

D, Fusobacterium nucleatum can be found in health and disease. This bacterium is an important bridge between early and late colonizers of the dental plaque biofilm

Your patient is 8 years old. Teeth #8 and #9 have approximately 50% of their crowns erupted. One month ago, the patient fell from a skateboard and hit teeth #8 and #9 on the sidewalk. The radiograph today shows open apices of these teeth, normal PDL, and no apparent periapical radiolucency. The patient has no reaction to electrical pulp tests. What is your treatment of choice? A. Calcium hydroxide pulpotomy B. Formocresol apexification technique C. Calcium hydroxide apexification technique D. Reappoint for exam and radiographs in 6 weeks

D, If a tooth is incompletely erupted or is being orthodontically treated, the tooth may be normal even if there is little sensitivity to electrical pulp tests. Certainly, in the absence of other symptoms, treatment is contraindicated.

A patient with the maxillary first permanent molar mesiobuccal cusp sitting distal to the buccal groove of the mandibular first molar has which type of malocclusion? A. Class I B. Class II, division 1 C. Class II, division 2 D. Class III

D, If the mandibular molar buccal groove is mesial to the mesiobuccal cusp of the maxillary molar, the relationship is described as Angle Class III

Why are implants not generally performed on a 12-year-old patient with congenitally missing lateral incisors? A. The patient would likely not be able to tolerate the surgical procedure. B. Waiting for the crowns is too much of an esthetic issue with most children that age. C. The gingival tissue will recede as the child gets older. D. The implants will appear to submerge as the child gets older.

D, Implants are osseointegrated and therefore behave as ankylosed teeth. As teeth erupt and alveolar bond formation occurs, an osseointegrated implant will appear to submerge

The primary purpose of a maxillary denture occlusal index is to _____. A. Maintain the patient's vertical dimension B. Maintain both the correct centric and vertical relation records C. Maintain the patient's centric relation D. Preserve the facebow record

D, In order to preserve the mounting relationship in the articulator of the maxillary cast (facebow record) after processing a denture, an occlusal index of the maxillary denture is made after occlusal adjustments, and before decasting the denture. This procedure has nothing to do with the mandible's relationship to the maxilla.

A researcher follows a group of individuals in a population over 10 years to determine who develops cancer, and then evaluates the factors that affected the group. What type of study is this? A. Cross-sectional B. Case control C. Randomized D. Prospective cohort E. Retrospective cohort

D, In this case, the investigator chooses or defines a sample of subjects who do not yet have the outcome of interest: cancer. He or she measures risk factors in each subject (such as habits that may predict the subsequent outcome) and follows these subjects with periodic surveys or examinations to detect the outcome(s) of interest

Which of the following local anesthetic techniques is recommended for anesthetizing a primary mandibular second molar which will be extracted? A. Buccal and lingual infiltration adjacent to the second primary molar B. Inferior alveolar nerve block C. Inferior alveolar nerve block and lingual nerve block D. Inferior alveolar, lingual, and buccal nerve block

D, Inferior alveolar, lingual, and buccal nerve blocks are required to adequately anesthetize this area when performing deep restorations, pulp therapy, and extractions. Some studies have shown that local infiltration anesthesia for primary molars is effective, but this is primarily reserved for restorative procedures because there is an increased probability for anesthesia failure using local infiltration for pulp therapy and extraction procedures.

Of the following, which is considered to be the least stable orthognathic surgical movement? A. Advancement of the mandible B. Advancement of the maxilla C. Superior movement (impaction) of the maxilla D. Inferior movement of the maxilla

D, Inferior movement of the maxilla, especially without bone grafting and rigid fixation, has been shown to relapse over time because of vertical occlusal forces generated by the masticatory musculature

Which of the following is not a classification of mandible fractures? A. Anatomic location B. Description of the condition of the bone fragments at the fracture site C. Angulation of the fracture and muscle pull D. LeFort level

D, LeFort level fractures are associated with maxillary injuries. Mandibular fractures are classified according to anatomic location, condition of the bone and soft tissue, and the muscle pull on the segments

Fluorosis is the result of excessive systemic fluoride during which stage of tooth development? A. Initiation B. Morphodifferentiation C. Apposition D. Calcification

D, Localized infection, trauma, and excessive systemic fluoride ingestion may cause hypocalcification. Disturbances in apposition result in incomplete tissue formation. For example, an intrusive injury to a primary incisor may disrupt enamel apposition and result in an area of enamel hypoplasia

How many hours after brushing does it usually take for a mature dental plaque to reform? A. 1-2 B. 5-10 C. 12-24 D. 24-48

D, Mature dental plaque usually reforms on the teeth within 24 to 48 hours after effective plaque removal

What tooth surfaces should be evaluated for furcation involvement on maxillary molars? A. Palatal, facial, and distal B. Mesial, distal, and palatal C. Facial, palatal, and mesial D. Facial, mesial, and distal

D, Maxillary molars usually have three roots (mesiobuccal, disto-buccal, and palatal). Furcation involvement can be assessed on these teeth from the facial (bifurcation between the mesio-buccal and disto-buccal roots), mesial (bifurcation between the mesio-buccal and palatal roots) and distal (bifurcation between the disto-buccal and palatal roots).

What is the minimum alveolar concentration of nitrous oxide (Vol %)? A. 50 B. 75 C. 95 D. 105

D, Minimum alveolar concentration is a measure of potency. It is the concentration required to produce immobility in 50 Vol % of patients responding to surgical incision. A minimum alveolar concentration of 105 Vol % indicates that nitrous oxide alone does not produce profound surgical anesthesia at a normal atmospheric pressure

The "willful failure of parent or guardian to seek and follow-through with treatment necessary to ensure a level of oral health essential for adequate function and freedom from pain and infection" is a definition of _____. A. Munchausen syndrome by proxy B. Emotional abuse C. Parental corruption D. Neglect

D, Munchausen syndrome by proxy is a condition in which a person, usually a parent, presents factitious symptoms and illnesses in a child, which may result in extensive testing and/or hospitalizations. Examples of emotional abuse include denial of affection, isolation, extreme threats, and corruption. A parent who knowingly and willingly does not seek care for a child who has pain, infection, or inadequate function is guilty of neglect

Which of the following cells produce antibodies? A. Neutrophils B. T-lymphocytes C. Macrophages D. Plasma cells

D, Neutrophils are one of the primary defense cells of the innate immune system. T-lymphocytes are important activators of the adaptive immune system. Macrophages are antigen-presenting cells. Plasma cells produce antibodies

The long-term histopathologic consequences to an irradiated organ depend on _____. A. The presence of oxygen at the time of irradiation B. The sensitivity of the parenchymal component C. The damage to the stromal component D. All of the above E. None of the above

D, Numbers 1, 2, and 3 are correct

A casting may fail to seat on the prepared tooth due to all of the following factors except _____. A. Temporary cement still on the prepared tooth after the temporary restoration has been removed. B. Proximal contact(s) of casting are too heavy or too tight. C. Undercuts present in prepared tooth. D. The occlusal of the prepared tooth was underreduced

D, Occlusal reduction would not affect the ability to seat a casting. However, temporary cement, heavy proximal contacts, or tooth undercuts could keep the casting from seating completely

Methicillin-resistant Staphylococci are most likely to be inhibited by which drug? A. Amoxicillin B. Clarithromycin C. Clindamycin D. Vancomycin E. Penicillin V

D, Of the choices given, only vancomycin is effective against many methicillin-resistant Staphylococci. Various penicillins, macrolides, and clindamycin are ineffective

The radiosensitivity of cells depends upon _____. A. Mitotic future B. Mitotic activity C. Degree of differentiation D. All of the above E. None of the above

D, Options A, B, and C are correct.

Oral contraceptives can cause gingivitis. Oral contraceptives can accentuate the gingival response to bacterial plaque. A. Both statements are true. B. Both statements are false. C. The first statement is true, the second statement is false. D. The first statement is false, the second statement is true

D, Oral contraceptives can exacerbate the impact of bacterial plaque on the gingival tissues. However, they cannot cause gingivitis

What is the active ingredient in PerioChip™? A. Doxycycline B. Tetracycline C. Metronidazole D. Chlorhexidine

D, PerioChip® is a biodegradable local delivery agent for chlorhexidine.

The supplemental fluoride daily dosage schedule for a 5-year-old child who lives in a community where the concentration of fluoride in the drinking water is less than 0.3 ppm is _____. A. 0 mg B. 0.10 mg C. 0.25 mg D. 0.50 mg E. 1 mg

D, Physicians and dentists can help prevent fluorosis by prescribing dietary fluoride supplements according to the Supplemental Fluoride Dosage Schedule recommended by the ADA Council on Scientific Affairs

Radiographs of the pregnant patient _____. A. Should never be made B. Cause fetal injury C. Should only be made with triple leaded aprons on the mother's lap D. Should be made when there is a specific need

D, Prudence suggests that radiographic examinations of a pregnant patient should be kept to a minimum consistent with the mother's dental needs

According to anxiety disorders research, it has been suggested that which of the following is the most important component of systematic desensitization? A. Cognitive restructuring B. Progressive muscle relaxation C. Diaphragmatic breathing D. Exposure E. Psychoeducation

D, Research suggests that the most integral component of the treatment of anxiety is exposure to the feared stimulus

The most common pin used in restorative procedures is a(an) _____. A. Friction-locked pin B. Cemented pin C. Amalgampin D. Self-threaded pin

D, Self-threaded pins are used by most operators, when pin use is indicated

Smooth surface caries refers to _____. A. Facial and lingual surfaces B. Occlusal pits and grooves C. Mesial and distal surfaces D. Both A and C.

D, Smooth surface caries occurs on any of the axial (facial, lingual, mesial, and distal) tooth surfaces but not the occlusal

Which of the following drugs blocks the aldosterone receptor? A. Amiloride B. Triamterene C. Losartan D. Spironolactone E. Furosemide

D, Spironolactone, a potassium-sparing diuretic useful in treating edema and heart failure, is a competitive antagonist at the aldosterone receptor.

The substitution of a relaxation response for an anxiety response (using a relaxation strategy such as diaphragmatic breathing) when one is exposed to a hierarchy of feared stimuli is called _____. A. Progressive muscle relaxation B. Habituation C. Flooding D. Systematic desensitization E. Biofeedback

D, Systematic desensitization is the process of systematically pairing a relaxation response with a hierarchy of feared stimuli

A 32-year-old male patient is fearful of receiving injections. You decide to use a cognitive behavioral strategy with him to help him through an injection. You have already instructed him in diaphragmatic breathing and ask him to practice this skill throughout the procedure. First, you show him the syringe. You talk about the characteristics of the needle. You then place the needle in his mouth with the cap on. Then, you simulate the procedure with the cap on. You then simulate the procedure with the cap off. Eventually, you proceed with the injection. What does this procedure exemplify? A. Habituation B. Cognitive control C. Flooding D. Systematic desensitization E. Behavior modification

D, Systematic desensitization is the systematic process of exposing the patient to a hierarchy of increasingly anxiety-provoking stimuli while the patient uses relaxation skills such as diaphragmatic breathing exercises.

The purpose of the "penny test" is to check _____. A. Developer action B. Fixer action C. For proper development temperature D. For proper safelighting conditions

D, The "penny test" is a test of darkroom safelighting. A penny is placed on an exposed film (after removing the film from its cover) for 2 minutes and then the film is processed. If the processed film shows a lighter area on the film corresponding to the penny, then the safelighting is too bright and is fogging the film.

Which of the following statements about transmissible diseases is false? A. The risk of transmission after percutaneous injury is higher for HBV than for HIV. B. HCV and HIV are both caused by an RNA virus. C. A vaccine to immunize against HBV is available. D. The average risk of infection for HBV after a needlestick injury falls between HCV and HIV. E. All of the above

D, The average risk of infection for HBV after a needlestick injury does not fall between HCV and HIV. For HBV, the risk of transmission after percutaneous injury is 30%; this figure is 1.8% for HCV and 0.3% for HIV

Well-controlled diabetics have more periodontal disease than nondiabetics. Well-controlled diabetics can generally be treated successfully with conventional periodontal therapy. A. Both statements are true. B. Both statements are false. C. The first statement is true, the second statement is false. D. The first statement is false, the second statement is true

D, The extent and severity of periodontal disease in a patient with well-controlled diabetes is usually no more than the extent and severity of disease in patients without diabetes. Patients with well-controlled diabetes can usually be treated with conventional periodontal therapy.

The purpose of a high-voltage transformer in an x-ray machine is to _____. A. Decrease the tube current B. Increase the wavelength of the x-rays C. Improve timer accuracy D. Increase the potential between the filament and the cathode E. Regulate the rate of release of photons from the anode F. Increase resistance in the filament

D, The high-voltage transformer increases the voltage from the line voltage to the high voltage between the anode and cathode necessary to impart sufficient energy to the electrons to convert some of their energy into photons at the target

What is the major clinical difference between the established lesion of gingivitis and the advanced lesion of periodontitis? A. Gingival color, contour, and consistency B. Bleeding on probing C. Loss of crestal lamina dura D. Attachment and bone loss E. Suppuration

D, The initial, early, and established lesions of gingivitis do not have attachment loss associated with them

Torsades de pointes, or polymorphic ventricular tachycardia, is linked most closely to what characteristic of the electrocardiogram? A. Inverted T wave B. Shorter P-R interval C. Shorter P-P interval D. Longer Q-T interval E. Normal electrocardiogram

D, The long Q-T interval observed as a result of certain drugs or as a hereditary condition makes the patient more susceptible to this condition

Motor adverse effects from phenothiazine antipsychotic drugs are due to drug effects in what region of the brain? A. Chemoreceptor trigger zone B. Cerebrum C. Cerebellum D. Nigro-striatal pathway E. Mesolimbic pathway

D, The nigro-striatal pathway contains dopaminergic neurons—important in muscle control. Many antipsychotic drugs block these, leading to the motor adverse effects

The following teeth are erupted in a 4-year-old patient. What is the space maintenance of choice? A. Band-loop space maintainer. B. Lower lingual holding arch. C. Nance holding arch. D. Distal shoe space maintainer

D, The only possibility within these choices is the distal shoe space maintainer. Some clinicians find that a removable "kiddie" acrylic partial can also be successful. These kiddie partials extend distally to the point where the mesial of the first permanent molar would be. Some advocate placing a 1-mm-deep labial-lingual groove in the cast on the alveolar ridge on the mesial of the first permanent molar. This results in extra acrylic at the tissue-acrylic interface that causes pressure. This may aid in keeping the unerupted first permanent molar in position.

Which of the following is the most common method of sterilization? A. Dry heat B. Ethylene oxide C. Glutaraldehyde at 2% D. Autoclave E. Chemi-clave

D, The proper time and temperature for autoclaving is 250˚ F (121˚ C) for 15 to 20 minutes, which yields 15 pounds pressure of steam, or 270˚ F (134˚ C) for a minimum of 3 minutes, which yields 30 pounds pressure of steam. Moist heat destroys bacteria—denaturation of the highprotein- containing bacteria

The setting reaction of dental amalgam proceeds primarily by _____. A. Dissolution of the entire alloy particle into mercury B. Dissolution of the Cu from the particles into mercury C. Precipitation of Sn-Hg crystals D. Mercury reaction with Ag on or in the alloy particle

D, The trituration process mixes the amalgam components and the reaction results in the alloy particle being coated by mercury and a product formed.

Two equal and opposite forces that are not collinear applied to a tooth are called which of the following? A. The center of resistance B. The center of rotation C. Root movement D. A couple

D, This is the definition of a couple. A couple results in a rotational tendency or pure moment.

An adolescent patient presents to your office with a skeletal and dental Class II malocclusion and a deep bite. Which of the following would be a proper treatment plan for this patient? A. Reverse-pull headgear, extrusion arch, and full fixed appliances B. Reverse-pull headgear, intrusion arch, and full fixed appliances C. Extraction of maxillary first premolars, extrusion arch, and full fixed appliances D. Extraction of maxillary first premolars, intrusion arch, and full fixed appliances

D, This patient, if still growing, may be treated with a growth modification approach using headgear (either cervical or high-pull, not reverse-pull) to correct the Class II malocclusion. Since deep overbite is present, a cervical headgear should be used because this type of headgear will extrude the molars which, in turn, will aid in reducing overbite; however, this was not one of the choices. If the patient is a nongrowing patient, the second approach to treat Class II malocclusion is Class II camouflage, which includes extraction of maxillary first premolars to correct the malocclusion. An intrusion arch along with full fixed appliances should be used to correct the deep bite

In what situation is the postganglionic nerve of the sympathetic system a cholinergic nerve? A. The nerves to the eye B. The nerves to the heart C. Most nerves to blood vessels D. Most nerves to sweat glands E. Most nerves to salivary glands

D, This situation for sweat glands is atypical for the sympathetic nervous system

Which of the following is least likely to be successfully treated with a bone graft procedure? A. One-walled defect B. Two-walled defect C. Three-walled defect D. Class III furcation defect

D, Through-and-through (Class III) furcation defects are least likely to be treated with bone graft procedures

After completing your postoperative instructions for dental implant placement for replacement of tooth #14, your patient asks you how long it will be before she can get her new tooth. Which of the following is most correct to allow complete osseointegration? A. 3 weeks B. 6 weeks C. 3 months D. 6 months

D, Traditionally 6 months has been the recommended period for integration and subsequent loading of posterior maxillary implants. Today, because of technological advancements in specified cases, earlier loading may be possible

If an unwrapped, nonprocessed x-ray film is exposed to normal light for just a second and then processed, it _____. A. May still be used but will be a little dark B. May still be used but will be a little light C. May still be used but will be brown D. Will be completely black E. Will be completely clear

D, Visible light will expose all the silver bromide crystals and the film will be black after processing.

Which of the following statements about indirect pulp caps is false? A. Some leathery caries may be left in the preparation. B. A liner is generally recommended in the excavation. C. The operator should wait at least 6 to 8 weeks before re-entry (if then). D. The prognosis of indirect pulp cap treatment is poorer than that of direct pulp caps

D, When doing an indirect pulp cap, some caries may be left, a liner (probably Ca[OH]2) is usually placed over the excavated area, and the area may be assessed 6 to 8 weeks later. Regardless, the indirect pulp cap prognosis is better than the prognosis for direct pulp caps

The function of the filament is to _____. A. Convert electrons into photons B. Convert photons into electrons C. Release photons D. Release electrons E. None of the above

D, When heated, the filament releases electrons (thermionic emission).

For a gold casting alloy, which of the following is added primarily to act as a scavenger for oxygen during the casting process? A. Copper B. Palladium C. Silver D. Zinc

D, Zinc is added to act as a scavenger for oxygen during the casting process. Copper and palladium increase the hardness and affect the color. Silver has an affect on the color as well.

The administration of which compound will give "epinephrine reversal" (drop in blood pressure from epinephrine) if given prior to administration of epinephrine? A. Atropine B. Guanethidine C. Propranolol D. Phenoxybenzamine E. Tyramine

D, α-Adrenoceptor blockers such as phenoxybenzamine will inhibit the vasoconstrictor effect of epinephrine but not the vasodilator effect of epinephrine. Therefore, the administration of α-blockers will result in epinephrine reversal. Atropine would have little effect since it does not act at adrenergic receptors. Propranolol would only block the vasodilator effect of epinephrine and the effect of epinephrine on the heart. Guanethidine and tyramine act largely at prejunctional sites and don't block adrenergic receptors

Psoriatic arthritis

Dactylitis, joint deformity, morning stiffness, onycholysis, nail pitting, geographic tongue, cardiovascular disease

Membranoproliferative glomerulonephritis

Dark urine, proteinuria, microhematuria, C3+ immunofluorescence, persistent complement activation, tram track on light microscopy

what exercise is used to rehab the knee?

DeLorme's

to raise you contrast what would you do to to your Kvp and MAs?

Decrease Kvp by 15% Increase MAs by 30%

Prostaglandin E2 (PGE2)

Decrease gastric acid Increase gastric mucous Increase blood flow to kidneys

Type 4 renal tubular acidosis

Decreased aldosterone, hyponatremia, hyperkalemia, non-gap metabolic acidosis

Define: Extension Malposition

Decreased interspinous space between the involved segment and the segment below with an increased interspinous space between the involved segment and the segment above.

Chronic myelogenous leukemia

Decreased leukocyte alkaline phosphatase, thrombocytosis, leukocytosis, anemia

Hypomobility

Decreased motion or segmental fixation.

Lymphocytic (Hashimoto's) thyroiditis

Decreased radioactive iodine uptake, pernicious anemia, lymphoma, doughnut sign, pseudocystic pattern on ultrasound, core biopsy

Ulnar nerve syndrome

Decreased sensation over 4th and 5th fingers, medial epicondyle nerve entrapment

what muscles are inhibited in Upper cross?

Deep neck flexors, lower trap, SA

Creutzfeldt-Jakob disease

Dementia, myoclonus, sharp triphasic synchronus discharges

APOM posit?

Dens and atlas (Jefferson's burst fx) Cass = 8x10 FFD= 40 Tilt = 0

Celiac disease

Dermatitis herpetiformis

Pemphigus vulgaris

Desmoglein antibodies, flaccid blisters, oral mucous membranes

Serotonin syndrome

Diaphoresis, tachycardia, dilated pupils, hyperthermia, hypertension, hyperreflexia

Pancreatic insufficiency

Diarrhea, alcohol, diabetes, abdominal pain, low stool elastase, B12 deficiency

Niacin deficiency

Diarrhea, dermatitis, dementia

Pneumocystis jiroveci pneumonia

Diffuse bilateral interstitial infiltrates, dry cough, dyspnea, CD4+ < 200, bronchoalveolar lavage, trimethoprim-sulfamethoxazole, steroids

Fibromyalgia

Diffuse musculoskeletal pain, absence of weakness, normal inflammatory markers, poor sleep

Osler-Weber-Rendu syndrome

Diffuse telangiectasias, epistaxis, AVMs, chronic hypoxemia, polycythemia, hemoptysis

Hypertrophic osteoarthropathy

Digital clubbing, wrist/hand pain, lung cancer

Viral myocarditis

Dilated cardiomyopathy, Coxsackie B

Diphenhydramine toxicity

Dilated pupils, urinary retention, physostigmine

Inclusion body myositis

Distal muscular weakness and atrophy, dysphagia, rimmed vacuoles

OFD and FFD controls what variable of the film

Distortion

Meningioma

Dural tail, bone condensation

Superior vena cava syndrome

Dyspnea, venous congestion, swelling of head and upper extremities, lung cancer, non-Hodgkin lymphoma

In the radiolysis of water, _____. A. Free radicals are formed which are nonreactive B. The presence of dissolved O2 reduces the number of free radicals C. The formation of free radicals is the "direct effect" D. The resultant free radicals may alter biological molecules E. Two of the above F. None of the above

E, "Direct effect" refers to production of free radicals from the ionization of water (C). These free radicals formed in the radiolysis of water are highly reactive and may alter biological molecules (D). The presence of oxygen increases the number of free radicals

A set of precautions designed to prevent transmission of HIV, HBV, and other bloodborne pathogens when providing first aid or health care is known as _____. A. Asepsis B. Infection control C. Sterilization D. Disinfection E. Standard infection control procedures

E, A thorough medical history, physical examination, and laboratory tests will not always detect patients who are carriers of infectious diseases. Therefore, you must assume that all patients are infected with HIV, HBV, or other bloodborne pathogens. Similar infection control procedures must be used for all patients, regardless of their medical history or the type of treatment to be performed

Which of the following statements is true concerning vertical dimension of rest (VDR)? A. VDR = physiologic rest position. B. VDR = position of the mandible when opening and closing muscles are at rest. C. VDR is a postural relationship of the mandible to maxilla. D. VDR = the amount of jaw separation controlled by jaw muscles when they are in a relaxed state. E. All of the above

E, All of the above statements are correct. Vertical dimension of rest (VDR) is a physiologic rest position; it is the position of the mandible when the muscles are in their minimum state of tonicity, which occurs when a patient is relaxed with the trunk upright and the head unsupported. In this position, the interocclusal distance is usually 2 to 4 mm when observed at the first premolar area

Which of the following is an example of a cognitive strategy that may be useful in pain management? A. Address expectations by providing information and addressing any questions and/or concerns. B. Suggest to patients that they learn to identify, evaluate, and eliminate maladaptive thinking. C. Encourage patient efforts to address their anxiety and pain management. D. Suggest to patients that they learn to generate, evaluate, and apply more realistic thinking. E. All of the above

E, All of the strategies listed may be considered appropriate cognitive interventions in pain management.

Which of the following statement(s) about the hepatitis B vaccination is(are) true? A. HBV vaccine must be offered to all potentially exposed dental workers. B. The HBV vaccine must be free to all potentially exposed dental workers. C. At the time of employment, each person should be asked to provide documentation of previous immunizations. D. Three doses are given to confer immunity. E. All of the above.

E, All of these measures help ensure the safety of dental personnel

Which of the following effects is a typical effect of an antimuscarinic drug? A. Bronchoconstriction B. Lacrimation C. Miosis D. Sweating E. Urinary retention

E, All other choices are typical of muscarinic cholinergic receptor agonists

Which drug has an antibacterial spectrum that is limited to anaerobes? A. Amoxicillin B. Clarithromycin C. Clindamycin D. Gentamicin E. Metronidazole

E, Amoxicillin, clarithromycin, and clindamycin are effective against some anaerobes but their spectrum is not limited to anaerobic bacteria. Aminoglycosides are effective only against aerobes. Metronidazole's action requires a reduced environment. Its antibacterial spectrum is limited to anaerobes. Metronidazole is also effective against many parasites.

The use of H2 histamine receptor blockers is most clinically useful at what cell type? A. Beta cells of the pancreas B. Basophils C. Mast cells D. Juxtaglomerular cells E. Parietal cells

E, Basophils and mast cells release histamine. However, the cell that responds to histamine stimulation at the H2 receptor is the parietal cell of the stomach. Stimulation of this receptor leads to proton release and a decrease in the pH of the stomach lumen. H2 histamine receptor blockers are used to reduce stomach acid.

Which of the following are characteristics of proper documentation in a dental record? A. Specific B. Objective C. Complete D. Timely E. All of the above

E, Being specific helps to avoid misinterpretation of reports. Being objective provides the basis for accuracy in describing events. Being complete provides the basis for a thorough review of the facts when reviewing the report. Being timely ensures the best opportunity to recall all relevant events

Which of the following is not a factor in the appraisal of stress? A. Familiarity—how familiar the situation is; the less familiar, the more stressful it may seem. B. Predictability—how predictable the situation is; the less predictable, the more stressful it may seem. C. Controllability—how controllable the situation seems to be; the less controllable, the more stressful it may seem. D. Imminence—the more imminent the situation is, the more stressful it may seem. E. Positive or negative valence—whether the situation is positive or negative; positive situations (e.g., a wedding) are typically experienced as less stressful than are negative situations (e.g., a divorce).

E, Both positive and negative events or situations are experienced as stress.

What behavior can you typically expect from an anxious patient in the dental chair? A. He or she is more likely to sit still, hands clasped together. B. He or she is more likely to sit casually, legs crossed, reading a magazine. C. He or she is more likely to keep to himself or herself and not speak unless spoken to. D. He or she is more likely to fidget in the chair, moving his or her hands and feet. E. Both A and C.

E, Contrary to their behavior in the waiting room, anxious patients are typically more likely to sit very still, often holding onto the arms of the dental chair, and engage in minimal verbal communication unless encouraged by the clinician.

With no other intervention or instruction, which is most likely to trigger a physiological relaxation response? A. Observing one's own physiological responses (e.g., heart rate, blood pressure) B. Muscle tensing C. Reassurance D. Thought stopping E. Diaphragmatic breathing

E, Diaphragmatic breathing naturally activates the parasympathetic nervous system, producing a relaxation response

Proper radiographic infection control includes all of the following except _____. A. Wearing gloves while making radiographs B. Disinfecting x-ray machine surface C. Covering working surfaces with barriers D. Sterilizing nondisposable instruments E. Sterilizing film packets

E, Film packets need not be sterilized because the goal is to prevent crosscontamination, not ensure that everything that goes into a patient's mouth is sterile

It is acceptable for the operator to hold the film in a patient's mouth _____. A. If the patient is a child B. If the patient or parent grants permission C. If the patient has a handicap D. If no film holder is available E. Never

E, If someone must hold a film and the patient cannot, then it should be a family member or friend of the patient, not an x-ray operator in the dental office.

How do people typically respond to stress? A. Physiologically (fight-or-flight response; i.e., autonomic arousal) B. Cognitively (beliefs of self-efficacy, stress appraisal) C. Behaviorally (e.g., disturbed sleep/appetite, impaired attention, acting out) D. Emotionally (e.g., anxiety, anger, fear) E. All of the above

E, Individuals respond to stress physiologically, behaviorally, cognitively, and emotionally

Which of the following recommendations must be followed when handling mercury? A. Train personnel involved in the handling of mercury B. Work in properly ventilated areas C. Use high-volume evacuation systems when finishing or removing amalgams D. Avoid direct skin contact with the metal E. All of the above

E, Mercury can be absorbed through the skin as well as absorbed by inhalation. Safe handling, resulting in part from proper training, helps reduce the risk of exposure

A 15-year-old patient has a numb lower lip and pain in her right posterior mandible. A radiogram shows uniform thickening of the periodontal membrane space of tooth #30. The tooth shows abnormally increased mobility. Which one of the following should be seriously considered? A. Periapical cyst B. Periapical granuloma C. Traumatic bone cyst D. Ameloblastoma E. Malignancy

E, Numb lip is malignancy of the jaw until proven otherwise. About half of the patients with numb lip have associated malignancies. The other half of the patients have acute bone infections or neurologic problems

A prescription for which of the following drugs requires a valid DEA number on the prescription? A. Amoxicillin B. Carbamazepine C. Dexamethasone D. Diphenhydramine E. Oxycodone

E, Only oxycodone is a scheduled drug, requiring DEA registration on the part of the prescriber

Oral squamous cell carcinomas present typically in which of the following ways? A. Vesicular eruption B. Pigmented patch C. Inflamed pustule D. Submucosal swelling E. Indurated nonhealing ulcer

E, Oral cancers (squamous cell carcinomas) present typically as indurated nonhealing ulcers. They can also present as white patches, red patches, or irregular masses

A generalized red, atrophic tongue would suggest all of the following except _____. A. Vitamin B deficiency B. Pernicious anemia C. Chronic candidiasis D. Iron deficiency anemia E. Peripheral giant cell granuloma

E, Peripheral giant cell granuloma is the exception here. Although it is red, it occurs only in the gingiva. Answers A through D are the differential diagnoses for red atrophic tongue

A 12-year-old patient presents with premature loss of primary teeth. On radiographic exam, a sharply marginated lucency is seen in the area of tooth loss. Biopsy shows a round cell infiltrate with numerous eosinophils. This would suggest which of the following? A. Cherubism B. Gardner's syndrome C. Paget's disease D. Fibrous dysplasia E. Langerhans cell disease

E, Premature tooth loss is seen in several conditions, especially malignancies and Langerhans cell disease because of cellular invasion of the periodontal ligament. Sharply marginated bone lesions are characteristic of Langerhans cell disease (and Paget's disease of the elderly). The eosinophils in a round cell infiltrate suggest Langerhans cell disease (the round cells would be Langerhans cells).

If an exposed radiograph is too dark after proper development, one should _____. A. Place it back in the fixer B. Place it back in the developer C. Decrease development time D. Increase milliamperage E. Decrease exposure time F. Decrease development temperature

E, Reduce exposure time. Do not change development parameters if they are correct

Scaling and root planing are used in which phases of periodontal therapy? 1. Initial (hygienic) 2. Surgical (corrective) 3. Supportive (maintenance) A. 1 only B. 1 and 2 only C. 2 and 3 only D. 1 and 3 only E. 1, 2, and 3

E, Scaling and root planing are used in all phases of periodontal therapy where there has been loss of attachment through periodontitis

Which of the following signs or symptoms suggest a chronic benign process? A. Paresthesia B. Pain C. Vertical tooth mobility D Uniformly widened periodontal membrane space E. Sclerotic bony margins

E, Sclerotic bone margins indicate a long-term, lowgrade process, as it takes a considerable amount of time for bone to become radiodense. The signs and symptoms listed in A through D are associated with malignancies

The following part of a scientific article summarizes the background and focus of the study, the population sampled, and the experimental design, findings, and conclusion. A. Introduction B. Background C. Literature review D. Methods E. Abstract

E, The abstract allows the reader to determine whether the study is of interest. The abstract usually appears at the head of the article and is reproduced in the literature database.

The use of the rubber dam is best indicated for _____. A. Adhesive procedures B. Quadrant dentistry C. Teeth with challenging preparations D. Difficult patients E. All of the above

E, The advantages and benefits of rubber dam usage are reflected in all of the items stated. The rubber dam isolation increases access and visibility

A clinical differential diagnosis of an asymptomatic submucosal lump or nodule in the tongue would include all the following except _____. A. Traumatic fibroma B. Neurofibroma C. Granular cell tumor D. Salivary gland tumor E. Dermoid cyst

E, The dermoid cyst occurs in the midline floor of mouth when above the mylohyoid and geniohyoid muscles, and in the neck when below the mylohyoid and geniohyoid muscles

Where do lesions commonly occur in the primary form of acute herpetic gingivostomatitis? A. Buccal mucosa B. Tonsils, hard and soft palate C. Tongue D. Gingiva E. All of the above

E, The location of lesions of primary herpetic gingivostomatitis is on mucous membrane, including tonsils, hard and soft palates, buccal mucosa, tongue, palate, and gingiva. Children with this disease can be very sick and require close supervision and support. They typically have a very significant fever, can become dehydrated, and the process lasts up to 2 weeks. Treatment may consist of: a. Topical anesthetics such as 0.5% dyclonine hydrochloride and viscous lidocaine b. Coating solutions such as diphenhydramine elixir and kaolin-pectin compound c. Antivirals such as acyclovir d. Analgesics such as acetaminophen and ibuprofen

All of the following characteristically present under the age of 20 except _____. A. Traumatic bone cyst B. Adenomatoid odontogenic tumor C. Ameloblastic fibroma D. Compound odontoma E. Ameloblastoma

E, The mean age for ameloblastoma is 40 years. All other lesions listed occur in children and teenagers

The following were the scores for six dental students in their Restorative Dentistry exam: 56, 64, 68, 46, 82, 86. Therefore, the median is _____. A. 68 B. 64 C. 67 D. 40 E. 66

E, The median is the middle of a distribution: half the scores are above the median and half are below the median. The median is less sensitive to extreme scores than the mean, making it a better measure than the mean for highly skewed distributions. For instance, the median income of a population is usually more informative than the mean income. When there is an even number of numbers, the median is the mean of the two middle numbers. Thus, in this case the median is (64 + 68)/2 = 66.

The recommended level of fluoride for community water supply systems in the United States ranges from _____. A. 0.2-0.5 ppm B. 0.7-1.2 mL C. 1.2-1.5 ppm D. 0.2-0.5 mL E. 0.7-1.2 ppm

E, The recommended level of fluoride for a community water supply in the United States ranges from 0.7 to 1.2 ppm of fluoride, depending on the mean maximum daily air temperature over a 5-year period. Thus, in a warm climate the fluoride level would be lower and in a cold climate it would be higher. In the United States, most communities are fluoridated at approximately 1 ppm, which is equivalent to 1.0 mg of fluoride per liter of water.

Which statement is true about the use of silence as an interviewing technique? A. It permits and encourages patient participation. B. It is a nonverbal technique for showing interest in the patient. C. It is a nonverbal technique for encouraging the patient to speak. D. It is done by silently attending to the patient, while maintaining eye contact. E. All of the above

E, The use of silence can be a useful technique to encourage patient comment following a statement or question posed to the patient.

What causes the foot to diverge toward midline?

EX ilium

What pelvic subluxation presents with the following: decreased width of the ilium, anterior lumbar curve increases, lowers femur head?

EX ilium

Ramsay-Hunt syndrome

Ear pain, external auditory canal vesicles, ipsilateral facial paralysis, deafness, herpes zoster

Diffuse alveolar hemorrhage

Ebola and hantavirus, Goodpasture, GPA, Churg-Strauss, Behcet, antiphospholipid antibody

Hydatid disease

Echinococcus granulosus, cholestatic jaundice, eggshell calcifications in wall

Struma ovarii

Ectopic thyroid tissue, pelvic mass, tremor, tachycardia

Angioedema

Edema of face, extremities, larynx, bowels, colicky abdominal pain, C1 inhibitor deficiency, ACE inhibitor

Vitamin B12 deficiency

Elevated MMA, elevated homocysteine, diminished reflexes, decreased vibratory sensation, weakness, metformin, glossitis

Hypothyroidism

Elevated creatine kinase, myopathy, slowed reflexes, hyperlipidemia, hyponatremia, hoarseness, confusion in elderly

Hepatitis B

Elevated transaminases, urticaria

Cholesterol embolism

Eosinophilia, renal insufficiency, abdominal pain, pancreatitis, livedo reticularis

Mycoplasma pneumonia

Erythema multiforme, bullous myringitis, cold agglutinins, positive direct Coombs

Guttate psoriasis

Erythematous plaques with silvery scale on trunk, strep infection, nail pitting, geographic tongue

Bacillary angiomatosis

Exophytic purple papules, collarette of scale, hemorrhagic hepatic lesions, Bartonella, immunocompromised

Acalculous cholecystitis

Extensive burns, prolonged TPN/fasting (decreased CCK), mechanical ventilation, biliary stasis

Lead poisoning

Extensor weakness, myalgias, constipation, microcytic anemia, interstitial nephritis, basophilic stippling

Structural Landmarks: EOP

External Occiput

Where is the gravity weight line when assessing in a lateral manner?

External auditory meatus, anterior body C7, middle of shoulder, anterior 1/3 of sacral base, middle of the hip joint, posterior to the patella, 1" anterior to lateral malleolus

HSV retinitis

Eye pain, conjunctivitis, keratitis, HIV, retinal necrosis

The mean energy of an x-ray beam is influenced by the _____. A. Kilovoltage B. Milliamperage C. Voltage in the filament circuit D. Quantity of electrons in the tube current E. Amount of filtration F. Two of the above G. None of the above

F, The mean energy (wavelength) of an x-ray beam is influenced by the kilovoltage setting on the machine and the amount of built-in filtration that preferentially absorbs low-energy photons.

what is the normal mode of injury for Radial Head fracture?

FOOSH injury

Porphyria cutanea tarda

Facial hyperpigmentation, hypertrichosis, painless blisters, estrogens, alcohol, hepatitis C

T/F ultrasonic scalers can be used on implants

False! They cannot be used on implants!

T/F; Respect is defined as the physician unerstanding the patient's complaint.

False; Resepect is the physcian accepting the patient as he is w/o judgement

T/F; carotid body hypersensitivity can be determined by many tests including auscultation of the neck region.

False; condition such as arterial dissection, arteriosclerotic plaques, bruits and other pulse sounds are a much better fit for neck auscultation

T/F; the period of silence betweent he first diastolic sound and the secon diastolic sound is called the auscultatory gap.

False; the auscultatory gap is defined as the period of silence from phase 1 to phase ii of korotkoff's ounds

T/F; disuse atrophy progresses more rapid than neurogenic atrophy is a good way to differentiated disuse atrophy from neurogenic atrophy.

False; the best way to differentiate is through peripheral limb patterns, nerve roots, peripheral nerve, nerve disease and tender motor points

Guillain-Barre syndrome

Fasciculations, increased CSF protein, Campylobacter jejuni infection

Pseudotumor cerebri (idiopathic intracranial hypertension)

Female, obesity, papilledema, pulsatile tinnitus, acetazolamide, abducens nerve palsy, headache, isotretinoin use, empty sella sign

L5/S1 spot posit?

Fergusons projection L5-S1 disc space Cass = 8x10 FFD= 40 Tilt = 25 deg down caud

Spontaneous bacterial peritonitis

Fever, altered mental status, abdominal pain, E. coli, Klebsiella, ascites, supplemental albumin

Acute hemolytic transfusion reaction

Fever, chills, flank pain, hemoglobinuria, DIC, acute renal failure, shock, within 1 hour

Bronchiolitis obliterans organizing pneumonia

Fever, cough, failed trial of antibiotics, rheumatoid arthritis, amiodarone use

Babesiosis

Fever, drenching sweats, malaise, tick bite, jaundice, hemoglobinuria, splenectomy

CMV mononucleosis

Fever, malaise, muscle aches, arthralgias, atypical basophilic lymphocytes with vacuolated cytoplasm

Ehrlichiosis

Fever, myalgias, leukopenia, thrombocytopenia, doxycycline, transaminitis

Acute ascending cholangitis

Fever, severe jaundice, right upper quadrant abdominal pain, confusion, hypotension

Structural Landmarks: AXIS

First Palpable Spinous

Blastomycosis

Flu-like symptoms, prostatitis, lytic bone lesions, lobar consolidation, ulcerated or verrucous skin lesions, microabscesses, Midwest

Takayasu arteritis

Flu-like symptoms, pulseless extremities, claudication, hypertension, localized aneurysms

Q fever

Flu-like syndrome, hepatitis, pneumonia, exposure to livestock

CMV retinitis

Fluffy retinal lesions, retinal opacification, hemorrhage, CD4 < 50

Carcinoid syndrome

Flushing, valvular heart disease, diarrhea, niacin deficiency, 5-HIAA

Analgesic abuse nephropathy

Focal glomerulosclerosis

Midbrain hemorrhage

Focal neurological deficits, uncontrolled hypertension

Charcot-Marie-Tooth disease

Foot drop, distal extremity muscle atrophy, loss of sensation

Minimal change disease

Foot process effacement

what exercises is used to increase coordination and uses a wobble board

Frenkel's

PA Caldwell projection positioning?

Frontal sinus Cass = 10x12 FFD= 40 Tilt = 15 deg caudad (feet)

what is the most common predisposing factor for adenocarcinoma of the esophagus

GERD

Peptic stricture

GERD resolution, symmetric narrowing of distal esophagus

what test is used to assess for any presence of a spinal cord lesion

Galant's test

DDx for right shoulder referral pain?

Gall Bladder, C/S, lung diaphragm, TRPs, tumor

Legionella pneumonia

Gastrointestinal symptoms, hyponatremia, transaminitis, pneumonia, neurological symptoms, high fever

Test Plan

General Diagnosis Neuromusculoskeletal Diagnosis Diagnostic Imaging Principles of Chiropractic Chiropractic Practice Associated Clinical Sciences

Pericardial effusion

Globular heart on CXR, low voltage EKG, diffuse PMI, electrical alternans

what is the name for medial epicondylitis?

Golfer's elbow

which law cover you for malpractic suits if you are providing care in good faith to a victim of an accident?

Good Samaritan Law

joint aspiration for crystals

Gout CPPD HADD

Malignant otitis externa

Granulation tissue, diabetes, osteomyelitis, peudomonas, facial nerve palsy

Bladder cancer

Gross painless hematuria

Erysipelas

Group A streptococcus, well-demarcated

what is another name for Cubitus varus and body part does it involve?

Gunstock deformity, elbow

Leydig cell tumor

Gynecomastia, precocious puberty, decreased LH/FSH

Mucosa-associated lymphoid tissue lymphoma

H. pylori infection, antibiotic therapy, diffuse large B cell lymphoma

Nephrotic

HEP hypertension, edema, proteinuria

Candidal esophagitis

HIV patients, dysphagia, oral fluconazole

Cryptosporidiosis

HIV, chronic diarrhea, CD4+ < 180, modified acid fast stain

HSV encephalitis

Hallucinations, fever, temporal lobe lesions, seizures, CSF lymphocytosis and erythrocytosis

Aspergillosis

Halo sign, air crescent lesions

In a posterior pelvic tilt, what muscles are hypertonic/facilitated?

Hamstrings, gluteals, abdominals

In an anterior pelvic tilt, what muscles will be lengthened/inhibited?

Hamstrings, gluteals, abdominals.

Cryptococcal meningitis

Headache, AIDS, amphotericin B, oral flucytosine, encapsulated yeast, elevated CSF opening pressure, lymphocytic CSF, India ink stain

Giant cell (temporal) arteritis

Headache, jaw claudication, vision loss, polymyalgia rheumaica, carotid bruit, fever, prednisone, aortic aneurysm

Glioblastoma multiforme

Headache, papilledema, personality and behavior changes, butterfly appearance with central necrosis, serpiginous border

Neurofibromatosis type 2

Headaches, deafness, acoustic neuromas, autosomal dominant, meningioma, ependymoma

what act was placed into law to protect patients privacy pertaining to personal and private medical information.

Health Insurance Portability and Accountability Act (HIPAA)

DDx for left shoulder referral pain?

Heart/MI, C/S, lung diaphragm, TRPs, tumor

Osteoarthritis

Heberden/Bouchard nodes, squared-off thumbs, crepitus, bony tenderness, brief morning stiffness, osteophytes, subchondral cysts

Post-streptococcal glomerulonephritis

Hematuria, proteinuria, low C3, periorbital swelling

Goodpasture's syndrome

Hemoptysis, proteinuria, red cell casts, alveolar infiltrates, plasmapheresis, anti-GBM antibodies, linear deposits on immunofluorescence

what are the contraindications for UV light therapy?

Hemorrhage and inflammation

Membranous glomerulonephritis

Hepatitis B, proteinuria, renal vein thrombosis, thromboembolism, electron-dense deposits, NSAID use

Cystinuria

Hexagonal crystals, urine cyanide nitroprusside

Avascular necrosis of femoral head

Hip pain, corticosteroid use, alcoholism, hemoglobinopathy, SLE, full range of motion, normal radiograph early

part of case history describing the circumstances surrounding the patient's primary reason for the visit is called what?

History of present illness

Vertebral artery stroke

Horner's syndrome, dysarthria, dysphagia, dysmetria, ataxia, vertigo, decreased pain and temperature sensation

Ichthyosis

Horny plates, lizard skin, extensor surfaces

Structural Landmarks: C3

Hyoid Bone, Carotid Artery

Crigler-Najjar syndrome type 2

Hyperbilirubinemia, phenobarbital

Squamous cell carcinoma of lung

Hypercalcemia, constipation, increased thirst, PTHrP, hilar mass

Hyperparathyroidism

Hypercalcemia, sestamibi scan, osteoporosis, osteitis fibrosis cystica, pseudogout

Addison's disease

Hyperchloremic metabolic acidosis, hyperkalemia, hyponatremia, hyperpigmentation of skin, vitiligo, low blood pressure, Hashimoto's thyroiditis

what are xanthelasmas and what condition are they associated with?

Hypercholesterolemia;

Nephrotic syndrome

Hypercoaguability, athersclerosis, hyperlipidemia

Non-ketotic hyperosmolar syndrome

Hyperglycemia, altered mental status, dehydration, blurry vision

Glucagonoma

Hyperglycemia, necrolytic migratory erythema, weight loss, liver metastasis

Acute myelogenous leukemia

Hypergranular myelocytes, Auer rods, disseminated intravascular coagulation

What type of lordosis is seen in an Anterior Pelvic Tilt?

Hyperlordosis

Delirium tremens

Hypertension, agitation, tachycardia, hallucinations, fever, benzodiazepine

Coarctation of the aorta

Hypertension, epistaxis, brachial femoral delay, LVH, rib notching, Turner syndrome

Primary hyperaldosteronism (Conn's syndrome)

Hypertension, hypernatremia, hypokalemia, metabolic alkalosis, adrenal adenoma, polyuria, polydipsia, muscle cramps, low renin activity

Polycythemia vera

Hypertension, thrombocytosis, peptic ulcers, gouty arthritis, plethoric face, splenomegaly, pruritis, absent urine erythropoeitin

Theophylline toxicity

Hyperthermia, hypotension, cardiac arrhythmias

Agranulocytosis

Hyperthyroid medications (PTU and MMI), pharyngitis, fever

Secondary hyperparathyroidism

Hypocalcemia, hyperphosphatemia, end-stage renal disease

Obesity hypoventilation syndrome

Hypochloremia, cor pulmonale, erythrocytosis, chronic hypercapnea, pulmonary hypertension

Cushing's syndrome

Hypokalemia, easy bruising, proximal muscle weakness, hyperglycemia, osteopenia, acne, cataracts

Bartter's syndrome

Hypokalemia, metabolic alkalosis, polyuria, polydipsia, normotensive, high urine chloride

What type of lordosis is seen in a Posterior Pelvic Tilt?

Hypolordosis

Diabetes insipidus

Hypoosmolar urine, hyperosmolar serum, lithium, hypernatremia, amiloride

Autoimmune polyendocrine syndrome type 1

Hypoparathyroidism, adenocortical failure, gonadal failure, candidal infections

Tinea versicolor

Hypopigmentation, spaghetti and meatballs pattern of hyphae and spores, topical ketoconazole

Cavernous sinus thrombosis

Hypopituitarism, headache, periorbital edema, cranial nerve palsy

Sickle cell disease

Hyposthenuria, Howell-Jolly bodies

Beta-blocker toxicity

Hypotension, bradycadria, atropine, glucagon

Autoimmune polyendocrine syndrome type 2

Hypothyroidism, type 1 diabetes mellitus, adenocortical insufficiency

Which interleukin is most common after perio disease?

IL-1 and IL-6, but IL-4 and IL-10 is healthy

What causes the foot to diverge away from midline?

IN ilium

What pelvic subluxation presents with the following: increased width of the ilium, decreased normal anterior lumbar curve, raises the femur head?

IN ilium

Cervical Oblique posit?

IVF's Cass = 8x10 FFD= 72 Tilt = 15 deg (Ceph/Post; Caud/Anter)

Structural Landmarks: L4

Iliac crest

as you ______________ OFD the image becomes bigger and blurrier.

Incrase

if the image of the xray gets smaller and clearer, while decreasing magnification and distortion what is happening of the FFD?

Increase FFD

Waldenstrom's macroglobulinemia (lymphoplasmacytic lymphoma)

Increased IgM, hyperviscosity, retinal vein engorgement

Define: Flexion Malposition

Increased interspinous space between the involved segment and the segment below with a decreased interspinous space between the involved segment and the segment above.

Leukemoid reaction

Increased leukocyte alkaline phosphatase, severe infection or inflammation

Latanoprost (Xalatan)

Indicated for glaucoma

Follicular small cleaved cell lymphoma

Indolent lymphoma, waxing and waning lymphadenopathy, low grade

Structural Landmarks: T6

Inferior angle of the scapula lying down

Structural Landmarks: T7

Inferior angle of the scapula seated

Erythema of the palate of a patient wearing denture is due to?

Inflammatory Papillary Hyperplasia

Azithromycin

Inhibits protein synthesis

Leprosy

Insensate hypopigmented plaque, muscle atrophy, acid-fast bacilli

Primary reason for replacing an overhang restoration?

Interfere in plaque removal

Printzmetal's angina

Intermittent nocturnal chest pain, female, smoker, diltiazem, nitrates

In the coupled motion of the lumbar spine, which way does the spinous rotate?

Into Concavity

In the coupled motion of the cervical spine, which way does the spinous rotate?

Into Convexity

Caroli disease

Intrahepatic bile duct dilation, hepatomegaly, jaundice, cholangiocarcinoma

Paroxysmal nocturnal hemoglobinuria

Intravascular hemolytic anemia, low haptoglobin, hyperbilirubinemia, elevated LDH, hepatic vein thrombosis, thrombocytopenia

what law explains how the intensity of the xray beam is affected by the changes in the FFD?

Inverse Square Law

Korsakoff's syndrome

Irreversible amnesia, confabulation, apathy

Progressive multifocal leukencephalopathy

JC virus, white matter involvement, non-enhancing demyelinating lesions, weakness, ataxia, hemiparesis, aphasia

what type of exercises uses biofeedback?

Jacobson's

Zollinger-Ellison syndrome

Jejunal and duodenal ulcers, steatorrhea, MEN1

Whipple's disease

Joint pain, PAS-positive macrophages, malabsorptive diarrhea

Pulmonary edema

Kerley B lines, peribronchial cuffing

Proteus mirabilis urinary tract infection

Kidney stone (struvite and apatite) formation, acute pyelonephritis

diverticulitis

LLQ pain no bloody diarrhea

what type of PT is used for ionophoresis?

LVG = Low volt Galvanic

Nasopharyngeal carcinoma (lymphoepithelioma)

Latent EBV infection, chronic serous otitis media, Southern China

In the cervical spine, with patient supine. How should position the neck?

Laterally flex over your contact and rotate head away.

most risky type of question to ask during a case history?

Leading question

Renal cell carcinoma

Left-sided varicocele, flank pain, hematuria, polycythemia/anemia, abdominal mass

In upper cross syndrome, what muscles are hypertonic/facilitated?

Levator scapulae, pectoralis major, upper trapezius, sternocleidomastoid, scalenes, suboccipitals

Which 3 diseases have desquamative gingivitis?

Lichen planus, mucous membrane pemphigoid, and pemphigus vulgaris

Name 3 drug types that causes xerostomia

Lithium (antipsychotic) Ca Channel blocker Anticonvulsants

Hypermobility

Loosened motor unit

Sick euthyroid syndrome

Low T3, normal T4, normal TSH, increased cytokines

Anemia of chronic disease

Low TIBC, normal/increased serum ferritin

Iron deficiency anemia

Low ferritin, high RDW, decreased reticulocyte count

Vertebral degenerative disk disease

Lumbar spinal stenosis, leg pain, neurogenic claudication

Apical Lordotic posit?

Lung apices (pancoast tumor) Cass = 14x17 FFD= 40 Tilt = 0 (lean back)

Graves disease

Lymphocytic infiltrate in extraocular mucles and orbital fat, increased radioactive iodine uptake

Listeria meningitis

Lymphocytic predominance in CSF

Pheochromocytoma

MEN2, primary hyperparathyroidism, hypertension, hyperglycemia, paroxysmal headaches, alpha followed by beta blocker

_________________ is the number of REMS a person can get in a lifetime without getting radiation poisoning

MPD = Maximum Permissible Dose

Pernicious anemia

Macrocytic anemia, vitiligo, atrophic gastritis, gastric cancer, gastroparesis, anti-parietal cell and anti-intrinsic factor antibodies

St John's Wort

Mainly used for depression

What are the absolute contra-indications to adjusting?

Malignancies, tumors, infections, fractures, abdominal aortic aneurysm, recent surgery.

All teeth have 4 buds except?

Mand 2nd premolar and 1st molars (5 buds)

Lateral periodontal cyst is common found where?

Mandibular premolar

Neuroma

Marfanoid habitus

Menetrier disease

Markedly thickened gastric folds, decreased acid secretion, protein-losing enteropathy

Water's Posit?

Maxillary sinus Cass = 8x10 FFD= 40 Tilt = 0

what type of exercises helps with lower back extension exercises used to increase lumbar lordosis and rehab disc

McKenzie's

Multiple endocrine neoplasia type 2

Medullary carcinoma of thyroid, pheochromocytoma, hyperparathyroidism (a), mucosal neuroma, marfanoid (b)

Folate deficiency

Megaloblastic anemia, chronic alcoholism, normal MMA, elevated homocysteine

Factitious diarrhea

Melanosis coli, pale lymph patches

Diabetic nephropathy

Mesangial expansion, basement membrane thickening

Diabetic retinopathy

Microaneurysms, dot hemorrhages, hard exudates, macular edema, cotton wool spots

Beta thalassemia

Microcytic anemia, splenomegaly, bone deformities, low RDW, increased HgbA2

Sideroblastic anemia

Microcytic, hypochromic, high/normal serum iron and ferritin, low TIBC, pyroxidine deficiency, acute myelogenous leukemia

Situational syncope

Micturition, benign prostatic hypertrophy, coughing fits

Bronchogenic cyst

Middle mediastinal mass

which diseases have Premonitory symtpoms?

Migrains, epilepsy, syncope

Gonococcemia

Migratory polyarthralgia, tenosynovitis, painless vesiculopustular skin lesions, ceftriaxone

what are the orthos for lateral epicondyllitis

Mills, Cozens

Hodgkin's lymphoma

Minimal change disease, nephrotic syndrome, large bi-lobed nuclei

Huntington's disease

Mood instability, chorea, caudate nucleus atrophy, lateral ventricle enlargement

Ankylosing spondylitis

Morning back stiffness, joint fusion, enthesitis, vertebral fracture, elevated ESR, negative Rf, increased lung volume, anterior uveitis, HLA-B27

Atrial myxoma

Most common primary cardiac tumor, left sided

Autoimmune hypoparathyroidism

Mucocutaneous candidiasis, polyglandular endocrinopathy type 1

Stevens Johnson syndrome

Mucocutaneous lesion, targetoid, conjunctivitis

Klebsiella (Friedlander's) pneumonia

Mucoid colonies, gram negative bacillus, capsulated, currant jelly sputum, alcoholics

Bronchiectasis

Mucopurulent expectoration, hemoptysis, antibiotic response, linear atelectasis, peribronchial cuffing

Febrile neutropenia

Mucositis, pseudomonas, chemotherapy

Acute tubular necrosis

Muddy brown casts

CMV pneumonitis

Multifocal diffuse patchy infiltrates, bone marrow transplant, gastrointestinal ulcers, arthralgias, myalgias, esophagitis

HSV esophagitis

Multiple, well-circumscribed, volcano-like ulcers, ballooning degeneration, eosinophilic intranuclear inclusions, acyclovir

Neuroleptic malignant syndrome

Muscle rigidity, elevated CK, dopaminergic antagonist, tremor, fever

what causes scleroderma?

Mutations in the HLA gene and exposure to silica, aromatic and chlorinated solvents, ketones, trichlorethylene, welding fumes and white spirits

Diffuse alveolar bacterial pneumonia

Mycoplasma, PCP

what are the indication of US?

NMS conitions, contractures and adhesions, Acute (pulsed), Chronic (cont)

Most common side of of NO?

Nausea and vomiting

Focal segmental glomerular sclerosis

Nephrotic syndrome, African American, obesity, heroin, HIV (collapsing)

What deficiency causes Pellagra inclding the 4 D's (dementia, dermatitis, diarrhea and death)

Niacin (Vit B3

Hemi-neglect syndrome

Non-dominant parietal lobe lesion

Acute respiratory distress syndrome

Normal PCWP, alcoholism, hypoxemia, bilateral alveolar infiltrates, reduced lung compliance

Familial hypocalciuric hypercalcemia

Normal PTH, decreased urine calcium/creatinine ratio

What is the Lovette Positive Classification of Scoliosis?

Normal compensatory deviation

Lung adenocarcinoma

Not smoking-associated, solitary nodule at periphery, pulmonary fibrosis

Diabetic mononeuropathy

Oculomotor nerve ischemia

Gastric adenocarcinoma

Older patients, acanthosis nigricans, pruritic seborrheic keratoses, signet ring cells

What type of exercise is done non-weight bearing, in which the hand or foot is free to move?

Open Chain Exercise

With a lateral disc protrusion, what side will the patient lean to in order to alleviate the pain?

Opposite side of protrusion

Primary open angle glaucoma

Optic disc cupping, tunnel vision

Methanol toxicity

Optic disc hyperemia, anion gap metabolic acidosis, osmolal gap

Open angle glaucoma

Optic disk cupping, asypmtomatic, loss of peripheral vision

Multiple sclerosis

Optic neuritis, periventricular white matter, elevated IgG in CSF, internuclear ophthalmoplegia, bilateral trigeminal neuralgia

Behcet's syndrome

Oral ulcers, uveitis, genital ulcers, erythema nodosum

What are the relative contra-indications to adjusting?

Osteoporosis, Spondylisthesis, Rheumatoid Arthritis, Cardiovascular predisposing factors, congenital anomalies, acute spastic muscle region.

Hyperthyroidism

Osteoporosis, atrial fibrillation, proximal muscle weakness, tremors

In Gonstead the first letter of the listing always begins with what letter?

P

What is the second letter in the National Listing System?

P

Polyarteritis nodosum

P-ANCA, bowel infarction, hepatitis B, aneurysms, livedo reticularis, peripheral neuropathy, hypertension, fibrinoid necrosis

Microscopic polyangiitis

P-ANCA, hematuria, purpura, fever, malaise, abdominal pain

Acute lymphoblastic leukemia

PAS-positive lymphoblasts

Palpatory findings: Spongy edema at posterior superior margin of SI joint. This is seen with what pelvic subluxation?

PI ilium

What pelvic subluxation presents with the following: low femur head, anterior sacrum, longer innominate, larger obturator foramen, and increased lumbar lordosis?

PI ilium

In a scoliosis, on what side should the heel lift be used?

PI ilium side

In the Lovette Positive classification, on what side is the Anterior Inferior Sacrum?

PI ilium side (short leg)

Structural Landmarks: S2

PSIS

Subacute granulomatous (DeQuervain's) thyroiditis

Pain in thyroid, recent URI, elevated ESR

Bacterial endocarditis

Painful fingertips, arthritis, conjunctival hemorrhage, staph aureus, strep viridans and enterococci (subacute), septic emboli, glomerulonephritis

Pancreatic adenocarcinoma

Painless jaundice, conjugated hyperbilirubinemia, migratory thrombophlebitis

Central retinal artery occlusion

Painless loss of monocular vision, cardiac disease, hypercoaguability, vasculitis, retinal whitening

Mixed cryoglobulinemia

Palpable purpura, proteinuria, hematuria, arthragias, hepatosplenomegaly, hypocomplementemia, hepatitis C

Migratory thrombophlebitis (Trousseau's syndrome)

Pancreatic/lung adenocarcinoma, atypical thrombophlebitis sites, upper GI discomfort

Scabies

Papules in skin folds, permethrin cream twice

Primary hyperparathyroidism

Parathyroid adenoma, hypercalcemia, fatigue, weakness, nephrolithiasis

Multiple system atrophy (Shy-Drager syndrome)

Parkinsonianism, autonomic dysfunction, orthostatic hypotension, neurological deficits

Viral arthritis

Parvovirus B19, polyarticular symmetrical arthritis, peripheral joints

Melasma

Patchy hyperpigmentation, normal skin texture, pregnancy, OCP use

________________ is cuase by having to short an FFD, Long OFD, too big of filament (too big of focal spot)

Penumbra; unsharp

Dressler's syndrome

Pericarditis after MI, malaise, fever, NSAIDs

Acute prostatitis

Perineal pain, tender boggy prostate, dysuria, alcohol use

Ventricular aneurysm

Persistent ST elevation, valve regurgitation, dyspnea on exertion

hard cervical collar

Philadelphia collar

_____________ _______________ is a low enery phont that is absorbed by the subject creating a latent imag on the film.

Photo electric effect

Frontotemporal dementia

Pick's disease, disinhibition, executive dysfunction

Organophosphate poisoning

Pinpoint pupils, bradycardia, muscle fasciculations, salivation, acetylcholinesterase inhibition

Coccidiomycosis

Pleuritic chest pain, erythema nodosum, erythema multiforme, arthralgias

what's another name for ballistics

Plyometrics

Histoplasmosis

Pneumonia, hilar adenopathy, palatal ulcers, hepatosplenomegaly, Midwest, itraconazole

Liquid in glass ionizer cement?

Polyacrylic acid Polyalkenoic acid and Fluoroaluminosilicate glass powder

what conditions is hirsutism found in?

Polycystic ovary syndrome (PCOS) Congenital adrenal hyperplasia Cushing's disease Growth hormone excess (acromegaly) Tumors in the ovaries adrenal gland cancer Von Hippel-Lindau disease Insulin resistance Stromal hyperthecosis

Non-seminomatous germ cell tumor

Positive AFP, HCG

CNS lymphoma

Positive EBV PCR in CSF, periventricular solitary weakly ring-enhancing mass, altered mental status, HIV

Seminoma

Positive HCG, negative AFP

Antiphospholipid antibody syndrome

Positive VDRL, thrombosis, indefinite anticoagulation

Autoimmune hemolytic anemia

Positive direct Coombs (antiglobulin), jaundice, splenomegaly, reticulocytosis, hyperbilirubinemia, microspherocytes

Staphylococcus aureus pneumonia

Post-viral infection, nodular infiltrates, cavitary abscesses

The first letter in the Gonstead listing refers to what?

Posterior

The ASIS's move posterior to the pubic symphysis.

Posterior Pelvic Tilt

Infectious mononucleosis

Posterior cervical lymphadenopathy, tonsillar exudates, autoimmune hemolytic anemia, atypical lymphocytosis

In the sub-occipital triangle, where is the Rectus Capitus Posterior Minor located?

Posterior tubercle to Occiput

What does the second letter refer to in the National Listing System?

Posteriority

Tophaceous gout

Postmenopausal women, diuretic use, DIP joints

Molluscum contagiosum

Poxvirus, dome-shaped papules with central umbilication, CD4 < 100

Multiple endocrine neoplasia type 1

Primary hyperparathyroidism, pituitary tumors, insulinoma, VIPoma, gastrinoma, nephrolithiasis

Amyloidosis

Proteinuria, chronic infection, psoriasis, inflammatory bowel disease, hepatomegaly, IV drugs, cardiomyopathy, Congo red, green birefringence

Polymyositis

Proximal muscle weakness, dysphagia, mononuclear infiltrate, malignancy, corticosteroids

Lambert-Eaton myasthenic syndrome

Proximal muscle weakness, increased strength with repeated use, small cell lung cancer, loss of deep tendon reflexes

Primary biliary cirrhosis

Pruritis, hepatosplenomegaly, xanthelesmas, anti-mitochondrial antibodies, hyperpigmentation, women

Aspirin sensitivity syndrome

Pseudo-allergic reaction, nasal polyps, asthma, leukotriene inhibitors

Diphtheria

Pseudomembranous pharyngitis, unilateral nasal discharge, cervical lymphadenopathy, endocarditis, corynebacterium

In a posterior pelvic tilt, what muscles are lengthened/inhibited?

Psoas, quadriceps, erector spinae

In an anterior pelvic tilt, what muscles are hypertonic/facilitated?

Psoas, quadriceps, erector spinae

Myasthenia gravis

Ptosis, diplopia, dysphagia, dysarthria, recovery with rest, thymoma

Tricyclic antidepressant toxicity

QRS prolongation, hyperthermia, seizures, dilated pupils, intestinal ileus, sodium bicarbonate for sodium channels

What are the two options for the first letter in the national listing system?

R or L

What are the two options for the second letter in the Gonstead listing?

R or L

Where is a chisel fracture found?

Radial head fracture from a FOOSH

De Quervain tenosynovitis

Radial wrist tenderness, pain with abduction and extension

what does RAD stand for?

Radiation abosrbed Dose

what does REM stand for?

Radiation equivalent Man

Uric acid stones

Radiolucency, low urine pH, urine alkalinization

Rheumatic fever

Recent strep infection, carditis, polyarthritis, Sydenham chorea, erythema marginatum, subcutaneous nodules, PR prolongation, antistreptolysin O

Essential thrombocytosis

Recurrent bleeding, giant platelets

Still's disease

Recurrent high fevers, salmon-colored maculopapular rash of trunk and extremities, arthritis

Heparin-induced thrombocytopenia

Recurrent thrombosis, elevated aPTT

Acute angle closure glaucoma

Red eye, steamy cornea, fixed dilated pupil, eye pain, nausea, vomiting

Ethylene glycol poisoning

Renal failure, anion gap metabolic acidosis

Thrombotic thrombocytopenic purpura-HUS

Renal failure, headache, confusion, fragmented red blood cells, plasmapheresis

Transfusion-associated acute lung injury

Respiratory distress, fever, pulmonary edema, hypotension, anti-HLA antibodies, cytokine release

what does RICE stand for

Rest, Ice, compression, elevation

Carpal tunnel syndrome

Rheumatoid arthritis, sarcoidosis, amyloidosis, Tinel sign

Pseudogout

Rhomboid calcium pyrophosphate dihydrate crystals, chondrocalcinosis, knee, leukocytosis

In upper cross syndrome, what muscles are lengthened/inhibited?

Rhomboids, lower & middle trapezius, serratus anterior, teres minor, longus coli and capitus

Autosomal dominant polycystic kidney disease

Right flank mass, CVA tenderness, hematuria, nephrolithiasis, hypertension, nocturia

Cerebral toxoplasmosis

Ring-enhancing lesion in basal ganglia, seizures, headaches, CD4+ < 100, trimethoprim-sulfamethoxazole PPX, sulfadiazine+pyrimethamine

Myelodysplastic syndrome

Ringed sideroblasts, macrocytosis, acute myeloid leukemia, splenomegaly

Dorsal Columns

Romberg's or closed eyes or problems in dark or at night

What are the two option for the third letter in the National Listing System?

S or I

What are the two options for the third letter of the Gonstead system?

S or I

Melanoma

S-100 tumor marker

Small cell carcinoma

SIADH, mediastinal adenopathy, Lambert-Eaton syndrome, Cushing's syndrome

With a medial disc protrusion, what side will the patient lean to in order to alleviate the pain?

Same side of protrusion

Cutaneous larva migrans

Sandboxes, pruritic erythematous papules, serpiginous red-brown raised lesions, tropical regions

Actinic keratosis

Sandpaper texture, cutaneous horns, squamous cell carcinoma, 5-fluorouracil

When assessing the gravity weight line in a posterior manner, what is being viewed?

Scapula winging/flaring, scoliosis, foot angle.

Chronic Heartburn and dysphagiaq to liquids and solids are common symptoms associated with what disease

Sceroderma

Disseminated intravascular coagulation

Schistocytes, microangiopathic hemolytic anemia, helmet cell, large platelets

Myelophthisic anemia

Schistocytes, obliteration of bone marrow with tumor, fibrosis, granulomas

Raynaud's phenomenon is associated complication with what condition?

Scleroderma

systemic autoimmune disease, that primarily affects the skin , is characterised by sclerosis that is hardening of the skin. What condition is this?

Scleroderma

Parkinson's disease

Seborrheic dermatitis, pill rolling tremor, cogwheel rigidity, bradykinesia, postural instability

HIV infection

Seborrheic dermatitis, severe psoriasis

Osteomalacia

Secondary hyperparathyroidism, blurring of spine, pseudofractures, vitamin D deficiency

Lateral skull positioning

Sella turcica Cass = 10x12 FFD= 40 Tilt = 0

LeFort II

Separation of maxilla and nasal complex from cranial base, zygomatic orbital rim area, and pterygoid maxillary suture

LeFort I

Separation of the inferior portion of maxilla

Boerhaave's syndrome

Severe vomiting, left sided pleural effusion, widened mediastinum, chest pain, elevated amylase in pleural fluid

VIPoma (vasoactive intestinal peptide)

Severe watery diarrhea, hypokalemia, leg cramps, dehydration, flushing, abdominal pain

Radiation pneumonitis

Sharply demarcated pulmonary infiltrate, leukocytosis, fever, elevated ESR, chest irradiation

Wolf-Parkinson-White syndrome

Short PR interval (< 0.12), delta wave, procainamide

Normal pressure hydrocephalus

Shuffling gait, incontinence, memory loss

IgA deficiency

Sinopulmonary infections, diarrheal illness, blood transfusion allergic reaction

Lacunar stroke

Small vessel hyalinosis, posterior internal capsule, motor deficits

Langerhans cell histiocytosis

Smoking, painful bone swelling

Pulmonary hamartoma

Smooth bordered nodule with popcorn calcification

Chronic lymphocytic leukemia

Smudge cells, thrombocytopenia, splenomegaly, lymphadenopathy, chlorambucil if symptomatic

Giant cell tumor of bone

Soap bubble appearance, muscle aches

Hepatic adenoma

Solitary mass, right lobe of liver, oral contraceptives, abdominal pain, intraabdominal bleeding, malignant transformation

Amyotrophic lateral sclerosis

Spasticity, decreased bulbar reflexes, deep tendon hyperreflexia, fasciculations, weakness, distal extremity wasting

Cirrhosis

Spider angiomata, painful gynecomastia

Structural Landmarks: T3

Spine of Scapula

Diffuse esophageal spasm

Spontaneous chest pain, odynophagia with hot/cold food, relief with nitrates

Cluster headache

Stabbing pain behind eye, ptosis, miosis, nasal congestion, sumatriptan, 100% oxygen, prednisone

Structural Landmarks: T4,5

Sternal angle of Louis

Emphysematous cholecystitis

Strep, E. coli, clostridium, klebsiella, diabetes, RUQ pain, fever

Nocardiosis

Subacute pneumonia, weak acid-fast gram-positive rods, trimethoprim-sulfamethoxazole

Saccular aneurysm

Subarachnoid hemorrhage, anterior circulation on circle of Willis

SIADH

Subarachnoid hemorrhage, hyponatremia, NSAID use, hypotonicity, euvolemia

Small intestine bacterial overgrowth syndrome

Succusion splash, abdominal surgery, hypomotility

Brugada syndrome

Sudden death, T wave elevation down-sloping from R'

Vitreous hemorrhage

Sudden loss of vision, floaters, obscured fundus

Distractive Osteogenesis

Surgical procedure used to reconstruct skeletal deformities and lengthen long bones

Osteotomy

Surgical procedure whereby bone is cut to shorten, lengthen, or change its alignment

Progressive supranuclear palsy

Symmetric Parkinsonism, vertical gaze limitation, axial rigidity, frontal lobe dementia

Chronic demyelinating polyneuropathy

Symmetric proximal and distal weakness, decreased deep tendon reflexes, loss of vibration and position sense

Hepatorenal syndrome

Systemic vasodilation, pre-renal failure, volume overload, octreotide, midodrine

Cysticercosis

Taenia solium

Alpha thalassemia

Target cells, HgbH, low RDW

Basal cell carcinoma

Telangiectasia, bleeding, ulceration, pearly nodule, rolled border

Rosacea

Telangiectasias, papules, pustules, erythema from sun or heat, conjunctivitis

Heat stroke

Temp > 105, hypotension, tachycardia, seizures, ARDS, DIC, hepatic failure, renal failure

Axillary

Temperatures taken where may be 0.5-1.0 degrees lower than oral temperature?

Rectal

Temperatures taken where, may be 0.5-1.0 degrees higher than oral temperature?

what pattern helps diagnose and exlcude all other DDx besides chronic HA?

Temporal pattern

Rubella

Tender suboccipital adenitis, facial maculopapular rash, subsequent truncal and extremity rash, palatal erythema

Bullous pemphigoid

Tense blisters, IgG and C3 deposits, no oral lesions

Dejerine-Roussy syndrome

Thalamic stroke, contralateral hemianesthesia, ballistic movements, athetosis, dysesthesia

Beriberi

Thiamine deficiency, high output heart failure

Amebiasis

Thin wall uniform liver abscess, bloody diarrhea, metronidazole

Eustachian tube blockage

This opening of a tube opens into the nasopharynx becomes swollen and may be occluded, this causes the air within the middle ear cavity to be absorbed creating a vacuum which pulls the tympanic membrane inwards resulting in partial deafness.

achilles rupture

Thompson's or Simmons Test

Structural Landmarks: C4,5

Thyroid Cartilage

pt presents with spots on chest and face and she says that the spots get worse with tanning. Condition

Tinea versicolor

What part of the titanium implant allows osseointegration (allows calcium phosphate precipitation to form bone)?

Titanium oxide layer

Hospital

To who do you refer someone with Acute glaucoma?

Neurologist

To whom do you refer a patient with Adie's pupil?

Medical doctor

To whom do you refer someone who can prescribe someone ophthalmic antibiotics for a hordeolum?

Neurologist

To whom do you refer someone who has Argyll Robertson pupil?

ENT specialist

To whom do you refer someone who has eustachian tube blockage, if chiropractic care is unsuccessful?

Ophthalmologist

To whom do you refer someone who has macular degeneration?

ENT specialist

To whom do you refer someone who has otitis externa?

Ophthalmologist

To whom do you refer someone with Iritis?

Ophthalmologist

To whom do you refer someone with a chalazion?

Dermatologist

To whom do you refer someone with blepharitis?

Ophthalmologist

To whom do you refer someone with chronic glaucoma?

Medical doctor

To whom do you refer someone with conjunctivitis

Opthalmologist

To whom do you refer someone with diabetic retinopathy?

Neurologist

To whom do you refer someone with horner's syndrome?

ENT specialist

To whom do you refer someone with meniere's disease if dietary changes and chiropractic care are unsuccessful?

ENT specialist

To whom do you refer someone with otitis media if chiropractic care is unsuccessful?

Ophthalmologist

To whom do you refer someone with pterygium once it begins to encroach on the cornea?

Dermatologist

To whom would you refer someone with Seborrheic dermatitis if treatment with shampoo with selenium sulfide is not helpful?

Non-alcoholic steatohepatitis

Transaminitis, obesity, diabetes, hypertrglyceridemia, hepatomegaly

Neuropraxia

Transient episode of motor paralysis due to blockage of nerve conduction. Temporary damage to myelin sheath

Enterotoxigenic E. coli

Travel, diarrhea, abdominal cramps, malaise

Doctor contacts the side they are standing on when a patient is supine or prone. T or F?

True

T/F A ditched restoration doesn't need repair

True

T/F; the nemerical difference between systolic and diastolic blood pressure is termed the "pulse Pressure"

True

Constrictive pericarditis

Tuberculosis, viral infection, radiation therapy, sharp x and y descent, pulsus paradoxus

Herpetic whitlow

Tzanck smear, multinucleated giant gells, throbbing hand pain, vesicles

Paget's disease of the breast

Ulcerating eczematous appearance of nipple, large cells with clear halos, adenocarcinoma

Sporotrichosis

Ulcerating papule at inoculation site, gardening, subcutaneous nodules

Primary sclerosing cholangitis

Ulcerative colitis, RUQ pain, beads on a string, p-ANCA

Toxic megacolon

Ulcerative colitis, fever, tachycardia, neutrophilic leukocytosis, volume depletion, anemia, corticosteroids

Structural Landmarks: L3

Umbilicus

____________ is th epicture made by the useful beam?

Umbra

Gilbert syndrome

Unconjugated hyperbilirubinemia, icterus, stressful trigger

Granulomatosis with polyangiitis (Wegener's)

Upper airway granulomatous inflammation, chronic sinusitis, skin ulceration, purpura, C-ANCA, RBC casts

Syringomyelia

Upper extremity areflexia, cape distribution of anesthesia, Chiari malformation

IgA nephropathy

Upper respiratory tract infection, short latent period, hematuria, RBC casts, dysmorphic RBCs

Meigs syndrome

Uterine fibroma, ovarian tumor, ascites, right sided pleural effusion

Retinal detachment

Vascular disease, photopsia

Condyloma acuminata

Verrucous, papilliform, skin-colored lesions, anogenital region, podophyllin

Structural Landmarks: T1

Vertebral Prominens 30%

Structural Landmarks: C7

Vertebral Prominens 70%

Medial medullary syndrome

Vertebral artery, contralateral extremity paralysis and sensory loss, ipsilateral tongue deviation

Phencyclidine toxicity

Vertical nystagmus

Wallenberg syndrome (lateral medullary syndrome)

Vertigo, gaze abnormalities, limb ataxia, Horner's syndrome, posterior inferior cerebellar artery

Meniere's disease

Vertigo, hearing loss, tinnitus, low salt diet

Vestibular neuritis

Vertigo, recent URI, nystagmus

Herpes zoster ophthalmicus

Vesicles on tip of nose, dendriform corneal ulcer, keratitis

Cat scratch disease

Vesicular and papular erythema, regional lymphadenopathy, positive Warthin-Starry stain, Bartonella henselae, azithromycin

Colon cancer

Villous adenoma, sessile adenoma

Kaposi sarcoma

Violaceous lesions, HHV-8

Lewy body dementia

Visual hallucinations, cognitive impairment, Parkinsonianism

night blindness is caused by what type of deficiency?

Vit A

what vitamin is used to treat carpal tunnel syndrome and over peripheral nerve healing?

Vit B6 (pyridoxine)

Hypopituitarism

Vitiligo

Hypertension, Cushing's syndrome, Conn's disease, Renal disease, hyperthyroidism, and pheochromocytoma

What are causes of elevated blood pressure?

what is the first question to ask a patient c/o of chest pain?

What brings on your chest pain?

Causes of Bradycardia

What do Hypothyroidism, athlete's heart syndrome, and raised intracranial pressure, all have in common?

Causes of tachypnea

What do asthma, lung infection, diabetic ketoacidosis, congestive heart failure, emphysema, sarcoidosis, and panic disorder all have in common?

Causes of bradypnea

What do drug reaction and brain stem compression both have in common?

Causes of Tachycardia

What do drug reaction, fever, panic disorder, hyperthyroidism, and severe anemia, all have in common?

Causes of fever

What do drug reaction, infection, heat stroke, and lymphoma, all have in common?

Adie's pupil

What does it mean when one pupil is dilated larger and reacts to light poorly?

Parasympathetic system

What has a defect when a patient has Adie's pupil?

Normal

What is a BMI of 18.5-24.9?

Overweight

What is a BMI of 25-29.9?

Class I Obesity

What is a BMI of 30-34.9?

Class II Obesity

What is a BMI of 35-39.9?

Class III Obesity

What is a BMI of greater than 40?

Underweight

What is a BMI of less than 18.5?

Adie's pupil

What is a benign condition that mainly affects young women?

Chalazion

What is a benign granulomatous lesion in the tarsal gland of the eyelid?

White coat hypertension

What is a cause of elevated blood pressure which may actually be the result of poor technique?

Cavernous sinus thrombosis

What is a clot formation in the cavernous sinus in the brain following spread of infection from the face?

Chronic glaucoma

What is a gradual increase in the intra-ocular pressure due to excessive production of aqueous humor to gradual obstruction of the canal of Schlemm?

Tachypnea

What is a respiration rate of greater than 20 breaths/minute?

Bradypnea

What is a respiration rate of less than 12 breaths/minute?

Acute glaucoma

What is a sudden increase in the intra-ocular pressure often seen with narrow angle between the iris and the cornea which obstructs the flow of aqueous humor into the canal of Schlemm?

Pinguecula

What is a yellow triangular thickening of the bulbar conjunctiva extending from the medial corner of the eye towards the cornea with the apex pointing towards the pupil?

Macular degeneration

What is age-related degeneration of the macula, that is more common in the elderly and in the whites?

Myasthenia gravis

What is an autoimmune disorder caused by antibodies to acetylcholine receptors at the neuromascular joints, middle aged females with thymic disorder?

Hordeolum

What is an infected hair follicle of an eyelash, often staphylococcal in nature?

Conjunctivitis

What is an infection of the conjunctiva, that is most commonly viral in origin, but it could be bacterial?

Chronic glaucoma

What is associated with increased cupping of the optic disc due to raised intra-ocular pressure?

Eustachian Tube Blockage

What is blockage that may follow an upper respiratory tract infection?

Hypertension

What is blood pressure taken 3 times all with readings higher than 140/90?

Shock

What is blood pressure with a reading lower than 90/60, a sign of?

Horner's Syndrome

What is caused by damage to the sympathetic chain or ganglia in the neck, presents with unilateral ptosis, miosis, and anhydrosis?

Otitis media

What is common in children because of the straight eustachian tubes, presents with ear pain and mild hearing loss, there might be a fever with loss of appetite, the tympanic mambrane is swollen and red?

Cocaine, barbiturate, or amphetamine abuse

What is considered the cause of bilateral dilation of the pupils?

Heroin use

What is considered the cause of bilateral pinpoint pupils?

Diabetic Retinopathy

What is damage to the retina seen in poorly controlled diabetes mellitus, characterized by microaneurysms and flame hemorrhages in the retina? Presents with blurred vision.

Tachycardia

What is greater than 100 heart beats per minute considered?

Seborrheic dermatitis

What is inflammation of skin with a high concentration of sebaceous glands?

Blepharitis

What is inflammation of the eyelids due to staphylococcal infection, allergy or may be due to seborrhea?

Iritis

What is inflammation of the iris, that may be associated with SLE, ulcerative colitis, Crohn's disease or sarcoidosis?

Otitis media

What is inflammation of the middle ear, may be viral or bacterial (hemophilus influenzae is most common in children)?

Bradycardia

What is less than 60 heart beats per minute considered?

Seborrheic dermatitis

What is pityrosporum ovale (normal fungal skin organism) may play a role, presents with dandruff, scaly itching eyebrow and eyelids?

Pterygium

What is related to chronic irritation and hot dry climates and may encroach on to the cornea?

Nerve conduction deafness

What is significant when air conduction and bone conduction are both greater in one ear?

Normal hearing

What is significant when air conduction is twice as long as bone conduction used in Rinne test?

Serous Otitis Media

What is significant when bone conduction is greater than air conduction with Rinne test?

Argyll Robertson pupil

What is the cause of one pupil being small and it reacts poorly to light but well to accommodation?

60-100

What is the normal beats per minute for a patient's pulse?

140-90

What is the normal diastolic range for blood pressure?

98.4-99.5

What is the normal range for a patient temperature?

12-20

What is the normal range for inspirations/minute?

90-60

What is the normal systolic range for blood pressure?

Infection

What is the probable cause of a chalazion?

Chronic glaucoma

What is the significance of a cupped optic disc on an eye exam?

Raised intracranial pressure or brain tumor

What is the significance of a finding of bilateral swollen optic discs?

Osteogenesis imperfect

What is the significance of a finding of blue sclera in infant with many fractures?

Myxedema or hypothyroidism

What is the significance of a goiter with brittle dry coarse hair?

Graves disease

What is the significance of a goiter with exophthalmos or tachycardia?

Hyperlipidemia

What is the significance of a gray ring around cornea, also called arcus senilis?

Cataract or neuroblastoma

What is the significance of a loss of red pupil light reflex?

External stye or hordeolum

What is the significance of a painful boil on the margin of the eyelid?

Iritis

What is the significance of a painful eye with circumcorneal redness?

Acute glaucoma

What is the significance of a painful red eye with a tense eyeball?

Conjunctivitis

What is the significance of a painful red eye with purulent discharge?

Internal stye or chalazion

What is the significance of a painless boil on the inside of the eyelid?

Argyll Robertson pupil or tertiary syphilis

What is the significance of a pupil that accommodates but does not react to light?

Adie's pupil

What is the significance of a pupil that is dilated and reacts poorly to light?

Horner's syndrome

What is the significance of a pupil that is small and constricted (miosis)?

Hyperthyroidism

What is the significance of abnormal protrusion of the eyeball or eyeballs, also called exophthalmos?

Hydrocephalus

What is the significance of an infant with a large head in a child with a open fontanelle?

Myasthenia gravis

What is the significance of bilateral ptosis?

Macular degeneration

What is the significance of blurred margins of the macula or pigmentation?

hypertensive retinopathy

What is the significance of copper or silver wiring of retinal arterioles seen on an eye exam?

Diabetic retinopathy

What is the significance of flame hemorrhages and dots, also called microaneurysms?

Sinusitis of foreign body in nose

What is the significance of foul purulent unilateral nasal discharge?

Problem with one of the ears

What is the significance of hearing better in one ear with Weber's test?

Hyperthyroidism

What is the significance of lid lag?

Hypothyroidism or leprosy

What is the significance of loss of outer third of the eyebrow?

Torus palatines

What is the significance of midline bony mass in the hard palate?

Thyroglossal cyst

What is the significance of midline neck mass that moves with the tongue?

Otitis media

What is the significance of mucopurulent discharge from the ear?

Cocaine abuse

What is the significance of nasal septum perforation?

Otitis externa

What is the significance of pain on pulling the ear up and backwards?

Anemia

What is the significance of pale conjunctiva?

Cellulitis or cavernous sinus thrombosis

What is the significance of periorbital edema?

Renal dysfunction

What is the significance of puffy eyelids especially in the morning?

Otitis media

What is the significance of red inflamed tympanic membrane?

Eustachian tube blockage

What is the significance of retracted tympanic membrane?

Seborrheic dermatitis

What is the significance of scales on the eyebrows?

Blepharitis

What is the significance of swollen itching eyelids?

Riboflavin deficiency

What is the significance of tongue being magenta color?

Iron deficiency

What is the significance of tongue being pale?

Vitamin B12 deficiency

What is the significance of tongue being red beef color?

Chronic gout

What is the significance of tophus on helix of the ear?

Pterygium

What is the significance of triangular fleshy growth in medial side of sclera?

Horner's syndrome

What is the significance of unilateral ptosis?

Meniere's disease

What is the significance of vertigo with hearing loss and tinnitus?

Central vertigo

What is the significance of vertigo with unable to walk?

Xanthelasma

What is the significance of yellow plaque in upper medial eyelid?

Jaundice or carotenemia

What is the significance of yellow sclera?

Pinguecula

What is the significance of yellow thickening of the bulbar conjunctiva?

Pterygium

What is triangular thickening of the bulbar conjunctiva extending from the medial corner of the eye towards the cornea with the apex pointing towards the pupil?

Snellen Chart

What is used to assess farsightedness?

Rosenbaum Chart

What is used to assess nearsightedness?

Horner's syndrome

What may be associated with pancoast tumor, carotid artery dissection or brain ischemia?

Argyll Robertson pupil

What maybe a sign of tertiary syphilis or multiple sclerosis?

Low blood pressure

What might be normal for the patient or might be caused by shock, depression, or addison's disease?

Chalazion

What presents as a painless swelling on the inside of the eyelid?

Hordeolum

What presents as a red painful swelling on the margin of the eyelid?

Iritis

What presents with a painful gritty feeling with redness mainly around the iris?

Acute glaucoma

What presents with a painful red eye and a hard eyeball, vision will hazy in the beginning?

Myasthenia gravis

What presents with bilateral ptosis and diplopia. Tensilon test will be positive?

Macular degeneration

What presents with central loss of vision; the macula appears blurred and may be pigmented? Remember that the macula is lateral to the optic disc on fundoscopy?

Eustachian tube blockage

What presents with decreased hearing with a retracted tympanic membrane?

Otitis externa

What presents with ear pain which is worse on traction? The ear canal is often swollen and there may be a scant discharge?

Chronic glaucoma

What presents with gradual loss of vision especially in the periphery causing tunnel vision?

Otitis externa

What presents with inflammation of the external auditory canal, may be bacterial or fungal in origin or may be allergic in nature?

Conjunctivitis

What presents with itching, pain, tearing, and red eyes with discharge?

Cavernous sinus thrombosis

What presents with swollen eyelids and there will be diplopia due to damage to CN III, IV of VI which pass through the cavernous sinus?

Blepharitis

What presents with swollen itching tender eyelids?

Meniere's disease

What syndrome is characterized by recurring attacks of vertigo, tinnitus, and hearing loss of unknown etiology?

VDRL

What will be positive when a patient has Argyll Robertson pupil?

Down and back

When examining a child's ear which direction should the ear be pulled?

Up and back

When examining an adult ear which direction should the ear be pulled?

Hypothyroidism in infants

When examining an infant, what is the significance of an enlarged tongue (macroglossia)?

512 Hz

When performing Weber and Rinne tests, what size tuning fork should be used?

Arteriovenous fistula

Wide pulse pressure, brisk carotid upstroke, systolic flow murmur, tachycardia, flushed extremities

what type of exercis usees abdominal crunches, pelvic tilts and hamstring stretches designed to reduce a hyperlordosis?

William's

lying on stomach worse flexion makes it better or walking up hill

Williams exercises

Neurologist

With whom should you co-manage someone with Myasthenia gravis?

Structural Landmarks: T10

Xiphoid

Ulcerative colitis

Yearly colonoscopies

what ortho tests help prove shoulder tendonitis/

abbot-Saunders (same motion for anterior apprehension), Speeds, Yergason (transverse bicipital tendon)

what muscles are inhibited in lower cross?

abdominals, gmax, hamstrings

What is a segment or group of segments that move in an inconsistent manner in the corresponding area?

aberrant motion

adynamic ileus aka paralytic ileus

absent bowel sounds excessive gas on x-ray?

what are contraindications of Infrared?

acute, inflammation, abnormal thermal sensation

what are the contraindications for diathermy?

acute, metal and epiphyseal centers

Ulnar N

adductor pollicis

NP/BP/HA

adjust

a chiled ingests an unknown amount of aspirin. if the child is conscious, the poison control center is likely to tell you to do what?

administer activated charcoal

what color is air, fat, muscle/water bone and metal?

air = black fat = lighter black muscle/water = gray bone = white metal bright white

subchondral cyst

aka geod osteoporosis

this part of the xray has less xrays produced smaller effective focal spot, more detail, used on thinner body part

anode side of xray tube

P/T ST tract

anterior cord

you have applied a pressure dressing and bandage to a hemorrhage and the dressing becomes blood soaked. you should what next?

apply addition dressings on top of it and hold firmly in place

leg is cold

arterial blood decrease

the most important step to take upon involvement in an emergency is to...?

assess the scene and environment

how doe you differentiate carpal tunnell syndrome from pronator teres syndrome

at elbow; +tinels at elbow, -phalens, dysestheis of palmar triangle, Pain on resistance of pronation

what conditions presents with tracheal deviation toward the affected side and decreased fremitus?

atelectasis and endobronchial tumor

what the fastest way to take a child's temp?

auricular (ear)

when caring for a fractured, dislocated or sprained extremity, when is it important to check for pulses, sensation, and motor function?

before and after applying a splint

what condition causes abnormal dryness and thickening of the conjunctiva and cornea due to vitamin A deficiency

bitot's spots

thing's to pay attention that may show up this round

blastic mets paget's psoriatic A gout fibrous dysplasia HME DJD/AVN legg Calves Perthes fibrous dysplasia osteochodritis dessicans

What anatomical structure does the National Listing System use as a reference?

body

What is the first letter in the national listing system referring?

body rotation

what is the contraindications of Traction?

bone weakness, lig weakness, acute injury

meniscal teat t/t

bounce home Apley's McMurray's

a patient presents with unexplained fainting spells, light headedness with touching of the anterior neck (shaving, applying make up), or even tight neck clothing. what is a possible dx?

carotid body sensitivity

the intensity of the xray is greater at what end of the anode?

cathode (negative) side

the most frequent poisoning happens in what age group

children

what condition presents with brodie's Abcess?

chronic osteomyelitis

what are the contraindications of cryotherapy?

circ compromise, frostibe, and raynauds

insidious

comes on slowly

Hutchison's teeth is usually associated to what condition?

congenital syphilis

If scoliosis is present what side of segment do you contact?

convexity

If there is a scoliosis the doctor stands on which side?

convexity

What plane of movement is seen in the lumbosacral facets?

coronal

What plane of movement is seen in the thoracic facets?

coronal

m/c/c/ bilateral AVN

corticosteroids

pop with knee and immediate swelling

cruciate ligament

what have you done to the FFD if the image gets bigger and blurrier?

decrease the FFD

what is the medical term for bed sores?

decubitis

stocking and glove

diabetes M B12 def PLS

chest pain and intermittent dysphagia to solids and liquids are common symptoms of what?

diffuse esophogeal spasm

what is the best method of controlling bleeding and should be attempted first?

direct pressure

if ice makes it better

don't use any heat or heating modality

vibration DC

dorsal columns

macular degen

drusen bodies

what muscles are facilitated in lower cross?

erector spinae, quads, iliopsoas

a branch of philosophy investigating how human actions can be judged as right or wrong is called what?

ethics

what is hirsutism

excessive hairiness on women in those parts of the body where terminal hair does not normally occur

kyphosis

extension rehabilitation

posterior cord syndromes

extremely rare - they don't usually ask about them

AS

eyes - uritis don't forget

rough borders

fibrous dysplasia

What is malposition?

fixated

Freiberg's

flattening of MT heads

What type of motion is seen in the lumbar facets?

flexion/extension

____________ deficiency in pregnant women can cause neural tube defects

folic acid (B9)

patient presents with pains in legs at night with glossitis and megaloblastic anemia. What is the deficiency?

folic acid (B9)

AP Towne's posit?

foramen magnum Cass = 10x12 FFD= 40 Tilt = 35 deg caudad (ft)

hemangioma

gen 1 vert maybe 2 see more =osteoporosis

a patient presents with a pale optic cup that is greater than one half of the optic diameter is characteristic of what condition?

glaucoma

When looking at the gravity weight line in an anterior view, what is being assessed?

head tilt or rotation, shoulder height, arm carriage, hip height, feet malposition.

structural scoliosis

hemivertebra

what does decreasing the kVp do

higher contrast (bone film) body absorbs more of the xray

Faster xray Screens = ______________ contrast = ______________ scale (____________ patient exposure)

higher; short; less large crystals (more light)

a patient exhibits a large smooth goiter with exopthalmos. what other test would be positive?

hyperreflexia

Chvostek's sign

hypocalcemia or Vit D latent tetany

pt presents with coarse dry hair, difficulty swallowing, fingernails bent upward toward the tips are s/s of what condition?

hypothyroidism

_____________ consent applies to you when you find a person unconcious on the sidewalk and bleeding an begin immediate tx

implied consent

to lower your contrast what would you do to your Kvp and MAs?

increase Kvp by 15% decrerease MAs by 30%

compensate for the inverse square law what needs to happen to the mAs if the FFD is increased?

increase the mA's 4 times

what is another name for Cubitus valgus and body part does it involve?

increased carrying angle

hyperkyphosis

increased lung capacity

what does increasing mAs do to your xray

increases film exposure

Nephritic

infection LBP, hematuria, proteinuria

fusion of joints

inflamm arth

meniere's dx

inner ear, vertigo, tinnitus ddx acoustic neuroma with facial weakness

what examinatino procedure can be performed from the first encounter to the end of pts visit?

inspection

what is the number one deficiency in the world?

iron

How do you differentiate Bells Palsy from Lyme Disease and Guillain Barre

it is localized to the face

pain does not go below knee

its referral think facet syndrome - mcnabb's, hadley S-curve, fergueson's extension makes it worse

DeMussett's sign is ___________ and is associated with _______________.

jerking and bobbing of the head; tremor or aortic insufficiency

Congestive heart failure

jugular vein pulsations ankle swelling and noctural cough

what is the primary method of controlling contrast?

kVp

What type of motion is seen in the lumbosacral facets?

lateral flexion

What type of motion is seen in the thoracic facets?

lateral flexion

The third letter in the National Listing System refers to what type of movement?

lateral flexion or wedging on the side of body rotation

What does the third letter in the Gonstead listing refer to?

lateral flexion or wedging on the side of spinous rotation

what condition is described as a persisting painless white patch in the oral mucosa and caries a possibiltiy of malignancy?

leukoplakia

you see chest movement in an unconscious pt after opening the airway. next you should...?

listen and feel for air exchange

what does increasing the kVp do?

low contrast (more shades of gray) less absorption of the body

Slower xray Screens = ______________ contrast = ______________ scale (____________ patient exposure)

lower; long; more small crystals (less light, more detail)

what is the prime method of controlling density of an xray

mAs

neurogenic

made better in flexion worse in extension

pop with knee and hours latter swelling

meniscus (collaterals- generally don't see swelling)

Gardner's syndrome

multiple polyps in GI with osetomas

what is faradic/sine wave used for?

muscle contraction to stretch fibrotic tissue and increase lymph flow

smooth borders and lisch nodules

neurofibromas

what condition displays Cafe-au-lait spots on lower lumbar region, buttocks and chest?

neurofirbrmatosis

Tender motor points is associated with what type of atrophy?

neurogenic atrophy; NOT disuse

what does OFD stand for?

object film distance

what is called when an interviewer sets aside preconceived conclusion about the patient?

objectivity

Do not do ultrasound

on a neuroma

metastatic

oncologist

menicus tx

open chain water resistance progressive

Generally Gonstead will contact which side of the wedge?

open wedge

What side of the segment do you contact assuming no scoliosis is present?

open wedge

BBT

orthopedist

Vischow'a lymph node

painless supraclavicular gastric carcinoma

what would be apparent upon visualization of no optic cup while the disc is hyperemica and displaced anteriorly?

papilledema

What is costovertebral or costotransverse disrelationship/sacroiliac subluxation characterized as?

paraspinal

Lumbar spine oblique posit?

pars Cass = 10x12 FFD= 40 Tilt = 0

what muscles are facilitated in Upper cross?

pec, upper trap, lev scap

_____________ is the blurry halo parts of th eimage (unsharpness)

penumbra

Lupus

photosensitive normochromic, normocytic often involves Kidney

Bonnet's Test

pirifomis

what condition presents with tracheal deviation toward the unaffected side and decreased fremitus

pleural effusion

increased fremitus and no tracheal deviation are usually physical exam findings associated with which condition?

pneumonia

osteitis condensans ilii

preg female or overweight male

an act of kindness that results in a positive outcome or removal of harm describes what principle?

principle of beneficence

M/C/C blastic mets

prostate

the medical term for itching is what?

pruritus

what type of deficiency causes cheilosis (sore cracks at the corner of the mouth)

riboflavin (Vit B2)

while performing CPR the patient begins to vomit. you should do what next?

roll the patient on his side, clean out his mouth, and reassess airway, breathing and circulation

What type of motion is seen in the cervical facets?

rotation

what childhood disease has a nondiscriptive maculopapular rash of short dura tion of 2-3 days with a 14-20 day incubation period

rubella

what is another name for measles?

rubeola

What plane of movement is seen in the lumbar facets?

sagittal

squamous means what?

scaly or plate like

De Quervain's

scaphoid or navicular Finkelstein's test

What is a scoliosis and/or alteration of curve secondary to muscular imbalance?

sectional intersegmental

What is a restricted or decrease segment?

segment can't move there

inflamm arthritis

send to rheumatologist

cervical myelopathy

sharp and shock like symptoms down arm like MS

what are xanthelasmas

sharply demarcated yellowish cholesterol deposit of fat underneath the skin, usually on or around the eyelids

all BBT

short zone of transition

Diathermy is comprised of two types of wave forms

shortwave and microwave

what is the anode heel affect?

since the anode produces less dense xrays it is better to shoot the thinner areas of the body with anode half

why would you use UV light therapy?

skin and bone contions like bacterial fungal and metabolic defects

which type of xray screen gives more detail?

slower xray screens

The Gonstead listing is based on what anatomical structure?

spinous process

In the sub-occipital triangle, where is the Rectus Capitus Posterior Major located?

spinous process of Axis to Occiput

In the sub-occipital triangle, where is the Obliquus Capitus Inferior located?

spinous process of Axis to transverse process of Atlas

The second letter of the Gonstead listing refers to what movement?

spinous rotation

what is the most common bacteria seen in osteomyelitis and septic arthritis?

staphylococcus aureus

What is a fixed segment?

stuck

small bubble of air trapped under the skin of the neck and chest region and usually cause by pneumothorax are called what?

subcutanteous emphysema

what vital sign is taken the least in adult patients?

temperature

what space is evaluated when determining whether the trachea is midline?

the space bw the trachea and SCM muscle

with HPT

they will never show you just one bone

What is Wernike Kosakoff syndrome?

thiamine (vit B1) deficiency from alcoholism

popliteal fossa

think baker's cyst seen with RA

hear asthma

think corticosteriods leads to pathological fracture

two week duration of baldndess on the top of the head which is "patchy" and is very contagious especially in school children?

tinea capitis

Lateral L/S posit?

top of iliac crest Cass = 14x17 FFD= 40 Tilt = 0

which condition is an acute febrile illness with a fulminiant course and does not produce tetanospasm?

toxic shock

What Plane of movement is seen in the cervical facets?

transverse

bloody diarrhea

ulcerative colitis

never immob shoulder

unless fracture dislocation AC separation

insidious LBP GI problems AS

uveitis anderson lesion carrot stick fx

what is the contraindication of massage?

vascular problems

leg warm

venous back up

multiple myeloma

vert plana path fx gen osteoporosis rev AG ratio Bence jones proteinuria ^ ESR M-spike electrophoresis renal failure

What is the reference structure of the vertebrae unless otherwise noted?

vertebral body

depigmented macules that lack melanin are known as what condition?

vitiligo

what are small benign growths caused by a viral infection

warts

How is palpatory systolic BP taken?

with sphygmometer

spontaneous pneumothorax

young man with emotional stress


Related study sets

Quiz 8: Managing Interpersonal Conflict

View Set

STUFF YOU NEED TO KNOW TO PASS THE AP TEST HOMES

View Set

Psychology Final Chapters 8,10,11,12,13

View Set

MD 1001-1200 ចម្លើយពេញ

View Set

Introduction to Ethics Final Review

View Set